Jump to content

Wikipedia:Reference desk/Science: Difference between revisions

From Wikipedia, the free encyclopedia
Content deleted Content added
Line 201: Line 201:
:[[Tardigrade]] <span style="font-size: smaller;" class="autosigned">—Preceding [[Wikipedia:Signatures|unsigned]] comment added by [[User:TrialicWave|TrialicWave]] ([[User talk:TrialicWave|talk]] • [[Special:Contributions/TrialicWave|contribs]]) 18:59, 7 March 2010</span><!-- Template:Unsigned2 -->
:[[Tardigrade]] <span style="font-size: smaller;" class="autosigned">—Preceding [[Wikipedia:Signatures|unsigned]] comment added by [[User:TrialicWave|TrialicWave]] ([[User talk:TrialicWave|talk]] • [[Special:Contributions/TrialicWave|contribs]]) 18:59, 7 March 2010</span><!-- Template:Unsigned2 -->
:If you're after animal species that are anatomically unchanged for several million years, then the [[living fossil]] article has several examples. [[Special:Contributions/62.56.68.180|62.56.68.180]] ([[User talk:62.56.68.180|talk]]) 01:20, 8 March 2010 (UTC)
:If you're after animal species that are anatomically unchanged for several million years, then the [[living fossil]] article has several examples. [[Special:Contributions/62.56.68.180|62.56.68.180]] ([[User talk:62.56.68.180|talk]]) 01:20, 8 March 2010 (UTC)

:I've heard [[triops cancriformis]] called [[Triops#Geological_history|the oldest animal species known to still exist]]. [[Special:Contributions/86.178.167.166|86.178.167.166]] ([[User talk:86.178.167.166|talk]]) 01:03, 10 March 2010 (UTC)


== Extra [[Life expectancy]] for the health-concious ==
== Extra [[Life expectancy]] for the health-concious ==

Revision as of 01:03, 10 March 2010

Welcome to the science section
of the Wikipedia reference desk.
Select a section:
Want a faster answer?

Main page: Help searching Wikipedia

   

How can I get my question answered?

  • Select the section of the desk that best fits the general topic of your question (see the navigation column to the right).
  • Post your question to only one section, providing a short header that gives the topic of your question.
  • Type '~~~~' (that is, four tilde characters) at the end – this signs and dates your contribution so we know who wrote what and when.
  • Don't post personal contact information – it will be removed. Any answers will be provided here.
  • Please be as specific as possible, and include all relevant context – the usefulness of answers may depend on the context.
  • Note:
    • We don't answer (and may remove) questions that require medical diagnosis or legal advice.
    • We don't answer requests for opinions, predictions or debate.
    • We don't do your homework for you, though we'll help you past the stuck point.
    • We don't conduct original research or provide a free source of ideas, but we'll help you find information you need.



How do I answer a question?

Main page: Wikipedia:Reference desk/Guidelines

  • The best answers address the question directly, and back up facts with wikilinks and links to sources. Do not edit others' comments and do not give any medical or legal advice.
See also:


March 5

Water for Hydrogen Fuel in Deserts (cont.)

I am continuing from my last question.

Who has proposed using an energy store in deserts? I have recently read a book called Solar Hydrogen Energy: The POWER to save the Earth which proposed that hydrogen be used to transport solar energy from deserts to other places. It also proposed that most production of solar energy be in deserts so they won't take up farmland.

The water in oceans is saltwater. Could we extract the hydrogen used to transport energy from saltwater?

Most combustion of hydrogen as fuel would be in places outside deserts, places all over the world. They would be very far away from deserts. Alot of the combustion of hydrogen would be in transports and not in producing electricity so they wouldn't be transported back to deserts.

An Unknown Person (talk) 04:44, 5 March 2010 (UTC)[reply]

There have been lots of proposals to use hydrogen as a storehouse for excess energy production from solar, wind, hydroelectric, etc. It will likely eventually become a common fuel source, however there is an utter lack of hydrogen-fuel infrastructure (basically, no fuel stations exist to sell the stuff; no distribution network exists to get it to the stations, etc.) the way there is with gasoline. So you get a catch-22 with hydrogen as a fuel: No one owns hydrogen-fueled cars because you can't get hydrogen anywhere to fill the tank with, and you can't find hydrogen stations anywhere because there are no customers with hydrogen cars to buy them from. This barrier is what has kept fuel cell vehicles off the road, despite being a feasible means of fueling a car for, oh, 50 years or so. --Jayron32 04:56, 5 March 2010 (UTC)[reply]
That doesn't mean it's impossible. I imagine you could start with solar hydrogen production facilities near desert cities, like Las Vegas and Phoenix, and provide refueling stations in those cities and maybe a fleet of rental cars that use hydrogen. You could eventually expand the network with hydrogen pipelines to cities near the desert, like Los Angeles. It might never be feasible to ship hydrogen all the way across the country to places like New York, however, depending on the prices of other fuels. StuRat (talk) 12:32, 5 March 2010 (UTC)[reply]
The idea isn't to "ship" hydrogen - it's to use it as a storage medium. The cheapest way to "ship" hydrogen is to burn it to make electricity, send the electricity over wires to the destination and use that electricity to make hydrogen at the other end. Sure, it's inefficient - but it's a LOT easier and safer than building hydrogen pipelines or having trucks or trains carry the stuff. SteveBaker (talk) 20:08, 5 March 2010 (UTC)[reply]
You may be right about the advantages of each approach, but the Q was clearly: could "hydrogen be used to transport solar energy from deserts to other places" ? So, that's what I answered. StuRat (talk) 21:24, 5 March 2010 (UTC)[reply]

What does the integral of the position function represent?

That is, suppose we have velocity v'. If we integrate (with respect to time), we get position v + c. What do we get if we integrate again?--70.122.117.52 (talk) 05:06, 5 March 2010 (UTC)[reply]

The integral of the velocity is displacement, not position. That is, if we integrate a velocity function between any two arbitrary points in time, you get the distance traveled over that time. I'm not sure that integrating the displacement function gets you any meaningful physical quality. Mathematically (especially if you have a complicated velocity function) you could repeatedly integrate any function until you get only a constant left, but that doesn't mean that the operation produces a meaningful physical result. --Jayron32 05:18, 5 March 2010 (UTC)[reply]
(ec)The definite integral of velocity is displacement. The OP is talking about the indefinite integral, which would be position if you take the constant as being the starting position. Also, if you repeatedly integrate you don't end up with a constant - that's repeated differentiation. If you repeatedly integrate you end up with some function plus a polynomial (of ever increasing degree). --Tango (talk) 05:32, 5 March 2010 (UTC)[reply]
I don't think the integral of position wrt time has any physical significance. It would have units of length times time (eg. metre seconds), which doesn't correspond to anything useful that I can think of (an angular momentum divided by a force, I suppose, but I can't think why you would ever do that). --Tango (talk) 05:32, 5 March 2010 (UTC)[reply]
It could have a meaning if there is some other property dependent on the position (∫f(x)⋅dt rather than just ∫x⋅dt). Two ideas:
  • A compressed spring. Force is based on displacement, so k⋅distance⋅time is the impulse or momentum-change of the object against which the spring is pushing.
  • Some other sort of energy transfer where the transfer/flux falls off linearly over distance. Total transfer is like k⋅(1-distance)⋅time. Hrm, actually all the specific ones I can think of are squared relationships not linear:( Something involving a dashpot/damper or other particle motion through viscous material I guess.
DMacks (talk) 21:39, 5 March 2010 (UTC)[reply]

Why would you only end up with a constant through iterated integration? Wouldn't that be what happens through iterated differentiation? —Preceding unsigned comment added by 70.122.117.52 (talk) 05:31, 5 March 2010 (UTC)[reply]

Yeah yeah yeah. Stop piling on. --Jayron32 05:34, 5 March 2010 (UTC)[reply]
Two people edit conflicting does not a pile make! --Tango (talk) 05:38, 5 March 2010 (UTC)[reply]
That depends on the size of the people. I have met people who constitute a pile all by themselves. --Jayron32 06:19, 5 March 2010 (UTC)[reply]
Fair point. I don't know about the OP, but I'm pretty skinny - I think you would need at least 3 or 4 of me to constitute a pile. --Tango (talk) 06:24, 5 March 2010 (UTC)[reply]
Neither iterated integration, nor iterated differentiation guarantee that you'll end up with a constant. Even with iterated differentiation, that only happens for polynomials. —Preceding unsigned comment added by 157.193.173.205 (talk) 08:22, 5 March 2010 (UTC)[reply]
The integral of displacement is sometimes called absement - our article on it has been deleted on account of its low notability, but here is a short-but-good explanation of the concept. --Link (tcm) 22:55, 6 March 2010 (UTC)[reply]

Sun

Could a star like the earths sun just collapse in on itself, like due to chaos theory or something? —Preceding unsigned comment added by Jetterindi (talkcontribs) 09:27, 5 March 2010 (UTC)[reply]

No. The sun is not massive enough to become a neutron star or a black hole. In about 5 or 6 billion years it will become a red giant, and then finally a white dwarf surrounded by a planetary nebula. See stellar evolution for more details. Gandalf61 (talk) 09:39, 5 March 2010 (UTC)[reply]
I heard this same thing in grade school about thirty years ago. Don't you mean "4,999,999,970 - 5,999,999,970 years"? Kingsfold (talk) 14:19, 5 March 2010 (UTC)[reply]
Also, "chaos theory" doesn't mean any crazy thing you can think of might happen. It just means that tiny variations in the beginning of an event can have large and unpredictable effects on the outcome. --Sean 14:19, 5 March 2010 (UTC)[reply]
Perhaps the OP is asking whether all the particles in the Sun could move inwards due to random chance (although this could apply to any bunch of matter at >0K). The probability is unfathomably low. --Mark PEA (talk) 11:53, 6 March 2010 (UTC)[reply]

Thinking brain?

The idea that our thinking activity is located inside the head seems very natural, but is it something we have learned, or do we know it because we feel the brain thinking? (like e.g. we feel our belly digesting the food). If we have learned it, when did men first have this intuition, and how did they came to it? --pma 10:07, 5 March 2010 (UTC)[reply]

comment removed. -- kainaw 21:36, 5 March 2010 (UTC)[reply]
You obviously saw it in a fictional movie (if your memory is correct), since a head transplant is totally beyond the abilities of science men. 82.113.121.110 (talk) 11:25, 5 March 2010 (UTC)[reply]
According to the Head transplant article, the procedure has been Performed with limited success on dogs, monkeys and rats.... Mitch Ames (talk) 11:34, 5 March 2010 (UTC)[reply]
even worse, if that was the end of the "experiment", it only proved: a thinking activity was either in the first monkey's head, or in the second monkey's body, and nowhere in the "scientist". But my question is more on the psychological side: how is that we feel that our thinking activity happens inside our head: is it cultural, that is we learn it when we are kids, or is it physiological, that is, we know what part of our body is thinking much the same way that e.g. we know which part of our body is eating or defecating. --pma 11:45, 5 March 2010 (UTC)[reply]
Scientists probably first knew this from people with head injuries, causing brain damage, which led to abnormal thought processes (speech problems, etc.). But people might have thought this intuitively because our perception is associated with our senses, and 4 out of 5 are located on the head alone. This is a rare case where the intuition was right on. One exception seems to be emotion, which instinct told people was in the heart, since it beats faster or slower based on our emotional state. However, emotion, just like logic, actually comes from the brain. StuRat (talk) 12:18, 5 March 2010 (UTC)[reply]
History of neuroscience has some useful information. The ancient Egyptians believed intelligence was situated in the heart; Alcmaeon of Croton (c. 500 BC) is claimed to be the first person to suggest the brain was used for thinking. Galen (AD 129 – 199/217) also did work on brain structure. Although little is known about him, Alcmaeon probably used vivisection and dissection to make his discoveries, e.g. cutting the optic nerve of live animals. --Normansmithy (talk) 12:24, 5 March 2010 (UTC)[reply]

if thez thought intelligence was situated in the heart, what did they think the material in the skull was for? Also, presumably they knew what some of the other organs did because people injured in those organs all suffered in the same way as a result. Didn't a single person in ancient times suffer traumatic head injury that caused brain damage, so it became obvious to everyone that intelligence and cognitive functions are done there? I seriously can't imagine how they could have thought brain-stuff was for anything else than thinking... what did they think it was? 82.113.121.110 (talk) 13:06, 5 March 2010 (UTC)[reply]

The first article linked by Normansmithy above answers your questions directly. Remember that for much of the times under discussion, there were almost no means of communication other than face-to-face, few libraries or other centres of learning existed, and very few people were literate anyway: consequently, most people's opinions could only be based on their own direct experiences. While some did deduce that the brain was the seat of thinking, some thought instead that the brain was merely packing material, and some (following Aristotle's opinion) that its function was to cool the blood which, since the brain does indeed have a very generous blood supply and does indeed create and radiate a significant proportion of the body's heat was, though wrong, not at all unreasonable. 87.81.230.195 (talk) 13:52, 5 March 2010 (UTC)[reply]

The best discussion of this issue that I've seen is in The Mind's I, in two essays called "Where am I?" (by Daniel Dennett) and "Where was I?" (by David Hawley Sanford). I think the basic answer is that we feel ourselves to be located in our heads because that's where our eyes are -- we are vision-dominated creatures. By using remote-vision equipment it is possible to create a very strong sense of being located outside your body in various ways. Looie496 (talk) 17:11, 5 March 2010 (UTC)[reply]

Thany you all! very interesting information. --pma 08:35, 6 March 2010 (UTC)[reply]

Goedel and Wikipedia

Does the Goedel incompleteness theorem imply that insofar as Wikipedia is comprehensive, it cannot be accurate, and insofar as it is accurate, it cannot be comprehensive? 82.113.121.110 (talk) 11:30, 5 March 2010 (UTC)[reply]

Gödel's incompleteness theorems deals with questions about the fundamental definitions of maths. It is quite a stretch to apply it to wikipedia, without taking more poetic licence than is normally allocated to pure maths. I think it is nonsense to suppose that wikipedia will ever be complete (even if we knew what that meant in wikipedia terms) not completely accurate. We can duduce all of that without the need to appeal to Gödel. --Tagishsimon (talk) 11:43, 5 March 2010 (UTC)[reply]
Wikipedia will not be complete until it contains an article that is about itself. Looie496 (talk) 17:02, 5 March 2010 (UTC)[reply]
Please see our article Wikipedia, which is both (IMHO) accurate and comprehensive. So there you go - Goedel disproved, once and for all. Phew! --NorwegianBlue talk 19:15, 5 March 2010 (UTC) [reply]
Gah, lexical scope ambiguity! I didn't mean an article about Wikipedia, I meant an article with a title something like This article. Looie496 (talk) 19:40, 5 March 2010 (UTC)[reply]
Gödel's theorem relates to provability in a formal axiomatic system. It has nothing whatever to tell us about completeness (or truth) in the phenomenal world. --ColinFine (talk) 17:32, 5 March 2010 (UTC)[reply]
The problem is the definition of "comprehensive".
  1. To truly contain all knowledge (and thereby to be complete) we would have to carefully document (for example) the precise position of every object in the universe and keep that information up to date on a moment-by-moment basis. For truly comprehensive information, we'd require to provide the location of every fundamental particle. The required data storage for such a system would be larger by far than the universe itself - and is therefore quite impossible. This is a kind of "diagonal" argument that Godel and Cantor would have been happy to provide...but it's unrelated to Godel's famous theorem. Hence, true "completeness" is indeed impossible...not just impractical.
  2. Fortunately, Wikipedia's own definition of "comprehensive" requires us only to describe things that are "notable" and "referenceable". It probably is theoretically possible to be fully comprehensive within that definition. If every human on earth were to sit down and write articles about every single notable thing that happened to them personally, or which they had written about in a document that Wikipedia would accept as a reference - then they could easily do it within their lifetimes. Most people would be done with it very quickly...and those people would have plenty of time to fill in the notable/referenceable information from past generations...especially since the world population is much larger than it was in previous generations. Doing that would result in a complete, "comprehensive" encyclopedia (within our own definition of that word) in rather a short period of time.
SteveBaker (talk) 19:42, 5 March 2010 (UTC)[reply]
First, Wikipedia isn't written in a formal language, so Godel's Incompleteness Theorem doesn't apply. If it were, it would be more like a big list of facts ("1+54 = 55" "7 is prime" "65=65") than a small set of axioms (like the nine Peano axioms, that start "for all x, x=x"). Incompleteness is about the limits of derivable facts. Any list of facts (about an interesting system) is trivially incomplete by virtue of not being infinitely long (but an infinitely long list of facts totally could be complete.) Paul Stansifer 23:08, 5 March 2010 (UTC)[reply]
Exactly. For example true arithmetic is complete. True arithmetic satisfies all the stipulations for incompleteness, except the one saying that it can be axiomatized by a computably enumerable set of axioms.
Basically the whole endeavor of trying to apply incompleteness outside of mathematics is fraught with pitfalls. That's not to say it can never be done, but it is to say that some very smart people have wound up making unsound arguments of this sort. Torkel Franzén has a whole book on the subject. --Trovatore (talk) 23:15, 5 March 2010 (UTC)[reply]

do we benefit in any way from c?

Is there any benefit for anyone from having a current universal speed limit of c? ie, if the Universe were a democracy, and its lawmaker obeyed the will of the people in it, is there any reason we would do well to have the lawmaker keep c? At least two good reasons for abolishing c from the laws of the universe would be: 1) easier intergalactic space travel, easier communication with probes on mars, etc, 2) smaller ping times to China, and faster processing within a single piece of electronics. In fact, in today's 3 GHz processors, I heard that electrons only have enough time between clock ticks to travel a few centimeters (you can verify this for yourself with a simple Google calculation). Now this means we can't possibly make processors much bigger than that, with logic that has electrons travelling far more than that distance, depending on what path they take. So, this might be a simplification, but abolishing c could also bring better computation power. But as I asked above, my question is now: is there ANY benefit at all from actually having c? 82.113.121.110 (talk) 12:41, 5 March 2010 (UTC)[reply]

Your question is nonsense. c is a fundamental constant. Okay, technically there are some limited theoretical circumstances where it bends (shortly after the Big Bang), but we have no control over it. Nature's laws are not up for a vote. —ShadowRanger (talk|stalk) 13:13, 5 March 2010 (UTC)[reply]
In English, when you say "if the Universe were", the were means that the speaker knows he or she is asking about an imaginary case that is contrary to actual reality. (Otherwise he or she would say "if the Universe turns out to be" or even "if the Universe really is"). So, your point is totally invalid, instead please answer my question: is there any benefit to c, and even though we can't, if we could petition to have this law abolished, would there be any practical benefit to us if we could and we did and it were? —Preceding unsigned comment added by 82.113.106.97 (talk) 13:22, 5 March 2010 (UTC)[reply]
There are obvious benefits, as you listed. The question is whether the universe could still exist (in any way still somewhat pleasant to us) if we raised c. After all, it's not a speed limit in isolation of the rest of physics. Fine-tuned universe addresses variations on the question. --Sean 14:32, 5 March 2010 (UTC)[reply]
Thanks. Unfortunately your benefit to c in that without it perhaps the Universe "couldn't exist in a way still pleasant to us" is so very broad. Could you, or anyone else, possibly give a more direct, narrow benefit to having c as a speed limit? Does this actually help in a practical, specific (rather than an overarching universal) way? Thanks. 82.113.121.103 (talk) 14:44, 5 March 2010 (UTC)[reply]
The slow speed of light has so far prevented the evil Zorn empire from invading from their galaxy and eating our brains. :-) StuRat (talk) 14:42, 5 March 2010 (UTC) [reply]
But, seriously, there may well be (or have been) other intelligent species out there which would, at some point, have colonized the Earth if they could only get here, meaning we may never have existed. We don't need to attribute evil motives to them, as they might not have found any intelligent life if they arrived long ago, but maybe they wondered what that slime was in those volcanically heated ponds. StuRat (talk) 14:48, 5 March 2010 (UTC)[reply]
This is exactly the type of answer I'm looking for. Do you know of any other answers of this specific nature? 82.113.121.103 (talk) 14:44, 5 March 2010 (UTC)[reply]
Perhaps next time, you could specify which type of answer you are looking in front, and save us and yourself some time? :-) DVdm (talk) 14:53, 5 March 2010 (UTC)[reply]
um, I didn't have any idea of this answer. I like it because it is specific, and OBVIOUSLY a benefit. I would easily pay any amount of my money not to have my brain eaten by Zorns. Are there any other, specific benefits like this you can list? 82.113.121.103 (talk) 14:58, 5 March 2010 (UTC)[reply]
George Gamow wrote a little book "Mr. Tompkins in Wonderlandwherein the Mr. Tompkins dreams about a world where the speed of light was 30 miles per hour. Relativistic effects are seen when someone rides a bicycle. Surely someone has written a similar work where c was orders of magnitude higher. Radio antennas would get bigger, at least for the same frequency. Optics might have to change their size, at least to focus the wavelengths used presently in vision and photography. I wonder if electron orbitals would have to change, along with the the size of atoms and molecules? Edison (talk) 15:03, 5 March 2010 (UTC)[reply]
Changing c would change the fine structure constant, and Fine-structure_constant#Anthropic_explanation suggests that that would result in a very different, and probably inhospitable, universe. Also, if you are talking about an infinite c (instead of just a larger c), I'm not sure you would have electromagnetic waves at all anymore. -- Coneslayer (talk) 15:11, 5 March 2010 (UTC)[reply]

Okay, the above is a real benefit: smaller radio antennas than if C were larger. Then Cones Layer says the same thing others have been saying, that it's a general requirement for our whole universe. Guys, I got this part. Are there any other specific benefits to the current c, as the bit about smaller antennas is? Thanks. 82.113.121.103 (talk) 15:38, 5 March 2010 (UTC)[reply]


I can think of a couple of Japanese cities that would not have suffered as much if the value of c in E=mc2 were not so large. TimBuck2 (talk) 17:07, 5 March 2010 (UTC)[reply]

Awesome answer. We have all benefited, since if c were larger we may have destroyed all of Japan, or even the entire earth with the first atomic test. Also, if c were a lot smaller, I imagine that we wouldn't be able to generate much electricity through nuclear reactors.24.150.18.30 (talk) 17:44, 6 March 2010 (UTC)[reply]

new Best answer (so far) as chosen by OP:Reduced from a spurious subheading that intruded in the contents list. Cuddlyable3 (talk) 18:09, 6 March 2010 (UTC)[reply]

As you increase c, I think it would get harder to implement the Global Positioning System with the same accuracy. Obviously it wouldn't work at all with infinite c. If you must split my name in two, it should be Cone Slayer, not Cones Layer. -- Coneslayer (talk) 15:43, 5 March 2010 (UTC)[reply]
Along the same lines, it would be more difficult for us to precisely measure the distance to the moon. —Bkell (talk) 15:54, 5 March 2010 (UTC)[reply]
the GPS is the best practical answer anyone here has given so far, and it is spot on, since it relies on specific timing of distances that are travelled at or nearly at c. Therefore, with an infinite or much larger c, this would become difficult. Can anyone come up with other practical aspects of our life that could not work but for c on the scale it currently is? Thank you. 82.113.121.103 (talk) 16:52, 5 March 2010 (UTC)[reply]
You're pretty insistent, aren't you? —Bkell (talk) 18:00, 5 March 2010 (UTC)[reply]
This is a meaningless question. If 'c' were even slightly different (either more or less) than it actually is, then humans would not exist...but radically different life-forms might. If it were significantly different then probably galaxies and stars wouldn't exist. One of the things you learn after enough years answering questions on the reference desks is that once one utter impossibility has been injected into a question, all else falls to the ground. We can't meaningfully list trivia like GPS being more or less accurate when the elephant in the room is that the existence of all things pretty much depends on 'c' being precisely what it is. GPS could not possibly function in any way whatever if 'c' were more than a percent or two different than it actually is because nobody would have been here to invent it...and in all likelyhood, there wouldn't even be a "here". Sorry - but you don't get to pick between answers you like and answers you don't. You get answers...hopefully true ones. SteveBaker (talk) 19:30, 5 March 2010 (UTC)[reply]
I thought he showed outstanding judgment and discernment with his choice of answer. -- Coneslayer (talk) 19:35, 5 March 2010 (UTC)[reply]


If c were infinite, and the universe were also spatially infinite, then the sky would be blindingly bright 24/7, would it not? Vranak (talk) 21:44, 5 March 2010 (UTC)[reply]
thank you for the response, can you explain your thinking in more detail, specifically what causes the sky to be brighter than it is now? thank you. 82.113.121.94 (talk) 21:57, 5 March 2010 (UTC)[reply]
Olbers' paradox --ColinFine (talk) 00:24, 6 March 2010 (UTC)[reply]

If you look up in the sky there is a giant thermonuclear furnace that relies on E=mc2. If you start playing with that c you could either turn off or explode the sun. Even a ~5% change in solar luminosity would change the temperature on Earth about 10 C, so that isn't a balance to be trifled with. Dragons flight (talk) 02:09, 6 March 2010 (UTC)[reply]

OK, I'll bite. Your question presumes that a "benefit" must be a "benefit" to "us," i.e., people. We have existed only a fraction of a million years, whereas the universe is 13.8 billion years old. So obviously nothing in the universe exists for "our" benefit. Perhaps you mean "life in general" -- does c benefit life? If light had no speed limit, then light would be infinitely fast. If so, every form of life in the universe would be blind, because all the light in the universe would endlessly travel around the universe; nothing could evolve "eyes," because no organic organ (developing from a primitive predecessor) could adjust to infinite stimulation. So, yes, the speed of light helps "us" because "we" like to be able to see things, which "we" couldn't if "we" were incapable of evolving optical organs. 63.17.82.123 (talk) 04:06, 6 March 2010 (UTC)[reply]
I disagree completely. Why would there be any more light, on average, at any given point ? We would see the light emitted from Proxima Centauri now, instead of the light it sent out some 4.2 years ago, but how would that change the total amount of light we see from that source ? Now apply that same logic to every other light source. If you're thinking there are an infinite number of stars out there, and the light from most of them hasn't reached us yet, due to a finite value of c, I don't think that's right. I believe there's a very large, but still finite, number of stars. Also, light doesn't travel an infinite distance, as eventually it gets absorbed by something, like interstellar dust. StuRat (talk) 16:12, 7 March 2010 (UTC)[reply]
Just for kicks, Wikipedia has a page on the variable speed of light (http://en.wikipedia.org/wiki/Variable_speed_of_light) which has some information on cosmologists investigating the possibility of c not always being what it is known as today.24.150.18.30 (talk) 17:52, 6 March 2010 (UTC)[reply]

Entropy

Is the Moon a higher or lower entropy environment then the Earth? What caused it? TheFutureAwaits (talk) 12:59, 5 March 2010 (UTC)[reply]

Our article on entropy suggests the definition "entropy is as such a function of a system's tendency towards spontaneous change." As such, the Earth is a lower entropy system than the Moon, as it is more prone to spontaneous change, and it is such primarily because it is larger -- large enough to retain an atmosphere and an active volcanic system. — Lomn 14:06, 5 March 2010 (UTC)[reply]

I think it's the opposite way : low entropy means highly ordered, for example instead of a moon, a perfect sphere made entirely of a single element and uniformly a single temperature would be highly ordered: you could basically describe it entirely in half a sentence, giving the diameter, the element it is made of, and the temperature (maybe I'm leaving out one or two things). Because you can describe it in very few words, it therefore has a very low entropy. Now the moon has much higher entropy than a perfect sphere made of a single element. You would need far more space to describe it fully. But the earth has a higher entropy still: it is much more complex. So, I would say that the Earth is a higher-entropy environment. To put it another way, as a percentage, you increase entropy far more when you put an American flag into the low-entropy conditions on the moon than when you place on in the high-entropy conditions on an Earthly mountain. Can someone better versed in math and science confirm my interpretation? Thanks. 82.113.121.103 (talk) 14:30, 5 March 2010 (UTC)[reply]

That's completely wrong. Entropy is not a measure of how many words it takes to give a macroscopic description of an object. It is measure of how many microscopic states are consistent with the macroscopic description. To labor on your example, if you were to melt the moon, mix if thoroughly and find a way to cool it fast enough to keep the mix uniform the final sphere would have a higher entropy. Read entropy of mixing. Dauto (talk) 15:42, 5 March 2010 (UTC)[reply]
I find this extremely hard to believe. You are telling me if we took the universe, melted it all together, and made a black hole out of it, with precisely 0 information in the black hole other than maybe it's total mass (a single real number, in grams) and MAYBE one or two more variables such as it's spin and charge (maybe) then there would be MORE entropy in Universe (even though you can just describe it as "1 black hole, in the "center" (ha ha) of nothing else, having mass x, charge y, and angular momentum z". Even if you give all of these to an obscenely unrealistic level of exactness, you still will use maybe a paragraph of digits. A paragraph, even using the best theoretically possible compression, is not enough to accurately describe (ie represent a compressed version of) even a single book (say, a collection of Shakespeare plays). So it seems to me that a SINGLE book would have more entropy than all of the universe, if you reduced the universe to a black hole. Likewise, it seems to me that a SINGLE city on Earth would have more entropy than the Moon, if the moon were a uniform substance you can perfectly describe in a few words. If I really am wrong, maybe it's because I'm conflating physical entropy with information entropy? For me, the fewer words you can use to give a second God in a different Universe enough information to fully reproduce an exact copy of something, the lower entropy it has. Our God would need to give a LOT of information to a second God in a different Universe to reproduce the Earth, but considerably less if the Earth were a uniform ball that is an exact geometrical sphere, of fixed temperature, density, etc. Don't you think? Can someone confirm whether I'm right, or whether Dauto above is right? Thank you. 82.113.121.103 (talk) 17:13, 5 March 2010 (UTC)[reply]
Yes, that's what I'm telling you. In fact for any given mass a black hole will be the state of maximum entropy. See black hole thermodynamics.Dauto (talk) 01:31, 6 March 2010 (UTC)[reply]
A small correction: I meant to say that for any given volume a black hole will be the state of maximum entropy. Dauto (talk) 04:20, 6 March 2010 (UTC)[reply]
These answers are just leading to more confusion. Dauto, is the Moon higher or lower entropy than the Earth? Why? TheFutureAwaits (talk) 15:56, 5 March 2010 (UTC)[reply]
If you take the planet as a whole I would venture that earth's mean specific entropy (entropy per unit volume if you will) will be higher simply because earth's core temperature is higher. Dauto (talk) 16:05, 5 March 2010 (UTC)[reply]

My wording may be off, what I'm getting at is which way is the energy exchange moving? So for example in the Sun-Earth system the Sun is increasing in entropy while the Earth is decreasing. How does this work in the Earth-Moon system? TheFutureAwaits (talk) 16:20, 5 March 2010 (UTC)[reply]

I doubt there's any meaningful give-and-take between the two. Entropy increases. — Lomn 16:36, 5 March 2010 (UTC)[reply]
I don't think the sun's entropy is increasing since the most important factor here is likely the fact that it is losing a massive amount of heat through radiation so its entropy is actually decreasing. The radiation exchange between the earth and the moon is not a very important factor since they are a similar temperatures which means radiation is moving in both directions. Dauto (talk) 16:40, 5 March 2010 (UTC)[reply]
Correct me if I'm wrong, but it seems relevant to highlight the increase in entropy of the system, e.g. as the sun radiates energy and mass. The mass that we call the sun one moment becomes a geometrically larger object the next, and the entropy of that system increases (while the entropy of the circumscribed orb we call "the Sun" may decrease). -- Scray (talk) 17:29, 5 March 2010 (UTC)[reply]

Probably the most important point is that theoretical thermodynamics shows that entropy per se is not very important -- what matters for understanding interactions is the dependence of entropy on energy, which is measured by a quantity we call the temperature. Looie496 (talk) 16:58, 5 March 2010 (UTC)[reply]

Since a black hole decays into hawking radiation, it can't be the state or maximum entropy, rather a bunch of randomly spread photons and leptons is a state of higher entropy, see Heat death of the universe and Black hole#Entropy and Hawking radiation 82.132.136.207 (talk) 00:32, 7 March 2010 (UTC)[reply]

The black hole is the state of maximum entropy for a given volume. The Hawking radiation that replaces the hole will ocupy a larger volume and can have a larger entropy then the hole as you've shown it must. Dauto (talk) 02:18, 7 March 2010 (UTC)[reply]

panda

Where can you find a giant panda in the United States? —Preceding unsigned comment added by Yelopiclle (talkcontribs) 14:02, 5 March 2010 (UTC)[reply]

From the Giant Panda article...

As of 2007, five major North American zoos have Giant Pandas:

  • Chapultepec Zoo, Mexico City – home of Xi Hua, born on June 25, 1985, Shuan Shuan, born on June 15, 1987, and Xin Xin, born on July 1, 1990 from Tohui (Tohui born on Chapultepec Zoo on July 21, 1981 and died on November 16, 1993), all females
  • San Diego Zoo, San Diego, California – home of Bai Yun (F), Gao Gao (M), Su Lin (F), Zhen Zhen (F), and Yun Zi (M).
  • US National Zoo, Washington, D.C. – home of Mei Xiang (F) and Tian Tian (M).
  • Zoo Atlanta, Atlanta, Georgia – home of Lun Lun (F), Yang Yang (M) and Xi Lan (M)
  • Memphis Zoo, Memphis, Tennessee – home of Ya Ya (F) and Le Le (M)

Googlemeister (talk) 14:18, 5 March 2010 (UTC)[reply]

Who's the best when nature calls?

Hi

1.I've often wondered when you have to go, (pee) but sometimes in a situation where you can't (whatever the reason might be). Who's better at holding it, males or females? -However I don't think females can hold it that long.

2. What are the complications of holding a number 2 for too long?

3. I'm sure most guys have had this happen to them somewhere along their lives. However if you've still somehow managed to elude this experience you're in for a treat. I've been hit a couple of times in the groin, but on one or two occations it hurt so bad that I felt I was going to need a new pair of undergarments.

3.1 Is this normal and what are the complications when the injury is serious?


Thanks, NirocFX

41.193.16.234 (talk) 14:05, 5 March 2010 (UTC)[reply]

Women can hold for slightly longer in tests, but it's negligible. There are no adverse affects to health of holing a poo in too long, you'll simply lose control of the bowls and crap your pants. And it's very normal to experience massive pain for both men and women when impacted on the genitalia area. They risks include hemorrhaging and sterility. R12IIIeloip (talk) —Preceding undated comment added 14:47, 5 March 2010 (UTC).[reply]
I disagree on "holding in poo". The large intestine removes water, and poo held in too long (several days) will thus be dried out and cause constipation. StuRat (talk) 15:37, 5 March 2010 (UTC)[reply]
Cecil Adams covered the potential hazards of holding in "Number 1" for too long here. Doesn't mention the other, though. APL (talk) 15:43, 5 March 2010 (UTC)[reply]

It's possible to get so constipated it comes out the wrong end. So you might want to hold your tongue too . . . or at least your breath. —Preceding unsigned comment added by 71.108.171.138 (talk) 23:32, 5 March 2010 (UTC)[reply]

Was that comment necessary? It is not pertinent to the discussion.--79.68.242.68 (talk) 01:24, 6 March 2010 (UTC)[reply]

Microbial locomotion time

If I am trimming the fat from some chicken thighs with a 5" chef's knife, how long could I expect it to take for the potentially harmful bacteria from the blade to make their way to the handle such that my right hand (holding the knife) should be reasonably assumed to be contaminated? I ask because I usually tend to perceive my right hand to be totally clean and use it without much discretion in terms of touching other things in the kitchen while preparing raw meat, such getting a pot from the cabinet, taking things from the fridge, etc. DRosenbach (Talk | Contribs) 16:40, 5 March 2010 (UTC)[reply]

Our Swarming motility article describes this as "rapid (2–10 μm/s) and coordinated translocation of a bacterial population across solid or semi-solid surfaces". If we take the upper end of that range (10 μm/s), then even if the bacteria were to move directly toward your hand, it would take more than 40 minutes for them to move an inch (or a couple of hours for a few inches). Add to that the less-directed nature of bacterial movement, the less-than-ideal culture conditions of a knife surface, and the fact that I assumed the upper end of the range of rates. For most users (perhaps not a trained health professional like yourself), it seems extremely unlikely that this mechanism would account for more contamination of your kitchen than a slip in technique (such as setting the knife down in a "clean" versus "dirty" area). -- Scray (talk) 17:13, 5 March 2010 (UTC)[reply]
I dislike that answer. By far the most likely way bacteria would get from something you're cutting onto your hand would be as a fine aerosol of particles sprayed from the object being cut. The mere act of cutting something is going to first stretch - then break and release fibers in the material. As they elastically return to their former shape, they could easily flick microscopic droplets from within the meat or whatever onto your hand. Also air currents and other things could easily be involved. Just measuring the speed a bacterium can move under its' own steam is not going to give you anything but a low-end estimate. I'd guess that the high end is probably 100 mph or something. SteveBaker (talk) 19:18, 5 March 2010 (UTC)[reply]
Not sure why you "dislike" that answer enough to make a point of saying so (twice). It's a direct answer to the OP, who asked whether the hand with which he holds the cutting knife is likely to get contaminated by direct spread of the bacteria. The question was not, "how do bacteria spread in the kitchen". I do like your conjectures - they seem plausible (except for the bit about bacterial coming from within intact meat). -- Scray (talk) 20:05, 5 March 2010 (UTC)[reply]

120 volt equipment

Hello, Q-what happens to 120 volt 60 hz equipment when it is plugged in to 120 volt 400hz power source? my theory is that it will run for awhile, but eventually it would overheat, example such as an electric motor that's 120 volt hz? —Preceding unsigned comment added by 205.200.77.222 (talk) 16:48, 5 March 2010 (UTC)[reply]

It's a difficult question because it depends entirely on the appliance. Somethings will die instantly, others will run perfectly happily, others will be somewhere inbetween. I live in the USA (60Hz, 120v) but have some things I brought with me from the UK (50Hz, 240v) - the cheaper kinds of converters convert to the correct voltage but put out the wrong frequency (ie 60Hz, 240v) and I have several gadgets that don't like that - one overheats, two others just don't work - and that's with just a 10Hz difference! A more expensive converter that I found corrects the frequency too - and that allows those gadgets to work OK. SteveBaker (talk) 19:12, 5 March 2010 (UTC)[reply]
In general, using electronics designed for low-frequency power on a high-frequency power input is safer than the other way around -- you don't need to worry about transformer cores saturating and overheating. Electric motors are one of the exceptions: if the input frequency is faster than the motor can spin, the motor won't move at all, and will act as a short circuit. --Carnildo (talk) 02:07, 6 March 2010 (UTC)[reply]
How about if it is a universal motor? Would a 120 volt universal motor run just as well on DC or 60 Hz or 400Hz? I would expect a purely heating appliance or an incandescent light bulb similarly should run on DC to 60 Hz to 400 Hz. Edison (talk) 21:28, 6 March 2010 (UTC)[reply]

Matter

I know matter can be converted to energy, but can it be converted to anything else? Dark matter perhaps?

Its not that matter can be converted into energy, like a magic trick. Its that matter and energy are different expressions of the same fundemental concept. Matter is merely energy which has been confined by the limits of a set of quantum numbers, but fundementally matter is energy and energy is matter. See Mass–energy equivalence for more information. --Jayron32 17:43, 5 March 2010 (UTC)[reply]
yeah, you might have heard of this equivalency as e=m times a constant which I forget. 82.113.106.100 (talk) 21:26, 6 March 2010 (UTC)[reply]

Ant

Which species of any has a stinger AND bites to inject formic acid into the wounds? —Preceding unsigned comment added by 518c&e (talkcontribs) 17:04, 5 March 2010 (UTC)[reply]

Species upper limit?

Have scientists discovered almost all species that exist? —Preceding unsigned comment added by SpiderLighting (talkcontribs) 17:09, 5 March 2010 (UTC)[reply]

As surprising as it might seem, only a minority of species have been identified. -RobertMel (talk) 17:15, 5 March 2010 (UTC)[reply]
Obviously we have not identified them all, but without knowing how many there are, how do we know we have more then 50% to go? Googlemeister (talk) 17:29, 5 March 2010 (UTC)[reply]
And in order to know if we've discovered them all, we'd first have to know how many there are. Which we don't. The species article has some things to say about this. Dismas|(talk) 17:18, 5 March 2010 (UTC)[reply]
Certainly not. Researchers have detected over 700 species of periodontal pathogens in the diseased gum tissue but have characterized and positively identified and marked less than 300 of them -- and that's just bacteria of the gums, let alone of the entire mouth and let alone the entire body and let alone the entire world. But in case you were referring to animal species, the answer would still be no. DRosenbach (Talk | Contribs) 17:21, 5 March 2010 (UTC)[reply]
Likewise, I have heard it speculated that if we counted up all of the discrete animal species we have identified, it would still be less than the number of beetle species yet unidentied. We are no where near ending the catalogueing of species. Furthermore, depending on how you define species, there are some life (bacteria) or pseudolife (virus) forms which speciate at a rate which means that we can get new ones within a human lifespan, meaning that we will never be done. Even if we confine ourselves to macrolife (plants & animals) we aren't even close to being done catalogueing them. --Jayron32 17:41, 5 March 2010 (UTC)[reply]
To add that many species will just vanish before we identity them. -RobertMel (talk) 18:04, 5 March 2010 (UTC)[reply]
The question of how to estimate the total number of species is interesting. One approach might be sampling. For example, if you take one small island and study it to death to hopefully identify every species, and find that there are 10 times as many as you knew initially, that might be some indication that we know less than 50% of the species worldwide. StuRat (talk) 17:51, 5 March 2010 (UTC)[reply]
I suppose you might also plot a graph of the number of newly discovered species per year (adjusted for the number of people hunting them and the amount of effort they put into it) and see if the graph was showing any signs of asymptoting out - which would be a clue that we were close to finding them all. However, (as others have pointed out) there is no way that we're 50% of the way through the process if you include microscopic stuff like bacteria and algea. That 50% number could only possibly be for things the size of insects and above. SteveBaker (talk) 19:06, 5 March 2010 (UTC)[reply]
Plus, the definition of the word "species" is far from settled, so you have to argue about that for another few hundred years before you even start counting beetles. --Sean 21:28, 5 March 2010 (UTC)[reply]
We've mapped maybe 3% of the floors of the oceans. It is unilkely that this is the coolest 3% out there. There are lots and lots of species in the oceans alone that we have not ever seen. Comet Tuttle (talk) 22:33, 5 March 2010 (UTC)[reply]
There are also likely millions of species of microorganisms in soil that we have not yet identified. Do those count? ~AH1(TCU) 04:01, 6 March 2010 (UTC)[reply]
Do you mean species on Earth? If there is life elsewhere in the universe, we are probably nowhere even close to discovering all of the species that exist.24.150.18.30 (talk) 18:01, 6 March 2010 (UTC)[reply]

For microorganisms, estimates of the number of species are made by counting the number of distinct DNA sequences in a sample of earth or water, and then doing calculations. This method yields numbers far higher than attempts to count species directly. Looie496 (talk) 18:07, 6 March 2010 (UTC)[reply]

relationship between entropy in physics and entropy in information science?

if you look at my above edit, you will see that I might be very confused indeed. Can someone help explain to me in simple terms what the relationship, if any, is between entropy as understood in physics and entropy as understood in information science? If there is no relationship, why is it the same word? Thank you. —Preceding unsigned comment added by 82.113.121.103 (talk) 17:19, 5 March 2010 (UTC)[reply]

We have an article about this. See Entropy in thermodynamics and information theory. In simple terms, entropy is used in both cases to refer to a loss of usability. In thermodynamics, it refers to the loss of Free energy in the universe, or in any closed system. In information theory, it refers to the loss of predictability or understandability in some bit of information. Its sort of like how both biology and chemistry use the word "nucleus" to mean "the bit in the center" (either a cell nucleus or an atomic nucleus.) However, the two entropies are far closer related than that. The mathematics of both is still controlled by the Boltzmann equation, which in information theory takes the slightly different form known as the Hartley function. There are also some gedanken experiments in physics which tie the two fields together nicely. Schroedinger's cat is essentially about information entropy (uncertainty in knowing the state of decay of a single particle is a form of information entropy, as is the fate of our poor cat). The paradox of Maxwell's demon, which conceives of an energy-less way of reducing entropy (and thus SHOULD be a violation of the second law of thermodynamics) can be resolved if we consider information to be negative entropy. I have a nice little "physics for laypeople" primer I should dig up which discusses the connections between information entropy and thermodynamic entropy. --Jayron32 17:37, 5 March 2010 (UTC)[reply]
Thank you. Again looking at my above edit, as I referenced at the start of this thread, could you try to understand the way in which I was trying to see black holes, planet-sized highly ordered/uniform/geometrically perfect and easily described objects, and the actual Earth, in terms of "entropy" (which I was using in more the informational sense) and tell me why it doesn't apply (if it doesn't) in a physical sense. ie someone in the linked thread said that I was wrong, and I wonder why (if it's true). Contrary to the implications you make above, physical and information entropy seems to me would have to be opposites in direction/sense in order for me to be wrong above... 82.113.121.103 (talk) 17:49, 5 March 2010 (UTC)[reply]
Well, the deal is that entropy is often a slightly misused term in the physical sciences (thermodynamics). Entropy is really just a mathematical concept, after all Ludwig Boltzmann was basically a mathematician. Entropy is just the relationship between the number of possible states of a closed system and the uncertainty of knowing which of those states you are in at any given moment. In thermodynamics, the term is used to basically mean the disorder in a system; it is expressed as negative free energy, or in other terms, entropy is expressed as "the energy needed to return a system to a state of perfect order, where all variables are eliminated and the location of all particles is known with perfect precision". Entropy as a concept is still the mathematical thing, but in thermodynamics we discuss it in terms of the free energy lost in reordering a system. In information theory, the entropy is the same idea; it is based on the number of possible states of a system and the uncertainty of knowing which state the system is in. Information theory actually uses a more pure form of entropy, in that it doesn't use a concept like energy as a surrogate for entropy; it uses a unitless value to express "pure" entropy. But the sign and directionality of the two are the same; the more disordered the system, the higher the value the entropy is. --Jayron32 01:26, 6 March 2010 (UTC)[reply]
Jayron, your final conclusion is essentially correct but your argument is stuffed with a potpourri of misconceptions. I won't go into all of them. Let me just point out that entropy and Free energy are not even measured with the same units so your explanation makes no sense. BTW your rant abount physical sciences misusing entropy is also a gem of nonsense. Dauto (talk) 03:20, 6 March 2010 (UTC)[reply]
Well, its not that they misuse the term entropy really. Just use it a bit differently than does information theory. And entropy is essentially negative free energy (for any given temperature). At a conceptual level, the two are basically opposite concepts. Entropy is merely disorder, the thermodynamic definition expresses this disorder as a unit of energy per temperature. Free energy is energy availible to do work; we can consider the theoretical "free energy of the universe" to have been at a maximum at the Big Bang, and it has been consitantly decreasing over time. Thus, the for the universe, spontaneousness is represented by a decreasing free energy, or ΔG < 0. The early physical chemists and mathematicians working in this area recognized that this was essentially the functional opposite of entropy; the entropy of the entire universe tends towards a maximum as the free energy tends towards a minimum. When we look at a chemical process, by Helmholtz's and Gibbs's conventions, we tend to take the perspective of "free energy". But removing the math for a second, and looking at the conceptual nature of it. Every process has two contributions to make towards affecting the entropy of the universe. Entropy is temperature dependant (warmer things have faster moving particles, so their position is less certain than cooler things), so any process that tends to heat its surroundings tends to increase the entropy at the universe level. However, most chemical processes also involve a change in organization of the particles, which is a direct effect on the entropy itself. So, if we want to look at a process, there are two things going on: Release or absorbtion of heat, which affects the entropy of the surroundings and reorganization of the substances involved, which affects the entropy of the system. Conceptually, entropy and free energy are basically the same thing in the opposite direction, and in thermodynamics, we tend to express everything in terms of energy, so by convention, the entire thing is expressed in terms of energy values, the classic G = H - TS (Or A = U - TS for the Helmholtzally inclined) which contains the expression of entropy as an energy value (J/K). But this is a convention to give us a number we can find meaning with (usually looking at the "spontanaity" or "extensiveness" of a chemical process) to compare different processes. --Jayron32 04:42, 6 March 2010 (UTC)[reply]
You spend a whole paragraph explaining how entropy and free energy are supposed to be the same concept and then write a (correct) equation that shows that they are related but are definitely different concepts and then conclude by saying that the equation is just a convention? Are you a Chemist? Dauto (talk) 15:45, 6 March 2010 (UTC)[reply]

Any known planets with satellites with satellites?

Are there any known cases in astronomy of a planet which has a moon orbiting it and that moon has another smaller moon orbiting it?20.137.18.50 (talk) 17:39, 5 March 2010 (UTC)[reply]

This question was answered a few years ago - the short answer is "No". The longer answer is that the nature of gravitational and tidal interactions between planet and moon would make a moon-of-a-moon orbitally unstable - so if one ever did somehow come into existence, it would either smack into the planet, the parent moon - or spin off into space within a relatively short period of time. The only exception to that are the various artificial satellites that humans put in orbit around various moons within the solar system - which were there only for short periods of time. SteveBaker (talk) 18:55, 5 March 2010 (UTC)[reply]
Thanks, Steve. When Lomm said in that original question "The key consideration (discussed at n-body problem) is that the smallest body must have a mass insignificant with regards to the largest body." with respect to the word "insignificant" are we talking 1/10, 1/100, 1/1000? I followed the link to the n-body problem and didn't see any elaboration of significance of mass difference there, though I have a little more idea of how crazily complex things get with moon-on-moon action. 20.137.18.50 (talk) 19:08, 5 March 2010 (UTC)[reply]
A close possibility is the tiny moon Nix and Hydra. They are orbiting a double body system, so in a sense, the satellite is orbiting a pair of satellites where Pluto is orbiting Charon while Charon is simultaneously orbiting Pluto. I suppose someone will come by soon and point out that none of these are technically planets. Googlemeister (talk) 19:16, 5 March 2010 (UTC)[reply]
Drat, I was just going to say that. I also wonder if Mercury and maybe Venus lack moons for the same reason, basically that the nearby Sun would knock the moons out of orbit. StuRat (talk) 21:17, 5 March 2010 (UTC)[reply]
The Lunar Reconnaissance Orbiter now orbiting the moon is a small artificial 'moon' of the Moon. You may be familiar with the planet. Cuddlyable3 (talk) 03:07, 6 March 2010 (UTC)[reply]
The LRO has the mass of a large automobile. Astronomical definitions are notoriously sketchy, but it's doubtful anything the size of a Lincoln Continental would ever be defined as a "moon," artificial or otherwise. 63.17.82.123 (talk) 04:25, 6 March 2010 (UTC)[reply]
@63.17.82.123 Enter "artificial moon" in the Wikipedia search box and you are taken to the article about artificial satellites. That is correct. The first satellite in fiction was called The Brick Moon. There is nothing that an artificial moon can be other than a manufactured satellite. Your objection to my use of the term "artificial moon" is groundless. Cuddlyable3 (talk) 18:04, 6 March 2010 (UTC)[reply]
Well clearly, the sun-earth-moon system has been stable for quite a while, so mass_moon/mass_sun = 3.69e-8 is "insignificant". —Preceding unsigned comment added by 83.134.176.244 (talk) 08:37, 6 March 2010 (UTC)[reply]
Doesn't distance play a role ? That might explain why Mercury (and maybe Venus) doesn't have any moons, while Pluto has at least 3. StuRat (talk) 13:22, 6 March 2010 (UTC)[reply]

Telescopes

Is there a functional limit on the level of magnification a telescope can achieve? So for example, how big would the lens have to be to read a car's license plate on Earth from Alpha Centauri?

Are there any technologies would could implement that would reduce that size or is it a hard limit? TheFutureAwaits (talk) 18:06, 5 March 2010 (UTC)[reply]

Generally speaking, the main purpose of a telescope is not to magnify to a great extent, but to collect as much light as possible, to study faint objects. That said, the angular resolution of a telescope depends primarily on its diameter. The larger the diameter of the telescope, the finer the detail it can resolve. On the surface of the earth, however, atmospheric seeing spoils the resolution of optical telescopes larger than ~10 cm, so you either need advanced technologies like adaptive optics or interferometry, or you need to put the telescope in orbit. -- Coneslayer (talk) 18:13, 5 March 2010 (UTC)[reply]
To address your question quantitatively, to read a license plate on Alpha Centauri, you need an angular resolution of roughly θ = (1 cm) / (1 pc) = 3e-19 radians. Using θ = 1.22 λ/D (as explained in angular resolution), and assuming λ = 500 nm (visible light), you need a space telescope with diameter D = 2e12 meters, or 2 billion kilometers, or 13 astronomical units. That's bigger than the orbit of Jupiter. -- Coneslayer (talk) 18:21, 5 March 2010 (UTC)[reply]
see http://en.wikipedia.org/wiki/Diffraction_limit —Preceding unsigned comment added by 83.134.176.244 (talk) 18:14, 5 March 2010 (UTC)[reply]
(ec) Even ignoring such things as atmospheric distortion and nearby bright things such as the Sun, one important question to consider (I don't know the answer) is how many photons you would need reflected from a license plate in order to be able to read it, and the rate at which these photons would reach Alpha Centauri. Probably you don't even have the remotest chance of being able to read a license plate, because at Alpha Centauri you'd receive one photon reflected from the license plate every 300 years (totally out-of-my-ass guess, but you get the idea). —Bkell (talk) 18:15, 5 March 2010 (UTC)[reply]
You could (in theory) solve the issue of the rare appearance of a photon reflected from that license place by integrating the incoming light over a very long period - but now we have a mirror the size of the orbit of Jupiter that's got to stay pointing in the right direction for (perhaps) tens of thousands of years - getting that kind of stability would be really tough - and in any case, that gigantic mirror would have to be moved to track the motion of the license plate relative to the telescope. If it has to point accurately to within 3x10-19 radians yet move by enough to track the motion of the license plate parent planet rotation - and that planets' orbit around the parent star - and the relative motion of that star with respect to the telescope (not negligable over the amount of time you'd need to be gathering light over)...then the mechanism that moves the mirror would be a horror to construct! This is so far from being a practical possiblity that we're going to have to say that the answer is "No" - you can't do that. SteveBaker (talk) 18:49, 5 March 2010 (UTC)[reply]
I wouldn't hold the mirror still, I would just measure its movements and have a computer compensate for them in the final image. --Tango (talk) 19:46, 5 March 2010 (UTC)[reply]

Interesting, so then how big of a diameter lens would be required to achieve sufficient angular resolution to read a license plate of a car on Earth assuming there was no atmosphere? TheFutureAwaits (talk) 18:17, 5 March 2010 (UTC)[reply]

Hmmm. I read license plates on earth without any lens all the time ;-). --Stephan Schulz (talk) 18:48, 5 March 2010 (UTC)[reply]
As Coneslayer already calculated...about the size of the orbit of Jupiter. SteveBaker (talk) 18:49, 5 March 2010 (UTC)[reply]
I was still drafting my calculation when TheFutureAwaits asked above. Given the "on Earth" part of this question, I'm not sure if it's meant to be a restatement of the Alpha Centauri question, or a different question. -- Coneslayer (talk) 19:01, 5 March 2010 (UTC)[reply]
The mirror does not need to be a disc 13 AU in diameter. You can have a series of smaller telescopes, all coordinated together. This already happens with radio telescopes and I think more rarely with optical telescopes. I cannot recall what this is called - I'm sure there must be an article on it. Edit: see Cambridge Optical Aperture Synthesis Telescope and http://www.mrao.cam.ac.uk/telescopes/coast/handout.html Very Long Baseline Interferometry Perhaps you could just have one telescope at Jupiter and put together what it sees with what it sees six "Jupiter months" later. 89.243.198.135 (talk) 19:13, 5 March 2010 (UTC)[reply]
That would be interferometry (or aperture synthesis), which I linked to in my original response. -- Coneslayer (talk) 19:14, 5 March 2010 (UTC)[reply]
In regards to Perhaps you could just have one telescope at Jupiter and put together what it sees with what it sees six "Jupiter months" later, no. Interferometric observations must be combined coherently. Without a direct way to measure optical phase, I do not believe there is any way to interferometrically combine optical observations taken at different times. -- Coneslayer (talk) 20:42, 5 March 2010 (UTC)[reply]
The Aperture synthesis article says that one telescope with the rotation of the earth is used. What stops a one-year rotation being used instead of a twenty-four hour rotation, at least for radio waves? 89.243.198.135 (talk) 20:53, 5 March 2010 (UTC)[reply]
Can you point me to where you're seeing "one telescope"? With a single baseline (two telescopes), you use the rotation of the earth to change the orientation of the baseline relative to the target, which lets you achieve high resolution along different axes at different times. But in any case, that's radio interferometry, not optical interferometry. It's possible to directly record phase of radio signals, which lets you do the interferometry in post-processing. You can't do that in the optical. -- Coneslayer (talk) 20:57, 5 March 2010 (UTC)[reply]

The alphacentaurians(?) could at best see only the license plate you had over 4 years ago. Cuddlyable3 (talk) 02:59, 6 March 2010 (UTC)[reply]

Even from earth orbit it is very hard to read license plates. I'm wondering why the alphacentaurians stick their license plates on the top of their cars instead of at the ends. Dmcq (talk) 08:44, 6 March 2010 (UTC)[reply]
Of course you don't look for license plates in the center of the Earth's disk, but at the very corner, when they are just driving over the horizon. You can even use head and tail lamps for targeting. Duh! --Stephan Schulz (talk) 09:10, 6 March 2010 (UTC)[reply]

OTC Cryogenics to treat warts or moles

Are OTC Cryogenics to treat warts or moles? Are they effective? What if a person would apply it on something else? For example, it is for moles and he applies it on a wart or the other way round? Quest09 (talk) 18:38, 5 March 2010 (UTC)[reply]

They are to treat warts (and are effective) and their instructions specifically state that they are not to be used on moles. I'll think on the rest of your question, but I think it's bordering on medical advice. -- Flyguy649 talk 18:56, 5 March 2010 (UTC)[reply]
(edit conflict)I have heard of warts being treated with liquid nitrogen which, amongst other things, stimulates the immune system into responding to the hpv virus that causes the wart. The effectiveness probably depends on the person and type of wart. I'm not sure about moles. — Ƶ§œš¹ [aɪm ˈfɹ̠ˤʷɛ̃ɾ̃ˡi] 18:59, 5 March 2010 (UTC)[reply]


I think in general it's a bad idea to think of warts and moles as having anything to do with each other. Warts are viral infections; they need to be gotten rid of, so the virus doesn't spread. Most moles, on the other hand, require no treatment at all; you can just leave them be unless they're bothering you in some way. There are of course exceptions — see malignant melanoma for the worst-case scenario. --Trovatore (talk) 19:01, 5 March 2010 (UTC)[reply]

120v v. 240v

How much power could you safely continuously draw through an ordinary domestic 120 volt American electricity socket versus an ordinary domestic British 240 volt (or possibly 220 volts) socket? British electric sockets are installed on a ring main which I think also allows more power to be drawn. Thanks 89.243.198.135 (talk) 20:30, 5 March 2010 (UTC)[reply]

I can't speak for UK circuits, but a North American 15 amp circuit is good for about 1800 watts, assuming no other loads. At that rate the circuit breaker may eventually trip if load is continuous. Acroterion (talk) 20:55, 5 March 2010 (UTC)[reply]
UK must be more than that because I had a 2000 watt electric fire plugged into the mains. —Preceding unsigned comment added by Dataport676 (talkcontribs) 21:25, 5 March 2010 (UTC)[reply]
The maximum current you can get from a UK socket is 13A - the nominal UK mains voltage is 230V ± 10% (see Mains power around the world). This means that 3kW (13A @ 230.8V) _might_ blow the fuse if the voltage is a bit high (or with a constant-power device and the voltage a bit low). 2kW and 2.5kW devices are common, though. Tevildo (talk) 21:26, 5 March 2010 (UTC)[reply]
You can also still get the old BS 546 unfused 15A round-pin plugs (giving you 3.6kW @ 240V), but that's not really an _ordinary_ domestic socket these days. Tevildo (talk) 21:35, 5 March 2010 (UTC)[reply]
In Britain the 13A limit mentioned by Tevildo is imposed by a fuse in the standard plug, not the socket. Modern UK houses are wired with a ring main that is capable of delivering 13A to multiple sockets. Therefore by bypassing or up-rating the 13A plug fuse one can draw more current from a British socket, limited by the main fuse. This is not a safe procedure for an amateur but it is possible. Cuddlyable3 (talk) 21:53, 5 March 2010 (UTC)[reply]
I would advise very, very strongly _indeed_ against this (unless one wants to burn one's house down). If you need more than 3kW from a socket, replace it with a proper BS 546 15A one. You _can_ get more than 13A out of an ordinary socket by shorting out the fuse. You _can_ get a higher steam pressure in a boiler by welding down the safety valve. The OP, however, included (quite rightly) the word "safely" in the question. This solution does not satisfy that criterion. Tevildo (talk) 22:15, 5 March 2010 (UTC)[reply]
As long as you only do it on one device, you should be fine, but if you do it too much you'll trip the circuit. The mains breaker would normally take more effort to trip. --Tango (talk) 22:05, 5 March 2010 (UTC)[reply]
Concern for safety is admirable if it is based on facts. BS546 is an outdated British standard for round-pin plugs and sockets that are incompatible with modern 13A BS1363 sockets and lack safety features of BS1363: shuttered socket, finger shrouds, fuse in plug and flat-wiping contacts. If recommending someone install new sockets for high current supply the Europe wide IEC 60309 standard family is preferable because of its modern design. Connectors are colour coded yellow 100-130V, blue 200-250V. Versions for 16 A, 32 A, 63 A and higher are available. Cuddlyable3 (talk) 02:52, 6 March 2010 (UTC)[reply]
Similarly in North America, while the standard 120 V socket is intended for circuits fused at 15 A or 20 A, there are higher-current sockets available (yes, for 120 V as well as for 240 V); but in any country such higher-current sockets are meant only for use on a special circuit with heavier wiring than usual. --Anonymous, 05:23 UTC, March 6, 2010.
In UK,ring mains are normally fused at 30 Amps.--79.68.242.68 (talk) 00:17, 6 March 2010 (UTC)[reply]
This means that, in the UK, 3kW appliances are common. (The 13a fuse allows up to 3.12 kW at the standard UK voltage of 240v (yes, I know that 220 is quoted, but 240 is standard)). For anything larger than this, it would (in theory) be possible to use multiple plugs and cables, but this would be very dangerous for obvious reasons (live plugs!), so any appliance that draws more than 3 kW must be permanently wired with a high current switch. Instant showers up to 9.8kW are available, fused at 40 amps. Drawing more than 3kW by shorting out the fuse will result in a dangerously hot plug and socket because the connectors are not designed to carry more than 13a. Dbfirs 11:30, 6 March 2010 (UTC)[reply]
Are you sure that drawing more than 13A via a 13 Plug /socket arrangement will result in a dangerously hot plug and socket? Since P = I^2*R, what resistance would the plug/socket have to have to make it 'dangerously hot'? And how does this square with the commonly accepted standard safe current density in conductors of 1000A/cm^2? —Preceding unsigned comment added by 79.76.171.183 (talk) 21:49, 6 March 2010 (UTC)[reply]
A compliant plug and socket has a safety factor that should allows for 3 kVA plus a 'bit more' . This is because the 'contact points' between the cable and the screw terminals are a fraction of their cross sectional areas (cm^2). Yes, plugs and sockets can get warm. Properly wired and carrying no more that the rated kVA they should not generate undue heat though. So, as stated above: do not over load you circuit. I have seen charred plugs (more than once) where people have wired up two appliances to one plug and drawn toooo much current. --Aspro (talk) 22:23, 6 March 2010 (UTC)[reply]


I've not seen "ordinary 120 volt domestic outlets" rated above 20 amps, but U.S. electric codes usually want you to stay below 80% of that rating, which would be 16 amps. That said, if it is rated at 20 amps it could probably supply 20 amps. RV hookups have 30 amp 120 volt outlets. 50 amp 125 volt connectors are available.For more than 20 amps it is common to go to 240 volt supply. There is really no physical limit on how many amps a 120 volt connector could be built to supply for industrial or utility applications. Some low voltage network grids in large cities are built to carry thousands of amps at 120 volts, in a 120/208 three phase circuit, and these have connectors. Edison (talk) 21:24, 6 March 2010 (UTC)[reply]

Genus name

From where did the Genus Zyzzyx derive it's name? Googlemeister (talk) 20:34, 5 March 2010 (UTC)[reply]

First paragraph of that article you linked to, emphasis added:
Zyzzyx is a monospecific genus of sand wasp, containing a brightly-colored, medium-sized species, Z. chilensis, named after the sound they make while flying. They were first studied in detail by H. Janvier (a.k.a. Claude-Joseph) in 1928, more than 100 years after they were first described. The unusual name is onomatopoeia for the buzzing sound they make.
Hope that answers your question. —Bkell (talk) 20:36, 5 March 2010 (UTC)[reply]
I read that but suspected it was maybe vandalism. Googlemeister (talk) 20:42, 5 March 2010 (UTC)[reply]
Ah, okay, that's possible. —Bkell (talk) 20:48, 5 March 2010 (UTC)[reply]
This source says Alan Solem named Zyzzyxdonta and Aaadonta "with the idea of being the first and the last entries in any list of endodontoid snails." Not sure if the sand wasp is related. Gobonobo T C 21:28, 5 March 2010 (UTC)[reply]
I've fact-tagged the claims that it's supposed to represent the sound made while flying. There's no citation. Of interest of course is that there is a "Zyzzyx Road" in California, found on Interstate 15 between Los Angeles and Las Vegas. Two films are named after the road, according to our articles; the film Zyzzyx Road is said to have earned a US box office total of US$30, and an international take of over 10,000 times that amount. Comet Tuttle (talk) 23:58, 5 March 2010 (UTC)[reply]
The real road is actually Zzyzx Road. —Bkell (talk) 03:57, 6 March 2010 (UTC)[reply]

A little confusion concerning E=mc^2

I've been told that the E in E=mc^2 stands for energy in general. But, the equation itself is derived from solving the work integral (with m(v) = γ*m_o), and so I would assume that it is only applicable for kinetic energy. When an object is heated, I can see why its mass would increase: the increased temperature means the atoms are vibrating faster, implying their individual masses are higher (by m(v) = γ*m_o), and thus the object has a higher mass. But for something like binding energy, I'm at a loss as to how the equation E=mc^2 can predict that CO2 would have a lower mass than a carbon atom and two oxygen atoms at the same temperature, or that an object at a higher altitude would have more mass. —Preceding unsigned comment added by 173.179.59.66 (talk) 21:02, 5 March 2010 (UTC)[reply]

You might like to ask the maths desk to help with this one
I don't see why. It is clearly a science question. --Tango (talk) 21:27, 5 March 2010 (UTC)[reply]
Yes, but it's also maths. I just felt the OP could benefit from a wider insight into his question, for example making a thread at the maths desk directing them here. —Preceding unsigned comment added by Dataport676 (talkcontribs) 21:29, 5 March 2010 (UTC)[reply]
It's only maths to the extent that all quantitative science uses maths. We don't notify the maths desk every time a science question is asked on the science desk. --Tango (talk) 21:54, 5 March 2010 (UTC)[reply]
E=mc^2 doesn't predict anything about energy, it just tells you the relationship between energy and mass. CO2 will have a lower energy than the atoms separately and E=mc^2 tells us that that means it will have a lower mass. To realise that it has a lower mass you need some results from physical chemistry, not relativity. --Tango (talk) 21:27, 5 March 2010 (UTC)[reply]
Right, but the derivation of E=mc^2 seems to be only applicable to kinetic energy. My question is how it can encompass potential energy too. 173.179.59.66 (talk) 21:45, 5 March 2010 (UTC)[reply]
There are more complicated derivations which are more general, I think. --Tango (talk) 21:54, 5 March 2010 (UTC)[reply]
E=mc^2 is applicable to all energy and mass, not just "kinetic". So potential energy stored between the bonds of a compound makes that compound slightly heavier than the atoms would be seperately. A hydrogen atom is a slightly lower mass than the sum of the mass of a proton and an electron because of the energy released because of the force of attraction between a positive and negative charge, etc. etc. Concepts like "kinetic energy" and "potential energy" aren't relevent to mass-energy equivalence. It's all just energy. So objects moving faster are heavier because of the higher kinetic energy, AND stored-up energy also generates more mass. --Jayron32 01:14, 6 March 2010 (UTC)[reply]
As you said, the derivation usually given in introductory books seems to be applicable only to kinetic energy but in fact is is applicable to all forms of energy. Please do the following gedanken experiment: Put some carbon and oxygen inside a box and burn the carbon. During the burning potential energy gets converted into kinetic energy leading to a higher final temperature. Now assume that an observer was passing by with a speed v. From the point of view of that observer the box was moving at constant speed. Since momentum is also conserved, the mass of the box must have remained constant throughout the process. That means that the mass-(kinetic energy) equivalence at the end must have come froma mass-(potential energy) equivalence at the start. Dauto (talk) 01:16, 6 March 2010 (UTC)[reply]
Interesting, I never thought of that! Out of curiosity, is this the official derivation, or is there a more "mathematical" method of coming to this conclusion (like through 4-vectors or something of the sort)? —Preceding unsigned comment added by 173.179.59.66 (talk) 07:06, 6 March 2010 (UTC)[reply]
If you want a derivation of e=mc^2 then this comes from special relativity. It comes from the fact that in natural units from a reference frame of time t E/m = dt/d(tau) where tau is proper time. Viewing in a frame t where the particle is at rest, t and tau are the same leading to E=m or E=mc^2 in conventional units. See Mass–energy_equivalence#Background 82.132.136.207 (talk) 01:00, 7 March 2010 (UTC)[reply]

Protein architecture, motif, domain, fold, topology.

I'm taking a 4th level biochemistry class titled Protein structure and function and, though I already learned them intro biochem courses, the way the materials are covered is much more detailed. I don't really understand the following: architecture, motif, domain, fold, topology (used in a different sense instead of the more common definition of how the transmembrane protein is embedded). Many of these terms are used interchangeable and I don't have a clear idea of the true definition of each. For example, I saw Alpha/Beta/Alpha domain being also referred to as Alpha/Beta/Alpha architecture. Also, it seems that Greek key motif is also called a Greek key domain, which can also be said to be an architecture. Also, fold and motif seem to be complete synonyms. I'm really confused. Please clarify with clear examples. (PLEASE DON'T JUST REDIRECT ME TO LINKS HAVING GENERAL DEFINITIONS OF EACH, BECAUSE I ALREADY CHECKED MANY OF THEM OUT) —Preceding unsigned comment added by 142.58.129.94 (talk) 21:43, 5 March 2010 (UTC)[reply]

Some of these terms are broader or narrower. Architecture is the most broad covering the construction from small to big. The architecture can be made up of those other components. Motif is a pattern that you can see over and over again in different places. domain is a piece of the protein that has a stable shape in itself, so if you chop it off te part will retain this shape. Domain is narrower term or architecture, and may include folds and motifs as components. Fold is pretty clear that it is a bend or change in direction, I would count it as a narrower term of motif, but some think otherwise. The term topology seems to be abused in biochemistry, referring to where parts of the protein chain is, whether embedded in the membrane or poking out. Toploogy in maths will refer more to how it is connected, loops, or multiple elements. Is that 4th year at a uni? I have never studied biochemistry in a class. Graeme Bartlett (talk) 23:31, 5 March 2010 (UTC)[reply]
First off, I'm a cell biologist, not a structural biochemist. However, I've seen and used these terms thus:
  1. Architecture - the general design of a protein, consisting of perhaps one or more domains or motifs. The architecture of the Src protein consists of an SH2 domain and an SH3 domain
  2. motif - a short, often linear, amino acid sequence that can be a target recognition sequence, e.g. LLDLD clathrin interaction motif or PxxP, SH3 binding motif where x = unimportant. These can have different possible amino acids at certain positions, e.g [D/E][G/A](0-1)F[G/A][D/E]F binds gamma ear domains of AP-1.
  3. domain - usually a larger part of a protein with a specific function and usually a certain fold/folds (i.e. its own 3D structure). e.g. SH3 domains have a consensus sequence and bind certain protein sequences containing a PxxP motif. ENTH domains are large (145 aa) domains at the N-terminus that bind membranes containing specific PIPs. BUT some people use "domain" for motif
  4. fold - usually a change in direction in the secondary structure of a protein, but it can mean a fold in the 3D shape of a protein, context dependent
  5. topology - the specific 3D shape of a protein either overall or locally, e.g. folds and pockets in a globular region.
Perhaps a structural biologist/protein biochemist can come around and weigh in. -- Flyguy649 talk 00:53, 6 March 2010 (UTC)[reply]
I dabble in structural biology, though it's not the central focus of my work. I'd like to emphasize that 'fold' in the context of structural biology usually doesn't mean the same thing as it would in a nontechnical context — it's not just a bend or a change in direction. A 'fold' in structural biology generally refers to a particular tertiary structure (at least, an arrangement of secondary structures) which may or may not have a common underlying primary amino acid sequence. For example, the ubiquitin-like proteins (including ubiquitin, NEDD8, and SUMO, among others) all share a 'beta-grasp' fold, in which a beta sheet snuggles up adjacent to (and partially around) an alpha helix. Despite having a very similar tertiary structure, the ubiquitin-like proteins have a relatively limited similarity in sequence. A 'fold' may describe the structure of an entire protein, or a common structure shared by parts of several proteins. (The beta-grasp fold, for instance, shows up in a lot of places.) TenOfAllTrades(talk) 03:34, 6 March 2010 (UTC)[reply]

Positive and negative features of CATH and SCOP

They both offer similar services, but how are they different? Can you say which is better? I checked out the tutorial manuals for CATH and SCOP but they aren't very user-friendly. —Preceding unsigned comment added by 142.58.129.94 (talk) 22:22, 5 March 2010 (UTC)[reply]

You want the article Systematic Comparison of SCOP and CATH: A new Gold Standard for Protein Structure Analysis, full text here. Cuddlyable3 (talk) 02:15, 6 March 2010 (UTC)[reply]

Atoms

Without using a microscope, would it actually be possible to see an atom using just normal magnifying glasses, perhaps using lots of them lined up to increase the zoom? —Preceding unsigned comment added by Firemansam490 (talkcontribs) 22:52, 5 March 2010 (UTC)[reply]

No. The most powerful microscopes are scanning electron microscopes. By comparison, optical microscopes have much poorer resolution. Dolphin51 (talk) 23:17, 5 March 2010 (UTC)[reply]

Uh, no. We can't even really "see" atoms with a microscope - we need a special kind of microscope (scanning electron microscope) that interprets the image for us, rather than just using lenses to make things look bigger than they are. By the time you lined up enough lenses to hope to see atoms through them, what you saw would just be a big blur anyway - you'd be magnifying the imperfections of the lenses over and over again and the image would be distorted beyond recognition far before you got to the atomic level. - AJ —Preceding unsigned comment added by 71.108.171.138 (talk) 23:22, 5 March 2010 (UTC)[reply]

If you line up good magnifying glasses in the right way you will make a microscope, but dont expect much magnification or clear view. Your best chance will be to see gazillions of atoms together making up a visible object. The wavelength of light is about 5000 times bigger than ab atom, so it is like trying to feel a grain of sand with a truck. Graeme Bartlett (talk) 23:37, 5 March 2010 (UTC)[reply]
Although it's not optical microscopy, atomic force microscopy lets people image individual molecules and even atoms. Brammers (talk) 00:34, 6 March 2010 (UTC)[reply]
Another part of this problem is that it's not at all clear what we mean by a "picture" of an atom. These scanning electron and atomic force microscopes are measuring the field surrounding the atom and turning that into a picture - but that doesn't mean that this is what an atom "looks like" any more than a congenitally blind person can imagine what someone's face "looks like" by feeling it with their hands. With quantum uncertainty and wave-particle duality effects, it's really meaningless to ask what an atom looks like anyway - even in principle. SteveBaker (talk) 03:22, 6 March 2010 (UTC)[reply]

The problem with visible light is that its wavelength is on the order of a few hundred nanometers - more than a thousand times as large as an atom. With ordinary techniques you cannot see details smaller than approximately half the wavelength. Icek (talk) 18:16, 6 March 2010 (UTC)[reply]

Anthropophobia?

I'm wondering if the following would fall under the diagnosis of anthropophobia or some other term. . . . It's a fear of people, but's it's not "phobic" in the sense of being intense or causing panic symptoms, except occasionally in crowds (at the mall or in line at the grocery store, for example). It's more of an acute discomfort in the presence of human beings in general. It's not sociopathy, because conscience and sympathy aren't lacking. It's not an inability to love or care for others as individuals - quite the contrary. It's just that the presence of even someone very well loved causes a kind of elemental discomfort, and as much as the dear one may be missed in his or her absence, there is a relief and a comfort in solitude that has NEVER been experienced in the presence of any human being under any circumstances. This is so to the point that anticipation of seeing the loved one, even after a long absence or in circumstances such as the loved one's coming home from the hospital which are otherwise incontestably a good thing, prompts mixed feelings. A person in this condition would thrive when left in solitude, but struggle to explain why simply living in close proximity to others induced a sort of emotional paralysis that made it hard to be helpful and productive, since it's hard to be taken seriously when the message they're conveying is "It's not you, really, it's just that you're human," especially when everyone's trying to convince them that what they really need is to get out and socialize more.

Is there a name for this?

It's called Social anxiety disorder.

Wouldn't social anxiety disorder be more about inability to deal with what are conventionally thought of as "social" situations (school, the workplace, gatherings of friends), rather than desiring to hide out from your own mother or spouse?

From our Schizoid personality disorder article: Schizoid personality disorder (SPD) is a personality disorder characterized by a lack of interest in social relationships, a tendency towards a solitary lifestyle, secretiveness, and emotional coldness. Is that close? Googling "schizoid" even yields message boards populated by people who claim they have this order, and discuss their behavior and feelings. (By the way, please type ~~~~ after each post, to sign it, so we can keep track of who is saying what.) Comet Tuttle (talk) 23:53, 5 March 2010 (UTC)[reply]

In the movie Men in Black the morgue doctor Laurel Weaver (Linda Fiorentino) has this line "I hate the living." Cuddlyable3 (talk) 01:48, 6 March 2010 (UTC)[reply]

Almost all of these 'phobia' words are made up and used indiscriminantly (check out reference 2 in List of phobias for example!). Basically, almost all phobias are basically anxiety disorders - but with different triggers for that anxiety that are probably as varied as the people who suffer from the disorder. SteveBaker (talk) 03:15, 6 March 2010 (UTC)[reply]

Changed the earth's axis??

This week's TIME magazine reports that earth's day has been shortened by 1.26 microseconds as one result of Chile's earthquake, and the reason is because the earth's axis got shifted about 3 inches.

Could someone expand on this, provide a little more detail on how one caused the other? The only explanation I can imagine -- that the diameter of the earth is now slightly smaller due to subduction -- doesn't make ANY sense at all! DaHorsesMouth (talk) 23:16, 5 March 2010 (UTC)[reply]

You are basically correct, the Earth's moment of inertia was reduced by about 15 parts per trillion as dense rock subducted deeper into the earth and displaced less dense rock (and/or changed the shape of the sea floor). Along 700 km of the fault, the Earth jumped up to 10 m, which is enough to have a tiny but perceptible effect of the Earth as a whole. Dragons flight (talk) 01:38, 6 March 2010 (UTC)[reply]
And, when you move the mass inward on a spinning body, it spins faster, resulting in a shorter day, due to conservation of (angular) momentum. StuRat (talk) 02:51, 6 March 2010 (UTC)[reply]
But how would that shift the earth's axis? 95.112.175.41 (talk) 09:02, 6 March 2010 (UTC)[reply]
The stuff about the Earth's "axis" is a bit of a red herring, and not surprisingly most of the newspapers have got the story confused. The BBC's version is particularly bad, showing a completely irrelevant diagram of the precession of the axis. If Richard Gross of NASA JPL is to believed, this is what happened: the Earth's mass distribution shifted, as Dragons flight correctly said. This shift had two effects. 1: the Earth's "figure axis" (the one that passes through its centre of mass) shifted by a few metres; 2: the rotation sped up due to the reduction in the moment of inertia. Effect 1 is not related to the length of the day, although I guess it might cause a few places on Earth to shift time zones by a few nanoseconds. Effect 2 is what (is predicted to have) shortened the day by a microsecond. --Heron (talk) 10:28, 6 March 2010 (UTC)[reply]
Sorry, can't find anything about shifted axis in Richard Gross article. Wouldn't that be even violating momentum conservation? 95.112.175.41 (talk) 11:51, 6 March 2010 (UTC)[reply]
I can't remember exactly where I got that claim from, but there are plenty of other examples on the web. For example, this similar article by National Geographic attributes the figure axis shift to Gross. --Heron (talk) 15:36, 6 March 2010 (UTC)[reply]
Now, well, that's better. Not the rotation axis has shifted but the figure axis. 95.112.175.41 (talk) 19:33, 6 March 2010 (UTC)[reply]

voltage

what is the maximum voltage that can be artificially generated within the limitations of current technology? —Preceding unsigned comment added by 129.67.118.243 (talk) 23:59, 5 March 2010 (UTC)[reply]

I aksed my science teacher this once, they said that voltage is infinite it's current that matters. You could have a balizzion volts with no current and it wouldn't kill you, or you could have 1 volt with a balizzion currents and it'd fry you like a deep fried mars bar. —Preceding unsigned comment added by Zonic4 (talkcontribs) 00:03, 6 March 2010 (UTC)[reply]
Some modern Van de Graaff generators can create a potential difference of up to 5 megavolts (that's 5 000 000 volts), Our high voltage article says that lightning can have a potential difference of up to 1 gigavolt (1 000 000 000 volts) (though I don't see a reference, and that isn't artificial). Buddy431 (talk) 01:03, 6 March 2010 (UTC)[reply]
According to our Van de Graaff generator article, the record is 25.5 million volts at Oak Ridge, although that particular statistic is uncited. The limiting factor is the breakdown voltage of the insulation between the collector dome and the rest of the structure - the Oak Ridge machine (and others like it) use sulphur hexafluoride. Tevildo (talk) 01:14, 6 March 2010 (UTC)[reply]
(edit conflict)Voltage merely means that you have two points in space with electrons at different levels of potential energy. When the potential energy difference becomes great enough, the electrons are "forced" to move from the higher energy state to the lower one (i.e. generate a current). Once the current starts to flow, the voltage will lower slightly, until the current reaches a steady state, and the system reaches equilibrium according to Ohm's Law. However, back to the main question, there is no theoretical upper limit to voltage, this article at google news: [1] shows that 5,000,000 volts was attained in 1923. We can only assume the number to be orders of magnitude higher today. I can't find anything else using google, but if you play around with search terms, you may be able to find something. --Jayron32 01:09, 6 March 2010 (UTC)[reply]
I would say tens of megavolts for a short period: but thats just my gut feeling. No evidence Im afraid.--79.68.242.68 (talk) 01:12, 6 March 2010 (UTC)[reply]
Plasma acceleration has shown a transient electron acceleration equivalent to 40 billion volts, but the process is not really the same as the way we usually think about voltage, and field only exists for about a nanosecond. Dragons flight (talk) 01:25, 6 March 2010 (UTC)[reply]
An Electron microscope built[2] at Osaka University uses a 3 000 000V supply from a Cockcroft–Walton generator. Cuddlyable3 (talk) 02:01, 6 March 2010 (UTC)[reply]
I was hoping to find answers in Orders of magnitude (voltage) - but the article didn't exist, so I had to write it - which means that there is nothing there that hasn't already been said. SteveBaker (talk) 03:08, 6 March 2010 (UTC)[reply]

There is some confusion above between electric fields and voltage, which is the antiderivative of the electric field. There are limits on the strength of an electric field because any material will undergo dielectric breakdown at some point, but it is difficult to set a limit on voltage. If there is one, it would arise from limited ability to concentrate charge within a finite region -- but the fact that black holes can be charged means that means that they can at least accumulate charge up to the point where gravitational attraction is outweighed by electrostatic repulsion. Looie496 (talk) 17:57, 6 March 2010 (UTC)[reply]


March 6

Genetics and cousins

If a man marries a woman then his brother marries the woman's sister and both couples have children, how many genes do the children share with their cousins compared to regular cousins? --124.254.77.148 (talk) 02:21, 6 March 2010 (UTC)[reply]

They would be double cousins. That article says "Genetically, they are as related as half-siblings, sharing 25% of their DNA (a coefficient of relationship of 1/4)." --Tango (talk) 02:26, 6 March 2010 (UTC)[reply]
But, of course, that's just an average. They could actually share anywhere from 0% of their chromosomes (unless they are both male, in which case the cousins must share a Y chromosome) to 100%, which would also require that their two sets of parents were 100% genetically identical. Of course, the extremes are extremely unlikely, but exactly 25% is unlikely, too. (Or should I say impossible, since 46 chromosomes aren't divisible by 4.) StuRat (talk) 02:45, 6 March 2010 (UTC)[reply]
It is more complicated than that since chromosomes don't stay intact - genes can move around when the chromosomes are splitting. See Chromosomal crossover. --Tango (talk) 02:55, 6 March 2010 (UTC)[reply]
Also their parents wouldn't have to be genetically identical. Half of the parents' chromosomes aren't passed on to the children and that discarded half wouldn't have to match. Rckrone (talk) 19:24, 7 March 2010 (UTC)[reply]
Yes, you're correct, they would need to at least 50% genetically identical. StuRat (talk) 00:20, 8 March 2010 (UTC)[reply]

Please note that there is absolutely nothing tabboo about this arrangement (two siblings marrying, respectively, two other siblings), it is not incestuous or dangerous or anything like that. 82.113.121.104 (talk) 10:07, 6 March 2010 (UTC)[reply]

Except, apparently, in West Virginia and North Carolina. --80.177.170.180 (talk) 11:15, 6 March 2010 (UTC)[reply]
[citation needed] --Tango (talk) 18:34, 6 March 2010 (UTC)[reply]
80 is just slightly confused I believe. Our article linked above mentions double first cousin marriages are forbidden in West Virginia and North Carolina which otherwise allows first cousin marriages. This isn't what's being discussed here however (the OP asked what is the relationship of children not whether the children of the relationships are allowed to marry and 82 didn't discuss that either). 82 is probably right that in most or all countries there's nothing illegal about marriages that result in double first cousins (i.e. siblings marrying siblings) even if the children/double first cousins can't marry in a few places. Even identical twins marrying identical twins likely isn't illegal in most or all places although given that their children would be genetically basically siblings, probably can't marry in many places. Of course, you also get the case where after divorce or death, a person may marry the former spouse of their sibling, in some cases it was expected upon death at least for a widow (i.e. Levirate marriage) Nil Einne (talk) 19:42, 6 March 2010 (UTC)[reply]

:Each of my grandfathers had some double cousins, and I've turned out orl write normal. --80.177.170.180 (talk) 11:14, 6 March 2010 (UTC)[reply]

red shift versus blue shift

Redshift and blue shift

In considering red shift and blue shift would not it make more accurate or reliable not to think in terms of eyes or ears as being stationary and the object moving as denoted by oneway pointers but rather as both observer and object being subject to motion using dual pointers such as <<<<<<>>>>>> and >>>>>>><<<<<<<? 71.100.11.118 (talk) 03:03, 6 March 2010 (UTC)[reply]

If you are talking about redshift of light in vacuum than it makes no difference. Dauto (talk) 03:12, 6 March 2010 (UTC)[reply]
I'm including a moving siren in air that passes me while riding down the highway. 71.100.11.118 (talk) 03:16, 6 March 2010 (UTC)[reply]
Than yes, you are right. It makes some difference whether the source, the observer, or both are moving.Dauto (talk) 04:05, 6 March 2010 (UTC)[reply]
For sound...yes...because the medium that it's passing through is "stationary" so the symmetry between moving observer and moving source is broken. But the redshift in light doesn't do that because in a vacuum, there is no stationary 'reference'. Light is weird. SteveBaker (talk) 06:53, 6 March 2010 (UTC)[reply]
Obligatory reading: the article Doppler effect. Cuddlyable3 (talk) 00:29, 7 March 2010 (UTC)[reply]

red shift and blue shift for objects in the Universe

Is the average amount of red shift the same in opposite directions when observing from Earth or slightly less or more in any particular direction? 71.100.11.118 (talk) 03:14, 6 March 2010 (UTC)[reply]

I'm not really sure what you mean by the average amount of redshift - what are you averaging over? The Cosmic Microwave Background radiation is redshifted more in one direction than the other (see Cosmic Microwave Background#CMBR dipole anisotropy), if that helps. --Tango (talk) 04:42, 6 March 2010 (UTC)[reply]
For instance perhaps in a binary star system where the red shift will vary in degree and possible to the extent of going int the blue (assuming the system were close enough, the orbital speed high enough, etc.) 71.100.11.118 (talk) 05:24, 6 March 2010 (UTC)[reply]
It would depend what you take the average over. For example you could quantify the amount of redshift in terms of the relative velocity of the object, and then the average value for an orbiting object would be the same as the value for an inertial object with the same average velocity by definition. If you're averaging over the Doppler factor then in general that wouldn't work since the Doppler factor isn't linear in the relative velocity. Rckrone (talk) 19:11, 7 March 2010 (UTC)[reply]

Freeze data outside U.S.

Is it possible to find information on the dates for the first freeze in the fall and the last freeze in the spring for places outside the U.S.A.? All my effort so far has yielded no fruit - all the information I can find (online at least) seems to be limited to the U.S.A. which seems very strange - don't gardeners in, say, Romania or Korea need to know just as well?

69.140.13.88 (talk) 03:45, 6 March 2010 (UTC)Nightvid[reply]

Even the data available for the USA might not be reliable during the oncoming of global warming. For instance I've lived in Florida since 1951 and I've never experienced more than two weeks of temperatures below 40 Deg F. This year we still have temperature down to freezing in early March. 71.100.11.118 (talk) 04:03, 6 March 2010 (UTC)[reply]
Here's a very specific example for a small city in Germany [3]. Aaadddaaammm (talk) 11:09, 6 March 2010 (UTC)[reply]
Northern England (UK): earliest autumn freeze in the last fifty years was September 1st when a whole row of my still-flowering hydrangeas turned a surreal black! Night-frosts occur well into June (summer here), but I don't have exact dates. Parts of Scotland will fare worse. This site [4] might be of interest. Dbfirs 12:15, 6 March 2010 (UTC)[reply]

Static electricity damage

I recently discharged static electricity accidentally on a stereo. It's not a very high tech one but ever since i was unable to power it on. When plugged in, the 12:00 keeps flashing, that's about it. Any way an electronic device like this can be repaired? What component is likely damaged? Maybe there is something I can replace inside? Thx.

Our article has only one line about this topic. --Kvasir (talk) 05:02, 6 March 2010 (UTC)[reply]

You'd have to take it to an electronic repair shop. Once static electricity "sparks" or discharges, it becomes an electric current (and thus no longer static). Sending a pulse of unregulated electrical current through a device with hundreds of tiny electronic devices and hundreds of more little connections and wires means the current could have literally "fried" anything. There are hundreds of options as to what you fried, and there is no way, without carefully analyzing the device, to identify which little connection or tidbit or doofulator you fried. It would only take one. So, you'll need to find someone that speciallizes in repairing these devices, though theres a better than 50/50 chance that the stereo is unfixable even by an expert. --Jayron32 05:07, 6 March 2010 (UTC)[reply]
Beyond the 50/50 chance of being unrepairable, there is a much higher chance that a brand new one will be cheaper than the cost of repairing the old one. Electronics keep getting better and cheaper. Many electronic devices are now considered nearly disposable, like cell phones and mp3 players. -- kainaw 05:12, 6 March 2010 (UTC)[reply]
Static damage is very insidious. It can cause chips to fail intermittently - or not fail at all for weeks after the event and then suddenly die. You could easily pay a stack of money to repair it - only to find that it fails again soon after. Also, diagnosing these faults is pretty tough so unless they do a "repair-by-replacement", swapping out all of the electronics in the unit - the cost of figuring out which chips died would be very hard. Electrostatic discharge may be of some interest. The most likely components to fail are the chips - knobs, buttons, wires, connectors, circuit board tracks, discrete transistors, resistors, capacitors, diodes and coils are unlikely to fail - and the fact that the display is flashing says it's not the display or the power supply. So probably there are a couple of chips in there - and one or more of them is 'fried'. Replacing the chips one at a time would be the most likely approach to succeed - but honestly, I agree with the others...much as it pains me to say so: forget it - buy a new one - contribute more nasty electronics waste to your local landfill. :-( SteveBaker (talk) 06:43, 6 March 2010 (UTC)[reply]

Most electrical devices have gold film to conduct the static to the sites of the device and way from the chips. This sometimes results in a slight shock when touching them, but protects the internal operations. —Preceding unsigned comment added by VCRVLC1010 (talkcontribs) 17:22, 6 March 2010 (UTC)[reply]

Old school trouble shooting: I might inject a signal in the audio output and if sound comes out the speaker, work back stage by stage until the bad one is found. No one should tinker with AC powered equipment unless he is trained in electrical safety. The spark might have fried the portion of the circuit which was touched (if the antenna, then the RF stage; if an RCA audio connector, then the audio stage. With soldered in ICs I wouldn't bother (because I am not good at unsoldering them). Once upon a time it was straightforward to replace a transistor. Edison (talk) 21:11, 6 March 2010 (UTC)[reply]
Thanks guys. I think it would be a fun project to diagnose the problem, knowing that it's *possible* to fix. I mean wouldn't hurt to play with it when you know you are prepare to replace it anyway, right? --Kvasir (talk) 08:57, 7 March 2010 (UTC)[reply]

Relativity

To my understanding, Einstein said that, in order for the laws of physics to remain consistent with the principle of relativity and the universality of the speed of light, a moving object must undergo length contraction, time dialation, and mass increase. If we were to view an object as consisting of a collection of charges, would it be possible to also derive the same results via electromagnetic theory? I saw the article on electromagnetic mass, which said that the velocity dependancy of the electron's mass was predicted before relativity using E&M, so I was wondering if the same was possible with time dialation and such, or whether they are considered to be beyond the perview of E&M theory. —Preceding unsigned comment added by 173.179.59.66 (talk) 07:15, 6 March 2010 (UTC)[reply]

None of the effects that you claim (length contraction, time dialation, and mass increase) actually occur in the reference frame of the moving object. An observer on the moving object would find everything normal, but would observe that the rest of the universe appeared distorted. (Sorry I don't know enough about E&M to answer your question, but Steve or some other expert will give a better answer.) Dbfirs 12:28, 6 March 2010 (UTC)[reply]
Let me try. You could solve the Maxwell equations for an electromagnetically bound system, say a low-mass negative charge orbiting a stationary high-mass positive charge (note: such orbits are not stable but that isn't important here). Then, you solve the Maxwell equations again for the same system, only now it's moving at a constant speed in a constant direction. Obviously, you don't get to use the Lorentz transformation to simplify your problem (that would be assuming your answer), no, you must solve it in the original frame of reference by using, among other things, a current rather than a stationary central charge. Since you solved it in the original frame of reference, you will find that your solution is length-contracted relative to your original non-moving solution. —Preceding unsigned comment added by 83.134.176.244 (talk) 12:52, 6 March 2010 (UTC)[reply]
Yes, and even Einstein himself derives "time dilation" from an argument based on a "light clock" (i.e. light bouncing back and forth between mirrors) which can be seen as a system comprised of an EM field obeying Maxwell's Equations with reflecting boundary conditions. If you want, those boundary conditions can be derived from microscopic physics based upon the Lorentz force of the light's EM fields on the charges in the conductor/mirror, which in turn reradiate. All of this could have been done with pre-1905 electromagnetic theory which is nonetheless fully relativistic! 69.140.13.88 (talk) 16:09, 6 March 2010 (UTC)Nightvid[reply]
Cool, thanks! —Preceding unsigned comment added by 173.179.59.66 (talk) 16:45, 6 March 2010 (UTC)[reply]
It's also relevant to note that Einstein's 1905 paper, in which he set out the principles of special relativity, was entitled "On the Electrodynamics of Moving Bodies" (emphasis added). See History of special relativity for more detail on the origins of the theory. Tevildo (talk) 21:23, 6 March 2010 (UTC)[reply]

Circular DNA

In published sequences of circular DNA molecules (I'm thinking chloroplast DNA specifically), how do they determine the numbering system? Ie. who/what decides which nucleotide is the first? Aaadddaaammm (talk) 11:07, 6 March 2010 (UTC)[reply]

it would make sense for the first one to be the first one that was created while the strand of DNA was being copied. 82.113.121.104 (talk) 13:18, 6 March 2010 (UTC)[reply]
That would make sense, but I seriously doubt that's the case - often which nucleotide is copied first is not known exactly. I imagine the choice is somewhat arbitrary, and made by the team who initially sequenced the DNA. They may choose a prominent feature (like the start of a gene), or they may not. For example, the start position of the E. coli genome is about a million base pairs away from the origin of replication. In the footnote to the E. coli sequencing paper (Blattner et al. Science, 277(5331) page 1453) they note that the zero position was originally defined as the first residue of the thrA gene, which had been previously used in mapping the chromosome, but that they decided to move the start position for the complete genome 189 nucleotides upstream, to include the thrA promoter with the gene. I do know that once a choice has been made for one piece of DNA (say the pBR322 plasmid), derivatives/related DNA (like the pET vector series) tend to use the same numbering system. I'll also note that even for linear DNA like nuclear chromosomes, the numbering system is arbitrary, as telomeres, centromeres, and other unsequenced/unsequencable repeat systems make a global numbering system unwieldy/impractical. -- 174.21.226.184 (talk) 16:51, 6 March 2010 (UTC)[reply]
This is more of a question than an answer: Doesn't the loop have to detach into a more conventional linear sequence in order to reproduce? With 'normal' DNA, the two sides of the helix are unzipped and replicated - that doesn't seem reasonable for circular DNA (unless it doesn't do the "double helix" thing) because you'd end up with an inextricable tangle. If my guess is right then the circle has to break somewhere when being copied - and maybe there is a special place where that has to happen? If THAT is true - then that would be the logical place to start the numbering. But I have no clue whether any of that is true - I'd like to hear the answer. SteveBaker (talk) 17:35, 6 March 2010 (UTC)[reply]
(edit conflict) Answering SteveBaker's question: Various topoisomerase enzymes either cut one strand of the double helix and rejoins it after it has rotated around the other strand or cut both strands, let another double helix pass through, and rejoin the strands. But this happens many times during the copying of the DNA. Icek (talk) 17:54, 6 March 2010 (UTC)[reply]
For general unknotting of DNA, there are a whole group of specialized proteins: the topoisomerases. (See also this link.) TenOfAllTrades(talk) 17:49, 6 March 2010 (UTC)[reply]
DNA replication forks can be generated without cutting the DNA exactly at the site of the fork, as described indirectly by Theta structure and more comprehensively in these lecture notes. That page also discusses negative DNA supercoils, which can compensate for some of the twisting related to unwinding of DNA, and which are released by the topoisomerases cited above. Rolling circle replication is another relevant concept, especially for circular bacteriophage DNA and viroids. -- Scray (talk) 19:43, 6 March 2010 (UTC)[reply]
Here's a great, freely available, textbook chapter relevant to the current discussion - specifically, it deals with various modes of DNA replication. This doesn't answer the OP, but does address Steve's question (the short answer is "no"; circular DNA does not get linearized per se, but topoisomerases are needed to relax negative supercoils and the daughter segments have to be joined to form the circular "progeny" DNA). -- Scray (talk) 20:02, 6 March 2010 (UTC)[reply]

Surviving in a Vacuum

Last nights episode of QI said that a person can survive for a short time in a vacuum, basically until they can't hold their breath anymore and die from suffocation. Is this actually true? I would have thought they'd be exploded by the lack of pressure and boil alive because the boiling point in a vacuum is much lower. Also, Stephen Fry said someone in a vacuum would simultaneously defecate, urinate and vomit because of the pressure. Is that also true?

This has been discussed many times before (well not sure about the defecate, urinate and vomit bit), you may want to search to read the previous answers. But basically Space exposure covers it well. In particular note that it's estimate people could survive about 90 seconds but holding your breath is a bad idea Nil Einne (talk) 11:14, 6 March 2010 (UTC)[reply]
And the pee, poo and puke is from Decompression sickness, linked to in the Space exposure article. Nanonic (talk) 12:44, 6 March 2010 (UTC)[reply]
The BBC should invite three representatives from the Ref Desk along to be panellists on QI. Highest-scoring copmetitors ever, I think. Vimescarrot (talk) 21:54, 6 March 2010 (UTC)[reply]
How could Steven fry still come over as the all encompassing smart arse?--Aspro (talk) 23:12, 6 March 2010 (UTC)[reply]
For this covered in fiction, watch the movie Sunshine. Aaadddaaammm (talk) 09:40, 7 March 2010 (UTC)[reply]
Or even better the movie 2001: a space odessey. 195.35.160.133 (talk) 11:43, 9 March 2010 (UTC) Martin.[reply]
Cos he has all the answers on the card?

Space question

I'm trying to understand the concept of space but I just doesn't make sense to me. Space has nothing in it right? Except for planets and moons and bits of rock floating about. So why aren't those things sucked towards each other to fill the emptiness? How can there be an area with absolutely nothing in it? On earth we have air to fill the gap, but in outter space there's nothing? Please help me to understand. How can something exist when it's nothing? —Preceding unsigned comment added by W359 (talkcontribs) 16:50, 6 March 2010 (UTC)[reply]

Space (even the space between galaxies) does have very small amounts of 'stuff' in it - odd stray molecules of gas. The "air" that we breathe is also "mostly vacuum" - there are big gaps between the molecules. It's only when you get a liquid or a solid that the molecules come close to touching - and even then, the individual atoms have nuclei that are an unimaginably tiny fraction of the size of the atom - so even something as solid as a rock or a table is almost all empty space. The difference between what we call "air" and what we call "vacuum" is just a matter of degree. The reason the air stays in our atmosphere is the same as the reason we stay stuck on the ground - gravity. The reason that gravity doesn't suck all of the planets into the sun is because the are orbiting and as they whirl round in their orbits, the centrifugal force of that motion exactly counters the gravity of the sun. The sun and other stars don't get sucked into the giant black hole in the middle of our galaxy is because they too are orbiting. Galaxies sometimes do collide with each other and get pulled apart by each other's gravity. That's believed to be happening to our galaxy right now. But what (in general) stops galaxies from just being smooshed together by their own gravity is because space itself is stretching as the universe expands after the Big Bang. That expansion is enough to overcome the mutual gravitation between distant galaxies simply because gravity gets so weak over such enormous distances. SteveBaker (talk) 17:26, 6 March 2010 (UTC)[reply]
When you ask why things aren't sucked together to fill the emptiness, perhaps you're referring to the fact that on Earth a vacuum tends to collapse—for example, if I suck the air out of a glass bottle, the resulting (partial) vacuum will noisily collapse the instant I remove the bottle from my lips. The reason for this is the surrounding air pressure. There's really no such thing as a "suck"—everything is a "push". So it isn't that the vacuum is sucking air into the bottle; rather, the relatively high-pressure air outside the bottle pushes its way into the bottle. In space there is no surrounding air, so there is no similar push. In fact, if I were to put a cap on an "empty" bottle here on Earth, travel into space, and then remove the cap, the high-pressure air inside the bottle would immediately push its way out of the bottle and drift off into space, rapidly spreading out until it's almost a vacuum itself (the same almost-a-vacuum that existed there already), and leaving an almost-vacuum inside the bottle. A vacuum has no inherent "sucking" power, so the vacuum of space doesn't suck stars and planets and things together; everything that we call a "suck" on Earth is really a "push" from the surrounding air pressure. The important force at work in the vacuum of space is gravity, which Steve talked about above. —Bkell (talk) 18:06, 6 March 2010 (UTC)[reply]

Last night, I started the Orders of magnitude (voltage) article (I love the 'orders of magnitude' series of articles - they are really handy for answering Ref.Desk questions!). I've found good examples for every order of magnitude from 100V (a AA battery) up to 108V (typical lightning bolts) which is probably about the largest voltage out there. So we're good for the positive orders of magnitude. But what I'm missing is good examples (preferably common/not-obscure) of voltages of 0.1V downwards (preferably with something I can reference). Help! SteveBaker (talk) 17:49, 6 March 2010 (UTC)[reply]

Nano volts? Each is a minute capsule filled with a gel-like substance excreted by the shark, sensitive to electrical discharges as small as .005 microvolts... [5] and Ampullae of Lorenzini --Aspro (talk) 18:29, 6 March 2010 (UTC)[reply]
How much voltage is generated by a radio antenna (or the antenna in a cell phone, GPS receiver, or wireless networking card)? I can't find the answer immediately, but it can't be very much. Certainly less than a volt, I would think. —Bkell (talk) 18:37, 6 March 2010 (UTC)[reply]
Yeah - I wondered about that - it seemed like it ought to be a great example at the very low end of the scale. SteveBaker (talk) 19:35, 6 March 2010 (UTC)[reply]
As our EEG article says, typical EEG voltages measured from the scalp are in the 10µV to 100 µV range. Looie496 (talk) 18:41, 6 March 2010 (UTC)[reply]
The voltage imposed on a portable radio's antenna or a TV's rabbit ears is extremely low. The output of a typical microphone is extremely low. The voltage from Pioneer spacecraft received at the antenna is very low. The voltage imposed in a short wire by moving it in the Earth's magnetic field is quite low. I say "quite low" because I do not have time right now to go from deciBels and impedance to voltage or to do the basic physics calculations. Edison (talk) 21:04, 6 March 2010 (UTC)[reply]
The dBm article contains a suitable list of typical radio-frequency powers - conversion from dBm to volts is left as an exercise for the reader. :) Tevildo (talk) 21:26, 6 March 2010 (UTC)[reply]
Thermocouple voltages are typically in the micro Volts. This might be intersting to your average reader. Best of luck! 65.209.6.2 (talk) 22:39, 6 March 2010 (UTC)[reply]
See the article Johnson–Nyquist noise. A 50 ohm resistor at room temperature delivers upwards of 1 nV = 10-9 V r.m.s. Cuddlyable3 (talk) 00:19, 7 March 2010 (UTC)[reply]
Electronvolt? 82.132.136.207 (talk) 01:21, 7 March 2010 (UTC)[reply]
An electronvolt is a unit of energy/mass, not a unit of electrical potential (voltage). Its dimensions are "charge" * "potential difference"; so its an energy unit, just like Joule = Coulomb * Volt. --Jayron32 01:23, 7 March 2010 (UTC)[reply]
A bit like trying to use light-years to measure time. Tonywalton Talk 01:30, 7 March 2010 (UTC)[reply]
You also get a voltage due to random electrical thermal noise, this will be of the same sort of order as the weakest radio signal you can detect in an antenna. Also voltages from thermocouples are under 1 volt. Graeme Bartlett (talk) 06:17, 7 March 2010 (UTC)[reply]
Thermal noise voltage is Johnson-Nyquist noise already mentioned.Cuddlyable3 (talk) 18:01, 7 March 2010 (UTC)[reply]

Steve, we already have a list of voltages at Volt#Common_voltages. Maybe you should merge the two lists. --Heron (talk) 10:49, 7 March 2010 (UTC)[reply]

There is certainly an argument for replacing the list in Volt with a link to the new article - but there is a family of "Orders of Magnitude..." articles that have value simply from being in the same format and with common names...so merging my new article into Volt would not be acceptable. Having orders of magnitude articles for all of the common units (mass, length, time, etc) has huge value. SteveBaker (talk) 23:11, 7 March 2010 (UTC)[reply]

I agree. Actually I was thinking of getting rid of the list, not the OOM article. --Heron (talk) 18:12, 8 March 2010 (UTC)[reply]

Uncommon features of a protein

How do you know if a feature of a protein is uncommon or common among other proteins? By features, I mean things like a binding interface made up mostly of hydrophilic residues, a disulfide bond connecting two monomers in a dimer, etc. —Preceding unsigned comment added by 70.68.120.162 (talk) 18:01, 6 March 2010 (UTC)[reply]

Hydrophilic or hydrophobic areas can be determined from the DNA sequence, which is the most easily accessible information. Long-distance interactions are much more difficult to detect. Looie496 (talk) 18:44, 6 March 2010 (UTC)[reply]
While the presence or absence of hydrophobic and hydrophilic residues (or, more likely, their relative abundances) can be readily established from the gene sequence, that information almost certainly won't tell you whether those hydrophobic residues are surface-exposed or packed snugly in a globular protein's hydrophobic core. One doesn't know if hydrophobic residues, separated in the primary sequence, are grouped together to form a binding region on the folded protein. Similarly, one can't easily determine from the primary sequence whether or not a cysteine residue is free or if it forms a disulfide bridge, nor if it is on the surface as part of a dimerization motif. (And you're totally out of luck if you're hoping to predict something really weird like this: a heptasulfane bridge connecting two monomers in SOD1.) That said, there are tools for secondary structure prediction (and limited tertiary-structure guessing); see the ExPASy link below. TenOfAllTrades(talk) 19:10, 6 March 2010 (UTC)[reply]
For a curated database of protein structures, you might try SCOP ([6]) or CATH ([7]) — though it sounds like you're most interested in inter-protein binding...? Perhaps someone else here has suggestions for databases of protein-protein interaction motifs and post-translational assembly of protein complexes...? I might recommend looking through the list of tools linked from ExPASy here to see if there's anything that does what you're looking for. TenOfAllTrades(talk) 19:10, 6 March 2010 (UTC)[reply]

To answer the basic premise of the question, "how can you find out if a feature found in one instance is common in all instances", testing testing and yet more testing while recording the results of all the tests and building up a database of information which can be queried for correlations. —Preceding unsigned comment added by Bartender, bloodwine! (talkcontribs) 22:21, 6 March 2010 (UTC)[reply]

The answer will depend on how you classify your features, if you use a fine grained classification there will be only a few things in it, whereas broad schemes will include many members. If you stick with features named in literature or Wikipedia you can expect that there will be many members. Graeme Bartlett (talk) 04:17, 7 March 2010 (UTC)[reply]

perfect circle

Is there is no such thing as a perfect circle in nature? —Preceding unsigned comment added by RiseWave (talkcontribs) 19:18, 6 March 2010 (UTC)[reply]

The word 'perfect' is a Value judgment.--Aspro (talk) 19:37, 6 March 2010 (UTC)[reply]
No, a perfect circle is defined as a figure which has all points on its edge the same distance from its center. It's not based on a value judgment. --70.250.214.164 (talk) 20:23, 6 March 2010 (UTC)[reply]
Measured from the centre with what ? A knotted string, ears of barely corn, ... ect.--Aspro (talk) 20:59, 6 March 2010 (UTC)[reply]
Maybe the question should be better phrased '“have human engineers matched or exceeded the Accuracy and precision of circles that occur in nature? “ '(As for my example of 'perfection': I think octopus rings are 'perfect' circles of deliciousness unmatched (as yet) by any processed food manufacturer -but that is just my own value judgement based on my own pallet).--Aspro (talk) 21:45, 6 March 2010 (UTC)[reply]
The geometric circle is an abstraction in the geometry introduced in the books of Euclid's Elements. It does not exist as a physical object. Buckminster Fuller observes in Synergetics that it is unrealizable.
...We find local spherical systems of Universe are definite rather than infinite as presupposed by the calculus's erroneous assumption of 360-degreeness of surface plane azimuth around every point on a sphere. All spheres consist of a high-frequency constellation of event points, all of which are approximately equidistant from one central event point. All the points in the surface of a sphere may be interconnected. Most economically interconnected, they will subdivide the surface of the sphere into an omnitriangulated spherical web matrix...--Synergetics 224.07 Cuddlyable3 (talk) 00:11, 7 March 2010 (UTC)[reply]
The problem here is the word "perfect" - absolute perfection is impossible in the real world because the world is ultimately made of lumpy molecules with fuzzy edges and all sorts of wiggling with temperature and they have this annoying habit of moving when you try to measure their positions accurately. So there is no such thing as a perfect anything - in nature or otherwise. We talk about the perfect symmetry of snowflakes (they aren't even close to perfectly symmetrical) or that light moves in straight lines (well, not if there is anything with mass causing gravitation anywhere within an infinite distance). However, if you're prepared to live with some very tiny imperfections, nature can come pretty close. Drop a small rock in a still pond and the ripples will move outwards in almost perfect circles. SteveBaker (talk) 00:17, 7 March 2010 (UTC)[reply]
Right, in sufficiently deep-water pond the ripple from a small rock form a near-perfect circle. Also, halos around sun or moon are usually near-perfect circles. A rainbow also has a shape of a circular arc (not a complete circle, though). In still air the soap bubbles are near-perfect spheres. "Near-perfect" really means that the size of the deviations -- stochastic or systematic -- from the perfect circular shape are much smaller than the circle radius. Some of it is indeed due to the "fuzziness" of edges, to thermal motion, to medium anisotropy, to spontaneous symmetry breaking in a general sense, and so on. --Dr Dima (talk) 04:34, 7 March 2010 (UTC)[reply]
Is the sun a perfect circle within the "noise smaller than radius" train of thought? Aaadddaaammm (talk) 09:42, 7 March 2010 (UTC)[reply]
The apparent Sun disk may or may not be close to circular. It depends on the atmospheric conditions, and on how close to the horizon the Sun is. If you are talking about the Sun proper, and not about its apparent disk, then the question arguably becomes moot, as the Sun does not have a well-defined surface at all. The Sun has very low oblateness (of order of 10-6), so in the sphere-vs-ellipsoid sense the Sun is a near-perfect sphere. However, on a more local scale there are convection cells, filaments, etc., so the "surface" (if you pick a criterion to define one somehow) will have pits and bumps on it. Those pits and bumps will emerge and dissipate on a fairly short time-scale(s). See supergranulation, Coronal loops, Solar flares, Coronal Mass Ejection. --Dr Dima (talk) 19:16, 10 March 2010 (UTC)[reply]
The best freehand drawer of circles (video). Cuddlyable3 (talk) 17:53, 7 March 2010 (UTC)[reply]

Convergent evolution

Do species via convergent evolution have similar DNA because they evolved to be the same, even though they have different ancestors? —Preceding unsigned comment added by Firesidedragon (talkcontribs) 21:25, 6 March 2010 (UTC)[reply]

No. There's lots of ways that DNA could work to produce the same results. A superficial similarity in physical appearance does not mean any genetic connection. --Jayron32 22:09, 6 March 2010 (UTC)[reply]
A similar phenotype can be expressed by different genotypes. The farther apart two species are, the more likely convergent evolution does not result in the same physical features. For example, a bat's wing is very different from a bird's. Imagine Reason (talk) 22:43, 6 March 2010 (UTC)[reply]
And hummingbirds wings are way different from bees, even though both can hover in a flower. StuRat (talk) 05:14, 7 March 2010 (UTC)[reply]
Yea, convergent evolution usually refers to phenotypes, but in principle there's no problem with convergent DNA sequences. Aaadddaaammm (talk) 09:44, 7 March 2010 (UTC)[reply]

Appropriate Technology Film

Hi

I saw an excellent film about 1987 produced I think by the BBC and a United Nations agency looking at the use of appropriate technology in a number of different areas: housing, birth control (Spain), using simple medical workers in a Far Eastern country etc.

I have been unable to trace its name and production details. I would like to get hold of it and find out if there has been a follow up.

If you can help with the these details it would be much appreciated. If not you may be able to refer me to someone at the BBC who may be able to assist.

Thanks —Preceding unsigned comment added by Michael in Dublin (talkcontribs) 23:32, 6 March 2010 (UTC)[reply]

I found a list of videos about appropriate technologies here. You may find a clip from the film you saw. Cuddlyable3 (talk) 23:47, 6 March 2010 (UTC)[reply]
You might find it by using the advanced search of the BFI website [8] --80.176.225.249 (talk) 23:51, 6 March 2010 (UTC)[reply]

March 7

Saturn and Jupiter

If the mass of Jupiter and Saturn were combined would there be enough mass, density and gravity to result in fusion? 71.100.11.118 (talk) 01:41, 7 March 2010 (UTC)[reply]

No. See Brown dwarf - the theoretical minimum size of a star in which fusion occurs is about 75 times the mass of Jupiter, and the smallest known star is AB Doradus C with a mass 93 times that of Jupiter. Saturn's mass is only about 0.3 times that of Jupiter, so adding them together wouldn't make a significant difference. Tevildo (talk) 01:56, 7 March 2010 (UTC)[reply]
At 75 times the mass of Jupiter, fusion occurs. At more than 13 times the mass of Jupiter, a star becomes a brown dwarf and produces heat and light by the star shrinking under its own weight. --The High Fin Sperm Whale 03:03, 7 March 2010 (UTC)[reply]
According to our article, "brown dwarfs heavier than 13 MJ do fuse deuterium and those above ~65 MJ also fuse lithium." Both isotopes/elements however would be present in far smaller proportions than the hydrogen that begins to be fused from 75 MJ upwards, so what fusion does occur is thought to be unable to prevent the brown dwarf from cooling quite quickly (by stellar standards) below fusion-sustaining temperatures. 87.81.230.195 (talk) 06:21, 7 March 2010 (UTC)[reply]

is Jupiter made of the right stuff to be a star, if it were bigger? If so, then is the only difference between a planet like Jupiter and a Sun that the former is too widdle to fuse? Instead of planets then why don't we call them little not-stars? 82.113.121.94 (talk) 11:49, 7 March 2010 (UTC)[reply]

You find the reason why we call them planets in the lead and the first section of the article Planet. DVdm (talk) 11:56, 7 March 2010 (UTC)[reply]
And yes, Jupiter is made of the right material - hydrogen and helium. --The High Fin Sperm Whale 03:32, 8 March 2010 (UTC)[reply]

classifying reaction?

What kind of chemical reaction is Cu+2H2SO4-->CuSO4+2H2+SO2

thanks in advance...

174.112.38.185 (talk) 02:46, 7 March 2010 (UTC)[reply]

see Redox, hydrogen production, solution, corrosion, pollution. Graeme Bartlett (talk) 04:08, 7 March 2010 (UTC)[reply]
I would be surprised if this reaction could really happen. 93.132.150.52 (talk) 09:30, 7 March 2010 (UTC)[reply]
The caption of one of the images on the Copper(II) sulfate page says it can be made by electrolysis. It wouldn't surprise me as it seems, at a glance, to be strongly entropically favourable, with a solid metal and six aqueous ions forming two aqueous ions and three equivalents of gas, which would then bubble off, slamming the equilibrium in favour of the products. Brammers (talk) 10:12, 7 March 2010 (UTC)[reply]
The reaction is a magical way to hide 2 atoms of oxygen. Cuddlyable3 (talk) 17:40, 7 March 2010 (UTC)[reply]
This sort of thing can actually happen when you heat copper in sulfuric acid, you may not get hydrogen produced however, getting water produced at the end instead. Same thing happens with mercury (don't try this at home). Graeme Bartlett (talk) 22:12, 7 March 2010 (UTC)[reply]
Is SO4 a molecule that could exist? If the last molecule were SO4 rather than SO2, then Cuddlyable3's objection would be satisfied. Otherwise if O2 were evolved it could balance. (I know something of electricity, less of chemistry).(Gee, I have some sulfuric acid and copper in the basement. Should I try the experiment? Naah!).Edison (talk) 23:33, 7 March 2010 (UTC)[reply]
It called a reaction that don't work :-). Copper is the "wrong" side of hydrogen in the electrochemical series. Most metals will react to give off hydrogen, copper is the odd one out. Copper only dissolves with an oxidant - nitric acid will do nicely.  Ronhjones  (Talk) 23:47, 7 March 2010 (UTC)[reply]
Pure sulfuric acid though can oxidise copper, I checked it out in an Inorganic Chemistry text book, in the sulfur dioxide section, it is one way to make SO2. Graeme Bartlett (talk) 00:33, 8 March 2010 (UTC)[reply]

star birth, life,and death equation and graph

is there an equation and graph that shows each of the basic conditions, i.e., gravity, diameter, volume density rate of fusion of a star over time for each sequence (big bang to cloud of hydrogen through the collapse of an iron core to the formation of a neutron star and eventually a black hole and beyond) over time? 71.100.11.118 (talk) 04:29, 7 March 2010 (UTC)[reply]

No. StuRat (talk) 05:04, 7 March 2010 (UTC)[reply]
How about, instead, thinking about saying "Not that I know of", which would be more accurate; and then realizing that it's better to not post than just to post either "no" or "not that I know of"? Comet Tuttle (talk) 07:30, 7 March 2010 (UTC)[reply]
How about keeping your sanctimonious pedantry to yourself? (Oh, I forgot -- there's no WP:KYSPTY rule, so you're perfectly legit.) 63.17.52.111 (talk) 11:02, 7 March 2010 (UTC)[reply]
Do you really advocate the unreferenced answer "no" for a reasonable refdesk question? That doesn't improve the refdesk at all. We are a reference desk, and we're supposed to supply references, not lazy rejections. Comet Tuttle (talk) 15:56, 7 March 2010 (UTC)[reply]
I said no because that's the correct answer. Many equations and graphs and charts and diagrams are needed to show all of that, it can't possibly all be put into one. And how exactly am I going to find a source proving that something doesn't exist ? That's just silly. If you think I'm wrong, let's see some sources supporting your position. StuRat (talk) 16:01, 7 March 2010 (UTC)[reply]
I'm not asserting that you're wrong; I'm asserting that an unreferenced "No" is an unacceptable answer on the refdesk. If you had typed the above paragraph as your original answer, I would not have complained at all. Comet Tuttle (talk) 16:55, 7 March 2010 (UTC)[reply]
It's still an unreferenced "No". Are you saying that it's now acceptable because I explained why the answer is no, or because I explained that it's impossible to provide a source for something which doesn't exist ? StuRat (talk) 20:36, 7 March 2010 (UTC)[reply]
Absolutely that would be a better answer. "No - because you can't fit all that stuff on one sheet of paper" would have been a useful answer. Particularly if you could explain why not. Just "No" is a very low-information-content response. So yes - your first answer was awful - with the clarification, it's borderline useful. The best answer would have been "No, you can't fit all of that on one sheet of paper - but here, here, here and here are the separate graphs and charts that will give you the information you need." As you will note below, the answer "Have you read out Stellar evolution article?" was judged by the OP to be useful. SteveBaker (talk) 23:02, 7 March 2010 (UTC)[reply]
They said it contained some useful info, but that it didn't answer the Q. The answer to the Q was "No". We can explain why that's the answer, but I doubt if you will find any links that say why you can't fit all that info on one page, which is what was actually asked. StuRat (talk) 00:08, 8 March 2010 (UTC)[reply]
I agree that making such a chart would be very difficult - but if you just say "No" then that might have been because the chart simply doesn't exist because nobody could be bothered to make one - or it might be because the information isn't available - or it might be because the sheet of paper would have to be too big - or (the truth) it might be because the resulting chart would be an unreadable mess. But the reason that there isn't such a chart is really very important. The purpose here is to help our questioner - not just to provide a literal boolean answer to the question. Your "No" response was true but 100% useless since one's first feeling on reading that is "How the hell could this guy possibly know that such a chart doesn't exists? He hasn't read every astronomy book in the world!" - so you aren't conveying the information needed for the answer to be remotely believable. Sure you can't find a reference for that - but there were ways you could have said something like: "There are X kinds of star and Y different parameters to be graphed for each star type so there would be X x Y lines on that graph and it would need Y columns of numbers on the vertical axis and you'd need to draw all of those curves with X x Y colors or line styles." Assuming that X and Y are reasonably large numbers - it would be clear why such a diagram would either be impossible, or really cluttered or actually unreadable. SteveBaker (talk) 04:55, 8 March 2010 (UTC)[reply]
While I agree that going beyond simply answering the question (as I did) to explain in depth how that answer was derived might be helpful, I don't agree that I should either do that or not answer. There's no requirement that each individual response be comprehensive. Thus, one person can answer and another can elaborate. I've often done the elaboration part myself. StuRat (talk) 10:26, 8 March 2010 (UTC) [reply]
How about for each sequence, including non-wiki articles? 71.100.11.118 (talk) 05:15, 7 March 2010 (UTC)[reply]
Have you read our Stellar evolution article? Comet Tuttle (talk) 07:30, 7 March 2010 (UTC)[reply]
Thanks for the ref. That article has some good stuff in it. What I had in mind though was a chart where core gravity, diameter, volume, density, rate of fusion, etc. (all of the basic measurements were plotted over life cycle and the equations that correlate rate of fusion with star diameter, etc. 71.100.11.118 (talk) 12:21, 7 March 2010 (UTC)[reply]

Life expectancy

In an interview I was watching, a man claimed that in the cave man era, if infant mortality were ignored, the average life expectancy of people would be roughly 70 years. I'm a little skeptical, is there any evidence that this is true? —Preceding unsigned comment added by 173.179.59.66 (talk) 09:19, 7 March 2010 (UTC)[reply]

The move away from nomadic hunter gather to neolithic settlements appears to have drastically increased the disease burden; which got worse as villages became towns and towns grew into cites. Things did not improve until the introduction of modern sewerage systems. There is little hard physical evidence to support the idea that palaeolithic people had longer life spans but there are good augments to suggest this: Grandmothering, menopause, and the evolution of human life histories--Aspro (talk) 11:10, 7 March 2010 (UTC)[reply]
The fact that human females are able to live for so long after they are no longer able to bear children suggests that there is evolutionary benefit to living beyond ~40 years that is nothing to do with reproductive ability. People who outlived their reproductive usefulness would consume food and other resources that the rest of the tribe needed - and that would impose evolutionary pressure for people to die at an age just beyond that at which their children became mature. People with the "die at 40" gene would have more successful children - and that gene would outperform the "die at 80" gene that would result in twice as much food consumption by a group of humans that would limit the number of surviving children into future generations.
But if that were true - then we wouldn't live much longer than 40 years...and we obviously do. Most other animals die of old age not long after they cease to be fertile. No amount of good health care, nutrition, comfortable living, etc will allow a dog to live more than about 15 years - or a mouse beyond about 3 years. But both are fertile until almost the end. So for humans to be so different, there must have been other benefits of long life (eg the ability to bear knowledge across multiple generations - or that 40 years of experience make for a better hunter than 20 years - so that the older people could produce more benefit than they 'cost').
We haven't really had time to evolve that long-life ability recently - so it was probably around in palaeolithic times - which probably means that at least some people lived to these kinds of ages. What's tough is to know the average age back then. Did just 1% live to 70 or did most people who survived childhood live that long? SteveBaker (talk) 22:52, 7 March 2010 (UTC)[reply]
The long childhood of humans meant that parents need to stay around for a much longer time after their last kid is born, if they expected it to survive. StuRat (talk) 01:47, 8 March 2010 (UTC)[reply]
Fascinating stuff, thanks for the detailed answer. By the way, is there any particular reason why diseases were more prevalent in sedentary societies? And how then were agriculture-based societies able to out-develop their nomadic counterparts? —Preceding unsigned comment added by 173.179.59.66 (talk) 01:12, 8 March 2010 (UTC)[reply]
Sedentary societies usually involve higher population densities and close contact with animals. Both of those factors contribute to increased disease. Agriculture allows people to produce more food than hunting and gathering (in some environments, anyway), which allows some people not to work on getting food. That frees them up for developing other crafts and for politics (which allows better coordination of large groups). --Tango (talk) 01:37, 8 March 2010 (UTC)[reply]
Unless such a statement came from a respected member of a culture that ostracizes community members who make controversial statements of fact without evidence (i.e., a scientist), I would be deeply skeptical that this is just an appeal to nature. --Sean 19:34, 9 March 2010 (UTC)[reply]

Temperature of outer space

If space is a vacuum, how can it be a temperature be associated with it? 173.179.59.66 (talk) 09:22, 7 March 2010 (UTC)[reply]

There is not usually a temperature associated with the vacuum of space. If someone does refer to the temperature of space, they usually mean the temperature of the cosmic background radiation or of the very low-density interplanetary medium or interstellar medium.83.134.173.114 (talk) 10:20, 7 March 2010 (UTC)[reply]

So the vacuum of space is temperatureless? —Preceding unsigned comment added by ApplePie456 (talkcontribs) 11:39, 7 March 2010 (UTC)[reply]
No, vacuum can still have radiation which wiil have a temperature. That would be the cosmic background radiation mentioned above.Dauto (talk) 14:14, 7 March 2010 (UTC)[reply]
The vacuum of space apparently does not hve a "temperature" other than the extremely low temperature of the cosmic background radiation referred to above. But an object in space will have a temperature, as would be measured by a thermometer in it or on its surface. A spacecraft in orbit will get very hot on the side facing the sun, depending on its absorption of the solar radiation, which amounts to something over 1 kilowatt per square meter. A white spacecraft would absorb less energy than a black one. See Albedo. The same is true for an object on the surface of the Moon where the sun is shining. Conversely, a spacecraft (or the side of a spececraft) which is in the dark will radiate off heat and can get extremely cold. NASA astronauts referred to the rotation of a spacecraft to even out the temperature "barbecue mode." Coolant can be circulated through tubes to even out the temperature. In the ill fated Apollo 13 moon mission, the spacecraft had to power down and it got very cold inside while they were travelling back to Earth after passing around the Moon. If the outside of the craft had been painted to absorb more solar energy, it would have been warmer. Edison (talk) 14:21, 7 March 2010 (UTC)[reply]
The key thing to remember about our definition of temperature is that it requires matter (unless you're talking about the background radiation mentioned above). You can only refer to conventional temperature when there is matter to study the kinetic energy of, so any floating material in the 'vacuum' of space (I use the term lightly, as there's actually quite a bit of matter floating around) will have a temperature, but the empty parts won't (except for the cosmic background radiation, see above). Regards, --—Cyclonenim | Chat  15:00, 7 March 2010 (UTC)[reply]
You say you use the term vacuum "lightly" to refer to outer space. I have read that space has a hard vacuum, equal to or better than any that can be achieved with the best technology on Earth. Maybe our vacuum pump technology has improved since I was last concerned with the quality of obtainable vacuum. Any vacuum tube would supposedly work fine there. Do you have a reference that supports your contempt for the quality of the vacuum seen in space, compared to what a vacuum pump can achieve on Earth? Edison (talk) 22:58, 7 March 2010 (UTC)[reply]
There is no reason to see the radiation temperature as any different from the temperature of matter and since radiation permeates space it is always possible to define the temperature of vacuum and by the third law of thermodynamics it is always different from zero. Dauto (talk) 20:29, 7 March 2010 (UTC)[reply]
Right. Cycloneim, this is important: Temperature per se has nothing whatsoever to do with kinetic energy per se. Temperature is defined by the relationship between entropy and internal energy. The only time you can use kinetic energy as an accurate proxy for temperature is when you're dealing with a monatomic medium, so that none of the energy goes into vibrational modes. --Trovatore (talk) 20:34, 7 March 2010 (UTC)[reply]
Actually I take back the "nothing whatsoever" — that was too strong. Certainly kinetic energy will increase when temperature does, and that in itself is some relationship.
The point I wanted to make is that many people seem to think that temperature is conceptually a matter of kinetic energy. And that's completely wrong. --Trovatore (talk) 20:46, 7 March 2010 (UTC)[reply]
Then I apologise, I'm no physicist but closer to a biologist so I suppose my definitions are very limited to molecular structures. Temperature is indeed a measure of kinetic energy in particles though, isn't it? If not, someone needs to go fix the article temperature if that's not it's primary definition, because that's the very first sentence in the lead. Regards, --—Cyclonenim | Chat  22:10, 7 March 2010 (UTC)[reply]
To the extent that the particles are like billiard balls, whose internal structure can be ignored, and which don't interact except via elastic collision, you can treat it that way. But molecules you specifically can't. Molecules have vibrational and rotational modes. The temperature gives you a unit of energy that is distributed equally among the modes, not equally among the particles. --Trovatore (talk) 00:26, 8 March 2010 (UTC)[reply]


Another way to see that the energy formulation (even when modes are considered) is inadequate is to note that some systems have negative temperature. (You can't have a whole object at negative temperature, but you can have, say, its system of magnetic spins at negative temperature.) But this certainly doesn't mean negative energy! Actually, it's higher energy than a positive-energy positive-temperature state of the same system. --Trovatore (talk) 00:38, 8 March 2010 (UTC)[reply]
The problem here is that there are at least two subtly different meanings for this word "temperature". Classically, it's a measurement of the speed that molecules are jiggling around in a liquid, solid or gas - and in that sense, a vacuum really doesn't have a temperature. However, you can talk about the kinetic energy of those molecules as "heat energy" which can be related to a temperature - and when you do that, you can speak meaningfully of "temperature" as a more general measure of the energy present. In that interpretation, the photons and other radiation within the vacuum "counts" in that energy measurement - so now you can talk about the temperature of a vacuum. The distinction is another one of these "what does this word mean" problems rather than some physical thing. SteveBaker (talk) 22:30, 7 March 2010 (UTC)[reply]
No, the "speed" version is really just wrong. See above. --Trovatore (talk) 00:40, 8 March 2010 (UTC)[reply]

OP here. So to make sure I understood, the temperature of outerspace near the earth would be quite high, because of the contribution of the sun's radiation, and this would be true regardless of whether there's actually any matter present. Meanwhile, far from the solar system (or any star for that matter), space is very cold, with a temperature associated with the cosmic microwave radiation. Is this correct? —Preceding unsigned comment added by 173.179.59.66 (talk) 01:30, 8 March 2010 (UTC)[reply]

Well, no, it's not that simple. "Outerspace near the Earth" is not in thermal equilibrium, so it doesn't have a single well-defined temperature. What it has, you might say, is multiple temperatures, for different subsystems that are in some sort of equilibrium. For example, the solar wind is very hot indeed (much hotter than the radiation temperature of the Sun) -- millions of degrees if I recall correctly, which I may not. The reason you don't get fried by it instantly is that it has very low heat capacity and thermal conductivity, so the interplanetary medium right next to your spacecraft quickly delivers its negligible heat and cools off, whereas the stuff just a little farther away can't get its heat to you efficiently at all, so it doesn't bother you.
On the other hand the cosmic microwave background is also there, and it's at 2.7 K (if I remember the number right), as always. --Trovatore (talk) 08:24, 8 March 2010 (UTC)[reply]
And there is also the radiation coming from earth at roughly 300K and the cosmic neutrino background at 1.8K, etc... 174.58.105.234 (talk) 21:12, 8 March 2010 (UTC)[reply]

mind waves

is there any thing like mind waves? can a person just receive those mind waves? what type of energy are they really sound waves? —Preceding unsigned comment added by Myownid420 (talkcontribs) 09:50, 7 March 2010 (UTC)[reply]

You might want to read Electroencephalography. 93.132.150.52 (talk) 11:56, 7 March 2010 (UTC)[reply]
By recording cortical evoked potentials, it has been possible to read the mind to a limited extent. The Pentagon theough DARPA funded such experiments in the 1970's. See [9]. The thing being collected is very weak electrical potentials on the scalp which result from electrochemical activity in various brain regions and neural pathways. It is a very indirect measure, like determining when there are breaks in a televised sports program by noting the variations in water pressure due to toilet flushing. Edison (talk) 14:29, 7 March 2010 (UTC)[reply]
There is electrical activity in the brain - and where there is variation of electrical potentials, there are (inevitably) radio waves. However, the voltages are incredibly low and so are the frequencies - so those waves are incredibly, insanely weak - vastly too weak to make it past the skull - let alone any distance beyond that. Also, disentangling the results of a few billion neurons that are firing (seemingly) randomly is utterly impossible. The only way we have to pick this up is with sensors attached close to the scalp in the form of some kind of helmet - and even then, the signal strengths involved are on the lower edge of detectability. Because of the "firing all at once" problem, only the simplest information is obtainable in this way. We can pick up simple overall, slow speed activity this way (See: Electroencephalography).
We should VERY clearly state, however, that we can't "receive" those waves in our brains. That's absolutely and utterly impossible. Firstly we have no radio signal reception sensors in our heads (if we did, we'd be totally deafened by the insanely high powered TV, Radio, Cellphone, etc signals that are shooting through us all the time!). Secondly, the signals involved are so incredibly weak that it would be impossible - even in principle to detect them at any distance whatever. Thirdly, there is no way to extract information from the cacophany of activity within those billions of neurons without having some spatial information. So telepathy without intrusive electronics is physically quite impossible.
Having said that, an exceedingly crude form of telepathy has been implemented by Kevin Warwick (his article is worth reading). He has about 100 electrodes in an implant attached to the "median nerve" of one of his arms. A computer can measure nerve impulses that he sends that way...and he has demonstrated moving a robot arm using his mind alone. In a subsequent experiment, they implanted another set of sensors into his wife and they can now communicate by mentally activating their arm muscles and causing some kind of tingly sensation in the other person. It would be possible to learn to communicate (eg using Morse Code) by thought alone...but that's an exceedingly long way from simply "reading thoughts". This is more like a rather clunky wireless morse code transmitter that you have implanted in your body. However, "brain waves" aren't involved here - it's a conscious effort to transmit a 'tingle' and the resulting feeling is just like a tingle on the skin - not a mental image or speech or anything like that which might just "pop up" in your head.
SteveBaker (talk) 22:24, 7 March 2010 (UTC)[reply]

“Mind waves” that can be used for “telepathy” may be said to exist after all ! (Note the quotation marks signaling unusual usage!)
Well, when you ask:

1. Is there any thing like mind waves?
2. Can a person just receive those mind waves?
3. What type of energy is it – are they really sound waves?

Then one might jump to the conclusion that what you are asking is: "Does telepathy (i.e. magical thought transferring) really exist?" As far as I know the answer to that question is simply:

"No! And the electromagnetic waves that do exist in the brain, may not (at least not yet) be used for any thought transfer resembling telepathy. So any belief in telepathy is mere superstition!"

BUT if one looks at those three questions from a slightyly different angle, interpreting: "any thing like mind waves" to mean: "anything that appears to work just as “telepathic waves” would work", then the truthful answers suddenly become:

1. Yes!
2. Yes! And it does not even require any training!
3. Yes they are surely a kind of sound waves!

Here it may be appropriate to repeat a quote which tend to pop up whenever people discuss magic, namely “Clarke's third law” which states that: "Any sufficiently advanced technology is indistinguishable from magic."
In this case, the advanced technology, that resembles telepathy, is the so called: "Directional loudspeaker" which makes it possible to direct sound from 5 meter (15feet?) away (or more?) towards one or two people in a crown, in such a way that nobody else in the crowd can hear any of it!

A few years back one popular-science television show had a story where they prooved that a hand-held directional speaker, located on a gallery at the side of the room, was powerful enough to play dance music to a dancing couple in front of a library reference desk, WHILE nobody else in the relatively silent library room could hear anything of the music! :-)

Unfortunately I do not have the exact reference to that story, but you may enjoy reading the article: "How directional loudspeakers work: An introduction to directional sound and how it's used in advertising, exhibitions, and the military." It is an explanation from ExplainThatStuff.com, in easy (not too scientific) language.

P.S.
Remember this, the next time you meet or hear of someone who "hears voices" in their head.
And be mindful of that maybe not all the prospective members of the "Hearing Voices Movement" (at least initially) are quite as psychotic or insane, as some mental health care workers would bet their reputation on that they are. Because, in fact, someone might just be playing a cruel trick on the victims – using directional speakers. 8-)
--Seren-dipper (talk) 00:04, 9 March 2010 (UTC)[reply]

Corrosion of copper

Does corrosion of copper involve esterification? —Preceding unsigned comment added by Kencankan (talkcontribs) 09:58, 7 March 2010 (UTC)[reply]

You might want to read Organometallic chemistry. 93.132.150.52 (talk) 11:56, 7 March 2010 (UTC)[reply]

Cow pats

Can methan be extracted from cow pats? Why doesn't the farmers collected the cow pats and instead of putting them on rose plants as fertilizer use them for methan production? —Preceding unsigned comment added by Firendofdarkness (talkcontribs) 10:52, 7 March 2010 (UTC)[reply]

Yes, see :Anaerobic digestion. Collecting cow pats by hand however is labour intensive. It might be more economical to burn them as fuel. --Aspro (talk) 11:24, 7 March 2010 (UTC)[reply]
Not generally "cow pats" as found on grassy ground where they are grazing - but the 'slurry' that comes from concrete areas where cows live and are milked, etc is increasingly used by farmers. They can scrape it up and pump the semi-liquid mix of cow poop and pee into methane-producing "digesters" that produce power for lighting and heating cow sheds. SteveBaker (talk) 22:03, 7 March 2010 (UTC)[reply]

pus-like sinus stuff

What is the strong-smelling, pill-shaped puslike substance found in the sinuses? I know, that whole thing sounds horrible. --Chris (クリス • フィッチュ) (talk) 13:01, 7 March 2010 (UTC)[reply]

We cannot give medical advice. Have you read the article Mucus and the truly disturbing article Dried nasal mucus? Edison (talk) 14:23, 7 March 2010 (UTC)[reply]
How can pus be pill-shaped? Pus is a viscous liquid of sorts. DRosenbach (Talk | Contribs) 17:06, 7 March 2010 (UTC)[reply]
It can be pill-shaped if it's enclosed in an abscess, perhaps? Either way, go to your medical practitioner to check it out. Regards, --—Cyclonenim | Chat  17:15, 7 March 2010 (UTC)[reply]
How is it "found in" the sinuses? Cat scan? Surgery? Autopsy? Endoscopic exam? Or do you mean expelled by nose-blowing? Again, we will not provide diagnosis or medical device, but can discuss theoretical questions and refer a questioner to appropriate references such as Sinusitis, Rhinitis, Paranasal sinuses, or Pus. depending on how we, acting as reference librarians, understand the question. Anyone who thought they might have a sinus infection should consult a doctor rather than random people on the internet. Edison (talk) 23:08, 7 March 2010 (UTC)[reply]
In the same geographical ballpark, and roughly pill-shaped and strong-smelling, are the tonsiliths. --Sean 19:39, 9 March 2010 (UTC)[reply]

Pineapple

I want to grow a pineapple plant. Can I just cut the top off an existing pineapple and plant it? Will it grow into a pineapple bush? —Preceding unsigned comment added by Hawclawlaw (talkcontribs) 14:45, 7 March 2010 (UTC)[reply]

I think pineapple seeds are in the lower portion of the fruit, in the interior of the fruit. I think they're pretty awkward to find in a pineapple you might have bought from the supermarket though as they're often removed. Regards, --—Cyclonenim | Chat  14:56, 7 March 2010 (UTC)[reply]
Actually, the original poster has it right. However, it's only going to grow in tropical (and maybe subtropical) regions. StuRat (talk) 15:51, 7 March 2010 (UTC)[reply]
It will grow in many places, but it will only fruit in tropical climates. --Tango (talk) 20:19, 7 March 2010 (UTC)[reply]
A Google search threw this up for me, which seems to be what you're after. I can't tell you how accurate the site is, but it seems ok to me. Brammers (talk) 15:19, 7 March 2010 (UTC)[reply]
Here's another interesting series of questions and answers about growing them. Scroll down to mid page for some good photos.Bielle (talk) 16:45, 7 March 2010 (UTC)[reply]
They'll grow anywhere as a house plant, too. I've always wanted to grow one, have tried a couple of times but has never worked. Good luck! Aaadddaaammm (talk) 17:13, 7 March 2010 (UTC)[reply]
Cut the top off with 1 inch of the fruit, plant in a very sandy soil mix, water sparingly. I've done in even in the UK - won't fruit, and won't survive any hint of frost.  Ronhjones  (Talk) 23:50, 7 March 2010 (UTC)[reply]
I seem to recall that, if you want them to fruit, you need the soil to be warm as well as the air: I saw a really interesting programme a while ago(in the UK), about how rich estates grew pineapples in England before they could be imported. Anyone know what that programme would have been? Anyway, it seemed to involve undersoil heating, because using a heated greenhouse wasn't enough. You could always try... 86.177.121.239 (talk) 00:03, 8 March 2010 (UTC)[reply]
I think it was the Victorian Kitchen Garden on the BBC. --TammyMoet (talk) 15:14, 8 March 2010 (UTC)[reply]

Oldest survival animal species?

I would ask the oldest surviving species in general, but I assume someone would come back with cyanobacteria or something similar. I'm looking for the oldest surviving species in what today we consider to be in one of the major branches of animalia (e.g. mammals, fish, birds etc)? Regards, --—Cyclonenim | Chat  14:54, 7 March 2010 (UTC)[reply]

Sorry to respond with a question, but to give a reliable answer I think we'd have to agree on the definition of Species, and how we would apply that to the available evidence. We cannot attempt interbreeding, nor can we sequence enough DNA from tissue recovered from an organism that lived (say) 100 million years ago and compare in the manner that might be required for certainty today. It's quite difficult to assign species-level taxonomy using fossils from many millions of years ago. Do you have a well-founded proposal for the definition to be used, before people start proposing candidate species? -- Scray (talk) 15:12, 7 March 2010 (UTC)[reply]
Sorry, what I meant was are there any species around now that we know (from say, as you mentioned, fossil records) existed a long time ago, and which is the oldest? Clearly eukaryotic creatures are likely to have changed at least a bit in hundreds of millions of years, compared to say cyanobacteria which are remarkably stable. Regards, --—Cyclonenim | Chat  17:13, 7 March 2010 (UTC)[reply]
I still don't think I've explained the issue clearly enough. For example, if we were to kill a Tree Swallow, retain only the bones, compress them a bit, and fossilize them, would we be able to confidently differentiate that fossil from a Golden Swallow? I only choose this as an example, to illustrate how specific a species designation is. Back to your clarification, then, how can we "know" that any fossil corresponds to an extant species? Fossils provide only some of the distinguishing characteristics. Your question is great, because it challenges us to think about what a species is. -- Scray (talk) 17:23, 7 March 2010 (UTC)[reply]
So because our current definition of a species is two organism which can reproduce to create fertile offspring, that becomes irrelevant when looking at ancient animals because we have no method of studying whether their DNA is compatible? Not being an paleontologist I cannot answer you question, but presumably there are differences in bone anatomy between any two species, but I have no citation for that. If there isn't any difference physically, then clearly we would not be able to ascertain which is which. So is this search for the oldest known living animal pointless? I don't think so, since paleontologists clearly do know that, say, horseshoe crabs have been around for approximately 450 million years since the fossil record shows that the horseshoe crabs existing then are at least stupidly similar to the anatomy of those that are alive now. So, going on that method, which is the oldest surviving 'species'? Regards, --—Cyclonenim | Chat  19:36, 7 March 2010 (UTC)[reply]
This is the Science reference desk, and I'm just pointing out that getting a good scientific answer depends on defining the question. You've got some pretty interesting answers already, but to weigh them (for the superlative "oldest" you seek) the question would need to be defined. I'm not criticizing you - from the start I thought that this raised an interesting challenge. -- Scray (talk) 19:49, 7 March 2010 (UTC)[reply]
Sorry if you thought I was having a go, it wasn't supposed to come across that way. I think I just underestimated how complicated the question could be. Regards, --—Cyclonenim | Chat  22:02, 7 March 2010 (UTC)[reply]
If we go by animals that we would "recognize" as being similar to a modern species, how about the millipede, which goes back some 428 million years ? See Millipede#Evolution. StuRat (talk) 15:44, 7 March 2010 (UTC)[reply]
The brachiopod genus Lingula comes pretty close, having been around for more than 500 million years, not sure about a species though. Mikenorton (talk) 16:51, 7 March 2010 (UTC)[reply]
Theres Crocodile at 200 million years--Jac16888Talk 17:32, 7 March 2010 (UTC)[reply]
Take a look at the Crocodile article - that's a Family designation; there are 2 sub-families, 5 major genera within one of those, and about a dozen species for the most common genus. The OP asked about species - are we talking about those now, genera, families, or just living things with bones that look kind of similar to some fossil? -- Scray (talk) 17:58, 7 March 2010 (UTC)[reply]
Like millipedes, the oldest known scorpion fossil dates to about 430 million years ago. A beautiful creature, that. – ClockworkSoul 17:59, 7 March 2010 (UTC)[reply]
Tardigrade —Preceding unsigned comment added by TrialicWave (talkcontribs) 18:59, 7 March 2010
If you're after animal species that are anatomically unchanged for several million years, then the living fossil article has several examples. 62.56.68.180 (talk) 01:20, 8 March 2010 (UTC)[reply]
I've heard triops cancriformis called the oldest animal species known to still exist. 86.178.167.166 (talk) 01:03, 10 March 2010 (UTC)[reply]

Extra Life expectancy for the health-concious

Are there any figures available for the extra life-expectancy of people who eat healthy foods, stay slim, exercise, do not smoke and seldom drink, compared with the average? Thanks 89.242.102.148 (talk) 16:14, 7 March 2010 (UTC)[reply]

Yes. Basically the standard life expectancy models can be extended for this, by looking at specific countries. For example, in Africa or India it's unlikely that people will be drinking and smoking heavily while consuming vast amounts of fast food, whereas in the Western world those forms of consumption are very common.
That sounds horribly imprecise. I'd expect health care access and capacity in the US, European nations, Africa, and India vary widely, among other things. I think a better study would be to talk to each region and ask individuals how much they pay attention to what they eat, how physically adept they are, and track people over their lifetime keeping this in mind. Mac Davis (talk) 19:21, 7 March 2010 (UTC)[reply]
Unfortunately I think smoking is common or at least becoming commoner in many third-world countries, eg China, and they also often have much higher rates of fatal diseases such as AIDS, higher infant mortality, malnutrition, and deaths due to wars. So it would be difficult to find a Shangri-La like that. 78.146.208.26 (talk) 21:47, 8 March 2010 (UTC)[reply]

You'll be interest in things such as Life Tables and in general Actuarial science. I don't know the stats but as MacDavis alludes to location of the individual is a huge contributory factor. I'm sure someone will be able to link some stats but not got time myself. ny156uk (talk) 23:04, 7 March 2010 (UTC)[reply]

Telescope

What is the difference between reflecting and refracting telescope? -- Extra999 (talk) 23:00, 7 March 2010 (UTC)[reply]

The optical telescope article explains this, but basically, a reflecting telescope uses mirrors, and a refracting telescope uses lenses. CS Miller (talk) 17:41, 7 March 2010 (UTC)[reply]
Or in simpler and more obvious terms; a reflecting telescope reflects light, while a refracting telescope refracts light to produce an image. —Preceding unsigned comment added by TrialicWave (talkcontribs) 18:46, 7 March 2010 (UTC)[reply]
Not just any old mirror, but one ground into a parabolic shape. For a number of reasons, the largest "visual" telescopes in the world are reflectors. Radio telescopes such as the Very Large Array also use "reflection" to gather signals. ←Baseball Bugs What's up, Doc? carrots20:25, 7 March 2010 (UTC)[reply]
Curved, but not necessarily parabolic. Ritchey-Chrétien telescope designs use two hyperbolic mirrors, for example. And the Schmidt camera uses a spherical primary mirror, but also has a refractive corrector plate. -- Coneslayer (talk) 13:31, 8 March 2010 (UTC)[reply]

Intron question

Do introns contain DNA from other species? —Preceding unsigned comment added by Celidia (talkcontribs) 18:22, 7 March 2010 (UTC)[reply]

Depending on what you mean by "DNA from other species", they can. It's not at all clear where introns originated, but some have speculated that (at least some) introns are derived from transposons (mobile DNA elements). Certain transposons (especially some retrotransposons) are thought to be derived from viruses (like retroviruses) which infected a cell, and then lost the ability to construct infectious viral particles). Viruses which insert and remove themselves from the host chromosomal DNA have been known be "inefficient" and take a chunk of host DNA with them when they leave. - The scheme goes like this: a virus excises itself from the host DNA, taking a chunk with it. It then infects a different species (which, depending on the virus, is a relatively common occurrence), inserting the other species DNA into the genome. This inserted sequence, over a very long period of time, eventually loses it's infectivity, leaving the chunk of foreign DNA in the cell. However, while this scheme sounds plausible to me, I'm not aware of any specific examples of it. - You may also be interested in the topic of horizontal gene transfer, which is the general mechanism by which DNA of one species gets transferred to another. -- 174.21.235.250 (talk) 18:55, 7 March 2010 (UTC)[reply]
And, in case you don't already know, our functional DNA also "contains the DNA of other species". We share over 98% of our DNA with chimps, for example. StuRat (talk) 20:24, 7 March 2010 (UTC)[reply]

Carbon planets

Resolved

Our article carbon planet says that the protoplanetary disc is carbon-rich and oxygen-poor. Why does it need to be oxygen-poor? Or, I suppose: What would happen if it was rich in both carbon and oxygen? Vimescarrot (talk) 18:41, 7 March 2010 (UTC)[reply]

Presumably if oxygen was present, it would react with the carbon, forming carbon dioxide. As this is a gas, it wouldn't settle on the surface to form a layer. CS Miller (talk) —Preceding undated comment added 18:49, 7 March 2010 (UTC).[reply]
And if by chance it did settle on the surface, it would cause rapid global warming because carbon dioxide is a greenhouse gas —Preceding unsigned comment added by TrialicWave (talkcontribs) 18:50, 7 March 2010 (UTC)[reply]

Thanks. I suppose what I really meant was, if it was oxygen-rich, why couldn't it still be a carbon planet? Why did Marc Kuchner think they need to exclude oxygen for it to be a carbon planet? Vimescarrot (talk) 19:37, 7 March 2010 (UTC)[reply]

If it contained both lots of carbon and oxygen, like Earth, wouldn't it develop in a similar manner ? I think the point is that carbon only develops into diamonds and such in the absence of oxygen, which is why diamonds are rare, here on Earth, where such oxygen is plentiful. StuRat (talk) 20:19, 7 March 2010 (UTC)[reply]
Okay...I think that answers it. Thanks. Vimescarrot (talk) 09:04, 8 March 2010 (UTC)[reply]
You're welcome, and I've now marked this question as resolved. StuRat (talk) 10:18, 8 March 2010 (UTC)[reply]

Brain calories

Does your brain burn more calories when you're thinking hard? When does it burn more calories (stress?) and less calories (sleep?), and has this been measured? Mac Davis (talk) 19:23, 7 March 2010 (UTC)[reply]

A rather brief and basic view of the brain's energy consumption during thought is given here. Let me know if it doesn't address your question. Regards, --—Cyclonenim | Chat  19:40, 7 March 2010 (UTC)[reply]
The answer is that it burns more calories when active but not a lot more. Many popular accounts give incorrect information about this; here is a recent authoritative discussion. Looie496 (talk) 20:32, 7 March 2010 (UTC)[reply]
(Original research/anecdote) Diabetics have reported greater blood sugar drop, a measure of metabolic rate, while they are asleep and having intensely visually detailed dreams with intricate plotting, compared to sleep without that extra brain work. Of course low blood sugar might have conversely caused the vivid dream images. Edison (talk) 23:24, 7 March 2010 (UTC)[reply]

Single source for printed label nutrition data and all or most UPC/PLC codes

Is there an existing single source for food label nutrition data and UPC/PLC code besides this start-up site or better still a site where a cross reference to the codes used for the extensive USDA food nutrition database at http://www.nutrition.gov/ (where the site is down right now) and the UPC/PLC codes can be found? 71.100.11.118 (talk) 19:26, 7 March 2010 (UTC)[reply]

In-text citation and reference for textbooks and websites in scientific journals

I searched for it but couldn't find instructions on the format for in-text and reference citations for textbooks and websites. I looked at different research papers to follow how they did it but it's hard to find one that cited textbooks or websites. Most papers only cite other research papers. —Preceding unsigned comment added by 70.68.120.162 (talk) 19:33, 7 March 2010 (UTC)[reply]

Here's one for citing websites: [10]. Franamax (talk) 19:48, 7 March 2010 (UTC)[reply]
And this should cover textbooks just fine, looking at the examples, several of them could be textbooks. Franamax (talk) 19:51, 7 March 2010 (UTC)[reply]
Generally, biological journals use either the (Authorname, Year) format for inline citations or a number. There can be some variation from journal to journal. I suggest reading the specific journal's "Instructions to Authors" or similarly named page, usually linked on the journal's home page (e.g. Here are the ones from Cell and Developement both found under "For Authors" tabs on the main page). It will tell authors which style to use and may have Endnote stylesheets available for download (which can also be used by free reference managers, like Zotero - you need to use v. 2.0 to import Endnote styles.). If you are going to be writing articles for publication I strongly recommend using a reference manager. Changing inline citation formatting is a 1 or 2 click operation! -- Flyguy649 talk 20:05, 7 March 2010 (UTC)[reply]
You could sign up for a free trial of RefWorks, import the book/site references you want, and see the product (you select the journal format). 30 days should be long enough to get a quick project done, or form a clear idea. Alternatively, have a look at our article on reference management software and try one of the open source products. Doing this by hand is so 20th century! -- Scray (talk) 20:39, 7 March 2010 (UTC)[reply]

Cyborgs

How close is science to having cyborgs similar to those in films and tv shows; ie a human whos organs are both organic and machine? —Preceding unsigned comment added by Heartofsand (talkcontribs) 21:23, 7 March 2010 (UTC)[reply]

Very far away indeed from what you are probably imagining. But you have to define your terms. There are people walking around with artificial hip joints who are (technically) part man, part machine today. People have had false teeth for hundreds of years. Wooden "peg" legs go back even further. What percentage of man has to become machine before you attach the label "Cyborg"? SteveBaker (talk) 21:50, 7 March 2010 (UTC)[reply]
Well, they did say "organs", by which I assume they mean to exclude bones and teeth. An artificial heart should pass that test, and we've had those, but they never really seemed to catch on. A pacemaker might qualify, and perhaps a cochlear implant and an insulin pump. There's also a type of implant for the blind. See artificial organ for more info. StuRat (talk) 22:04, 7 March 2010 (UTC)[reply]
Bones and teeth are collections of different tissues serving a common purpose, so they are organs. The first sentence of bone seems to agree with me. --Tango (talk) 01:41, 8 March 2010 (UTC)[reply]
Yes, but that's clearly not the definition the OP is using, and I don't think anyone would consider somebody with false teeth to be a cyborg. StuRat (talk) 09:01, 8 March 2010 (UTC)[reply]
Exactly. So we can say that there are already plenty of "Cyborgs" in our society today - and have been for a long time. The problem is that this clearly isn't what Heartofsand is asking. "Similar to films and TV shows" is clearly something dramatically more than we're talking. So let's try to pick a couple of TV/Movie "Cyborgs". I guess at one extreme, we have something like "The Terminator" - which is pretty much all machine - but with organic skin. At the other, perhaps something like "The Million Dollar Man" - who is a regular human with a bunch of machine parts added. Clearly, if we can replace one hip joint, we could replace both of them - and probably a bunch of other joints too. But we can't replace all of the bones because bones are where things like blood cells are made and you'd die pretty quickly if we replaced them all. But there are certainly artificial hearts and all sorts of things like artificial limbs of varying kinds. But making someone who can run at 80mph is really out of the realms of possibility. Our implant technologies are almost all inferior to the original body parts. I can't think of anything we implant that doesn't need to be replaced after 20 to 30 years.
The Terminator is a different problem. We might be able to grow skin and fat and figure out how to wrap them around a machine - but we can't build fully active, intelligent robots - we don't have the motor and battery technology - we don't have the AI capability. So forget the 'cyborg' angle - we can't build intelligent robots - so the whole wrapping them with skin thing is irrelevent.
SteveBaker (talk) 04:33, 8 March 2010 (UTC)[reply]
The main problem with building a cyborg is that you cannot easily mate the human nervous system with a machine "body"; and the human brain -- or at least the human mind, whatever part of the brain it is confined to -- is the key to being a proper cyborg. Now, cyborgs in sci-fi come in several flavors. "Terminator" - style cyborgs are not technically cyborgs, as they have no human parts to begin with; they are androids. So are the replicants in the "Blade Runner" and the type A7Mx in "Yokohama Kaidashi Kikō". Those are many tens of years away; maybe hundreds. "Ghost in the Shell" - style cyborgs are cyborgs with more and more human parts replaced by cybernetic devices, dealing directly with the Ship of Theseus issue of at what point a human is no longer human; YMMV. But this is also tens of years in the future. "Hacking the mind" may be further away yet, as the human brain does not internally use a deterministic code of any sort the way modern computers do (although motor commands to the muscles and sensory signals from the skin are pretty deterministic, or at least stereotypical). Taking a human head and spine (or head and torso) and mating them to a machine body, Darth Vader style, may be not that far away. Fairly advanced artificial limbs are available at present. Artificial hearts, lungs, and kidneys are also available, but not yet the easily portable ones. Artificial liver - not as far as I know, but maybe in a decade or two. Human liver can regenerate anyway, if the damage is not too severe. We'll see how fast this is going. The progress seems to have slowed down quite a bit now that we don't have the Cold War anymore, but hopefully it will pick up the pace again; maybe when cyborg building becomes commercially appealing. It may also be of interest to you that we actually have List of fictional cyborgs and articles on Cyborgs in general and Cyborgs in fiction in particular. --Dr Dima (talk) 04:24, 8 March 2010 (UTC)[reply]

F-ATPase

How is that pronounced? "ai, ti, pi, ase"? "ai, ti, pase"? "Adenosine Triphosphatase"? Thanks, 21:25, 7 March 2010 (UTC) —Preceding unsigned comment added by 89.139.173.179 (talk)

It's pronounced /ɑː(ɹ)s/
Whilst true, I suspect the OP wants a simpler answer. I've always said "ai-ti-pi, ase" and "adenosine triphosphatase" Regards, --—Cyclonenim | Chat  22:00, 7 March 2010 (UTC)[reply]
Concur; it's always referred to as F--A-T-P-ase (the letters + -ase suffix). That's roughly Template:Pron-en. -- Flyguy649 talk 22:10, 7 March 2010 (UTC)[reply]

measurement for light

What is the measurement for light? I've heard people using candlepower, is that an official term? What constitutes one unit of candlepower? —Preceding unsigned comment added by Venture4356 (talkcontribs) 22:45, 7 March 2010 (UTC)[reply]

If you mean brightness of light, that's called luminosity. Note that there's a difference between the total amount of light given off by an object and the amount which falls on any particular area. The unit candlepower is now largely replaced by the candela. There's also albedo, which is a measure of the reflectivity of a surface, and the frequency/wavelength (color) of light is often measured in THz or nM. StuRat (talk) 22:53, 7 March 2010 (UTC)[reply]
There are many confusingly similar ways to specify the brightness or light output of a light source. While in the 19th century they referred to "candlepower" now they specify a light bulb output in "lumens" A candlepower was the brightness of a certain kind of "standard" candle, burning at a certain rate, but in practice the observed brightness varied widely between standard candles and in the same candle over time. One candlepower = 12.57 or 4 pi lumen per [11]. But the conversion is misleading: it assumes uniform output in all directions. Candlepower is the light emitted in a particular direction, so a bulb in a reflector would have more candlepower in the direction where the reflector concentrates it. Lumens integrate the light emitted in all directions,per [12]. A light source with a lens or reflector might have lots of candlepower in a very small angle, but not all that many lumens of total light output. The SI unit of luminous intensity is the Candela, selected to be about equal to the old "candle." Edison (talk) 01:51, 8 March 2010 (UTC)[reply]

Disulfide bond connecting two adjacent helices not connected by a loop/turn

How common are disulfide bonds that connect two adjacent helices which are not connected by a loop/turn? —Preceding unsigned comment added by 70.68.120.162 (talk) 23:18, 7 March 2010 (UTC)[reply]

Not sure regarding the loops and turns bit, but disulphide bridges are pretty common. Regards, --—Cyclonenim | Chat  09:31, 8 March 2010 (UTC)[reply]

Talking parrots

Why is it that some species of parrot/cockatoo are better at mimicing human speech than others? Is there an anatomical or inteliigence reason? Something like an African Grey Parrot or an Amazon parrot is a very clear talker and sounds a bit like a human child speaking, wheras other species can talk - but their voices are basically just birdy squawks, tweets and growls shaped into words and syllables, or approximations of words and syllables.

Also - why is it that the Mynah is such a good mimic? It's even better than parrots, from what I've heard, despite not being a very intelligent creature. --95.148.104.245 (talk) 23:51, 7 March 2010 (UTC)[reply]

It's not just intelligence that matters. An animal that can talk does need to be able to remember what it hears, but not all animals that are intelligent can talk (take Chimpanzees for example. The biggest factor of whether or not an animal can talk are the organs that make the noise and how they are arranged (see Speech production), so a human can never bark and a dog can never talk. --The High Fin Sperm Whale 23:58, 7 March 2010 (UTC)[reply]
I should point out that the term "talk" is being used here to mean "mimic human speech". No animal actually talks, meaning using human speech fully (although they do communicate). StuRat (talk) 00:01, 8 March 2010 (UTC)[reply]
OK - StuRat - you're going to have to stop guessing and saying that things aren't true without some evidence. We have articles Alex (parrot) and N'kisi that certainly show at least a strong possiblity that African Grey parrots can learn to use language "for real". "Fully" is a tough standard to meet - quite a few humans that I've met don't use the language "fully". But I've seen plenty of documentaries about Alex and he had me convinced. Several of the great apes have been trained to use sign language - complete with grammar, inventive use of signed speech, etc. It's OK to be wrong - but it's not OK to be persistently, annoyingly and definitely wrong. So how about you slow down your answers and actually do some checking and referencing before you jump to a negative conclusion. SteveBaker (talk) 04:13, 8 March 2010 (UTC)[reply]
It's a bit rude to just accuse me of guessing. I've read several scientific journal articles and seen several documentaries of animal intelligence and specifically speech, and my answer was based on their conclusions. The sign language is an impressive form of communication (particularly in the case of Koko), but that's not speech, which our article defines as "vocalized human communication". They go on to define it as using a large number of words, typically over 10000. No animal uses anywhere near that many human words. Read our article and then tell me how, in any way, Alex or any other animal meets those definitions. Also, the question was about animal species capable of speech, which I take it means that most of the species is so capable, not just a rare exception. StuRat (talk) 08:33, 8 March 2010 (UTC)[reply]
Simple English uses a vocabulary of just 1,500 words - Basic English just 850. You don't need anything like 10,000 words to communicate effectively. Koko knew 1,000 signs and 2,000 words - so she certainly managed "Basic English". Alex's vocabulary was small (150 words) - but could put them together to make composite words for things he didn't know the name of. Alex didn't know the name for an "Almond" - but, all by himself, he came up with "Cork nut" and used it consistently from that point onwards. Arguably, "Corknut" is another word - and one major problem with estimating vocabulary size is the degree to which prefixes, suffixes and composite words inflate the total. SteveBaker (talk) 04:48, 9 March 2010 (UTC)[reply]
According to our article, Alex seemed to be limited to describing objects and answering simple questions about them. He seems to have had a very limited knowledge of verbs (the only one mentioned in the article that I can see is "go", used to say where he wanted to go). I think without a significant number of verbs you can't be said to be using language fully. --Tango (talk) 06:00, 9 March 2010 (UTC)[reply]
And the apes that are taught sign language tend to "babble" quite a bit, meaning that there's a high degree of interpretation from their handlers as to what they're actually saying that's meaningful. I'm skeptical of how well they actually understand what they're "saying". I hadn't heard of Alex, who does appear to have learned a few (~150) words and actually communicate with them. See also Animal Language; it's not a cut and dried field. Buddy431 (talk) 21:53, 8 March 2010 (UTC)[reply]
AFAICR Alex learned to respond appropriately to questions and instructions, in a way that indicated that he had some grasp of the meanings of the words; but I don't think he ever initiated verbal communication, did he? --ColinFine (talk) 23:56, 8 March 2010 (UTC)[reply]
Sure he did. Read through the references in Alex (parrot) - you'll see plenty of occasions when Alex asked people to get him food and toys and such. SteveBaker (talk) 04:58, 9 March 2010 (UTC)[reply]

Chicken butt

Hello all. When I was little, my grandfather used to warn us that the qiao qiao (the back part of a chicken that sticks up) of a chicken would make us sick if we ate it, reasoning that the chicken stores its toxins there. I came across such a "qiao qiao" and that made me think of this again. I was wondering: Is there any scientific merit to this assertion? 68.76.146.111 (talk) 23:53, 7 March 2010 (UTC)[reply]

No. StuRat (talk) 23:58, 7 March 2010 (UTC)[reply]
Now, no doubt others will complain that I need to add detail, so here goes:
1) I doubt if we have an article on that part of the chicken. I don't know it's name, so don't know how to find it, if we do. And, even if we do have such an article, I don't believe it will address the issue of toxicity in any way.
2) Since chicken is a common food item, we would know if any part of it is toxic.
3) The tail of a chicken is there to hold tail feathers, not toxins. StuRat (talk) 00:10, 8 March 2010 (UTC)[reply]
Of course there is an article, see Pygostyle, although it does not have the food properties, it is very fatty. Graeme Bartlett (talk) 00:39, 8 March 2010 (UTC)[reply]
My mother, who introduced me to the terms Parson's nose and Pope's nose, said that it was bitter and unpleasant. This book and our article suggest many people disagree with her. 86.177.121.239 (talk) 00:58, 8 March 2010 (UTC)[reply]
I added a redirect to fix the redlink on Parson's Nose. StuRat (talk) 01:10, 8 March 2010 (UTC)[reply]

Perhaps she was getting confused with the cloaca, which is where the eggs, feaces and urine leave the bird. Probably not the cleanest part of it. I think this is normally removed with the rest of the digestive systems when the bird is butchered. 62.56.68.180 (talk) 01:16, 8 March 2010 (UTC)[reply]

The issue of toxins deserves address, as in ordinary language it usually represents an imaginary concept from the century before last. In its strict sense a toxin is a biologically produced substance that functions as a poison to other organisms. One of the mechanisms of disease theories of 19th century medicine was that a mechanism of illness could be the failure to eliminate endogenously produced toxins, leading to "autointoxication". Other 19th century mechanisms of disease included subluxation, miasma, masturbation, and evil spirits. There are two major organs that eliminate dangerous compounds produced endogenously or ingested (the liver and kidneys), and hence the only real diseases produced by autointoxication are the effects of liver or kidney failure. The other major diseases of toxins result from exogenous toxins such as botulism or mycotoxins. We can acquire bacteria (not toxins) from food that has been kept in unsafe conditions. We can acquire bacteria and viruses from feces of other humans, and occasionally from the fecal bacteria of other animals like salmonella from chicken or E. coli from beef. Uneducated people conflate cultural categories of "clean" and "unclean" food with food-borne illnesses. Scientists moved on from the old autointoxication theory in the early 20th century, but quacks and con men continue to find it very profitable to sell people ways to eliminate these imaginary toxins. alteripse (talk) 01:44, 8 March 2010 (UTC)[reply]
I think that part of a cooked chicken used to be known as the parson's nose in Britain. 78.146.208.26 (talk) 20:52, 8 March 2010 (UTC)[reply]
I assume he's referring to the gland just over the tail and not the tail itself. In the US, this gland is routinely cut out during processing because of its offensive odor/taste. Some cultures leave it in, so we can infer it's not too toxic. I've cut it out many times in a cargo cult fashion, as it looks unappetizing (like an overgrown pimple). --Sean 19:14, 9 March 2010 (UTC)[reply]

March 8

Scientific journals

I went investigating today to see about subscribing to a meteorological journal. Upon seeing the prices, I quickly reconsidered the wisdom of that idea. Am I interpreting it correct that these prices are up to around $110 an issue?! If so, what on earth causes such massive prices for an academic journal of all things?! Ks0stm (TCG) 01:21, 8 March 2010 (UTC)[reply]

The usual problem is very low circulation, meaning they have to charge each subscriber a lot. A lack of advertising can also be a factor. StuRat (talk) 01:40, 8 March 2010 (UTC)[reply]
Price is determined more by what people are willing to pay than by cost. Cost sets a lower limit on price, but that is all. People (well, institutions, more often) are willing to pay that much for the journal, so the journal charges that much. Presumably, the publishers have determined that charging less wouldn't result in sufficiently more sales to get a higher total revenue. --Tango (talk) 01:44, 8 March 2010 (UTC)[reply]
Except not really, because most academic journals are actually sold to libraries and institutions in subscription bundles with various other journals from the same publisher. The cost of the bundle is generally much less than the sum of the face value prices on the individual components, so the institution generally ends up paying much less per work than the apparent price anyway. Dragons flight (talk) 03:47, 8 March 2010 (UTC)[reply]

The principal purchasers are institutional libraries, as they have priced themselves out of the individual market. alteripse (talk) 01:59, 8 March 2010 (UTC)[reply]

If you are interested in reading papers from a meteorological journal without having to subscribe, you should look into universities in your area. I see you're from Kansas. I don't exactly know how things work in Kansas, but in Nebraska, since UNL is a state school, the library offers library cards to Nebraska residents [13], and also has a high school students program [14]. Perhaps the universities in Kansas have similar policies. —Bkell (talk) 04:59, 8 March 2010 (UTC)[reply]
I'll check my local universities, but I don't know if they will have them since neither of them have a meteorology program. I might have to discuss this with a student at one of them who started a weather club. Thanks. Ks0stm (TCG) 14:46, 8 March 2010 (UTC)[reply]
Even if your local university libraries don't have this particular journal, they can probably get it through interlibrary loan. Anyway, it's worth investigating. —Bkell (talk) 19:46, 8 March 2010 (UTC)[reply]

Traditionally, academic journals provided important services - organization of peer-review, proof-reading, typesetting, printing, and dissemination. Due to the low circulation, prices had to be high - and, in fact, library subscriptions are often much more expensive than individual subscriptions. Within the last 20 years or so, this role of publishers has gone down. Authors are now expected to submit camera-ready manuscripts in many fields, and dissemination is increasingly electronic. Peer review is cheaper and easier too, thanks to electronic communication. As a result, scientific publishing is undergoing massive changes, and its hard to predict what will happen over the next 10-20 years. --Stephan Schulz (talk) 10:37, 8 March 2010 (UTC)[reply]

I'm not sure which journals you're looking at, but the link you provide above seems to say that the online addition of the Bulletin of the American Meteorological Society (BAMS) is free to non-AMS members. It looks like they have open access to all articles after 1970.[15] I'd suggest starting there. :-) -Atmoz (talk) 00:29, 9 March 2010 (UTC)[reply]

Atmospheric Chemistry and Physics is also an open access journal. -Atmoz (talk) 00:34, 9 March 2010 (UTC)[reply]

Colour of glass

How is colour added to glass? I especially want to know for 'blue'. -- Extra999 (talk) 07:52, 8 March 2010 (UTC)[reply]

Colour is added to glass by adding certain metal salts to the molten glass (see Glass colouring and colour marking). Blue is made with Cobalt salts, I believe. --The High Fin Sperm Whale 02:51, 8 March 2010 (UTC)[reply]
Substances Used in the Making of Coloured Glass may be your friend. Cobalt seems to be the key to blue glass. --Tagishsimon (talk) 02:48, 8 March 2010 (UTC)[reply]

Thanks. -- Extra999 (talk) 18:00, 8 March 2010 (UTC)[reply]

The Lions

Hello, I live in Langley, BC, and I was wondering if you can see The Lions from here. I did see one mountain that looked a bit like them, but it was tiny and off in the distance. So can you see them from Langley? --The High Fin Sperm Whale 02:40, 8 March 2010 (UTC)[reply]

Assuming clear air. the old-fashioned pre-metric rule for how far away the horizon is when you are at h feet above sea level is: distance (miles) = sqrt(h x 7/4). So if you stood on top of the taller of the two peaks (5,400 feet) - then the horizon would be about 97 miles away. So you should be able to see the peak from sea-level locations within 97 miles. Google maps says that the center of Langley is about 45 miles away. So the curvature of the earth certainly isn't an obstacle. Whether there are intervening obstacles...I'm not sure. Whether the atmosphere is clear enough...I don't know. But if I had to guess, I'd say you'd be able to see the peaks on a clear day. SteveBaker (talk) 03:52, 8 March 2010 (UTC)[reply]
That's a neat formula. A back-of-the-envelope calculation (starting with the formula Steve provided) yields a metric approximation: distance (km) = sqrt(h[meters] x 15). This metric version is gives answers that are 0.5% greater (about a km for most peaks) than the formula SB gave above. In this specific case, 5400 ft is about 1646 m, yielding a visibility estimate of 157 km, or 97.6 miles (pretty close). The error (relative to the original formula) drops to 1.2% if you use 14.9 instead of 15 in the square root - but I am not sure that's worth it -- Scray (talk) 12:25, 8 March 2010 (UTC)[reply]
If I could see them, would they be quite small? --The High Fin Sperm Whale 21:32, 8 March 2010 (UTC)[reply]
I just looked at this using Google Earth -- I don't think you'll be able to see The Lions from Langley; it looks like Grouse Mountain is directly in between, and high enough that you won't be able to see over it. Looie496 (talk) 23:49, 8 March 2010 (UTC)[reply]
Google Maps shows the peak of Grouse is about 10 miles from the Lions and at 1200' it's only a quarter the height of the Lions - the geometry doesn't seem enough for it to block the view. Using the magic equation again says that you can see Grouse Mountain from 46 miles away and The Lions from 96 miles away. So at 46 miles away from Grouse (which would be 56 miles from The Lions), Grouse would be below the horizon - and the Lions would still be in your line of sight. Langley is only about 30-ish miles from Grouse Mountain - I don't see how it could possibly block the further peaks. I'm skeptical about your Google Earth results - the numbers just don't work. SteveBaker (talk) 04:24, 9 March 2010 (UTC)[reply]
Grouse Mountain is actually 1200+ meters high, not 1200 feet. It might still fail to block the view, but it's a close call. (Our article seems to have some confusion on the numbers, but it definitely should be meters.) Looie496 (talk) 20:54, 9 March 2010 (UTC)[reply]

bombardment of hydrogen isotopes with neutrons

What happens when a plasma made up of all the different isotopes of hydrogen is bombarded with neutrons? 71.100.11.118 (talk) 03:32, 8 March 2010 (UTC)[reply]

Then it would become heavier isotopes, first deuterium then radioactive tritium. --The High Fin Sperm Whale 03:34, 8 March 2010 (UTC)[reply]
...and thereafter (in the presence of all three isotopes)? 71.100.11.118 (talk) 03:51, 8 March 2010 (UTC)[reply]
I suppose that if tritium was bombarded with neutrons, a neutron would decay into a proton and electron, forming helium. --The High Fin Sperm Whale 03:57, 8 March 2010 (UTC)[reply]
Is there any evidence for this say at higher densities and lower temperatures as close to absolute zero as possible and under maximum pressure or in high temperature, high pressure plasma? 71.100.11.118 (talk) 04:14, 8 March 2010 (UTC)[reply]
It is interesting that Hydrogen-4 and Hydrogen-5 and Hydrogen-6 will not be produced, since they actually decay back to tritium in sub attosecond timescale by giving off neutrons. (actually in a zeptosecond which apparently is well below the measurable time scale). Graeme Bartlett (talk) 06:50, 8 March 2010 (UTC)[reply]

Can persistent high blood pressure be someone's fault?

(1) Is it possible for someone (annoying and clingy and argumentative and stubborn, but never really cruel - just basically someone immature who doesn't play well with others) to drive someone's blood pressure up so high that they would be in medical danger and possibly even suffer long-term damage after being stabilized? (2) Or is it more likely that the hypertensive crisis would cause the person with high blood pressure to perceive the immature individual as more of a problem than s/he actually is? (3) And if the latter is the case, is it likely that the animosity the person with high blood pressure felt would pass when the blood pressure returned to a normal range? —Preceding unsigned comment added by 71.108.171.138 (talk) 03:50, 8 March 2010 (UTC)[reply]

There is a tested and proven correlation between stress and hypertension. The statement "stress causes hypertension" is not the same as "stress is correlated with hypertension." Therefore, you are making two jumps: annoying person causes stress and stress causes hypertension. As such, it is unlikely to find many references that justify that double-claim. -- kainaw 03:54, 8 March 2010 (UTC)[reply]

(4) Does that mean it's unlikely that one person's faults could singlehandedly drive another person into hypertensive crisis? (5) And more importantly, could high blood pressure cause a person to think or behave irrationally, particularly to prompt feelings of anger or paranoia? —Preceding unsigned comment added by 71.108.171.138 (talk) 03:58, 8 March 2010 (UTC)[reply]

It's hard to prove a negative, but all evidence suggests the answers are 1 No, 2 No, 3 No, 4 Yes, and 5 No respectively. alteripse (talk) 04:21, 8 March 2010 (UTC)[reply]

Hmmmmm . . . if I follow your answers correctly, you're saying that high blood pressure can prompt or exaggerate angry feelings (which is all I meant by "irrational thinking," by the way . . . such as blaming someone for something that really isn't his or her fault, and no clear-headed person would truly think it was . . . but ira est furor brevis)?

No. Ira sit furor brevis, sed non hypertensio. I went back and numbered your questions so you can match up the answers correctly. Hypertension is dangerous because it is asymptomatic (silent). The fluctuation with emotional upset results from adrenaline, and adrenaline elevations from emotions may be perceptible because of other adrenaline effects, but tend to be too brief to represent a long term health risk. There is a less well understood epidemiologic relationship between chronic life stress and hypertension but the mediating factors are not known and may include other things associated with poverty rather more than simply prolonged emotional stress. alteripse (talk) 11:42, 8 March 2010 (UTC)[reply]

Thanks. And Alteripse, you rock. Okay, let me try to sign this one properly, because I'm not one of those idiots out to destroy Wikipedia. 71.108.171.138 (talk) 17:21, 9 March 2010 (UTC)[reply]

menstrual period information

menstrual period information —Preceding unsigned comment added by Lorir7 (talkcontribs) 03:55, 8 March 2010 (UTC)[reply]

Asking or offering? You know, I'm sure Wikipedia has an article on this. At least one. —Preceding unsigned comment added by 71.108.171.138 (talk) 03:59, 8 March 2010 (UTC)[reply]

I suggest you start at Menstruation, and go from there. Dolphin51 (talk) 04:00, 8 March 2010 (UTC)[reply]
See below a question "menstrual period correlation with Metonic and other lunar cycles". Cuddlyable3 (talk) 11:40, 8 March 2010 (UTC)[reply]

Orbit

The moon orbits around the Earth and the Earth orbits around the Sun and the Sun orbits around the center of the Milky Way. What does the Milky Way orbit around? 71.100.11.118 (talk) 04:32, 8 March 2010 (UTC)[reply]

I don't believe the Milky Way follows a periodic orbit. They usually only exist when there are just two bodies of significant mass to consider (eg. when considering the Earth and the Sun, you can ignore the Moon and other planets as being too small or too far away to do much, or when considering the Earth and Moon, the Sun and everything else is too far away to worry about). For the Milky Way, that doesn't really work. The Local Group contains a large number of galaxies, several of which are too large or close to ignore. Over a short period of time, the Milky Way will act as though it is orbiting the centre of mass of the Local Group, which is somewhere roughly in the middle of the Milky Way and the Andromeda Galaxy (which, between them, make up most of the mass of the Group). Over longer time periods, it is much more complicated. --Tango (talk) 04:49, 8 March 2010 (UTC)[reply]
...humm. Thinking in terms of center of mass and the fact the that whole Universe is suppose to be related or connected by gravity what center of mass would the center of mass of the local group be attacted to? 71.100.11.118 (talk) 08:46, 8 March 2010 (UTC)[reply]
There is no center of mass for whole universe. See Cosmological Principle. manya (talk) 09:16, 8 March 2010 (UTC)[reply]
But nearby us, Great Attractor plays some role. - manya (talk) 09:24, 8 March 2010 (UTC)[reply]
The centre of mass of the Virgo Supercluster, I guess. I don't know where that is, but I would guess it is somewhere fairly near the Virgo Cluster, which is near the geometric centre of the Virgo Supercluster. Beyond the scale of superclusters, objects aren't usually gravitationally bound to each other, so it doesn't really make sense to talk about what they are attracted to. The Hubble Flow is far more important on those scales. --Tango (talk) 17:10, 8 March 2010 (UTC)[reply]
The ISS orbits the earth every 90 minutes or so. The moon orbits the earth once a month. The earth orbits the sun once a year. The sun orbits the galactic core every 250 million years. If our galaxy did orbit something, the orbital period would be so long that it could easily be longer than the age of the galaxy. Put another way, the objects involved would be changing so much over the period of any possible orbit that this couldn't be anything really stable. SteveBaker (talk) 04:10, 9 March 2010 (UTC)[reply]

Why is hunger so mild?

Is there a biological reason for why starvation is not intensely painful? It seems to be one of the most painless ways to die naturally, excluding instantaneous deaths, and is nothing like having cancer or dehydrating. --99.237.234.104 (talk) 04:48, 8 March 2010 (UTC)[reply]

Well, I've never had cancer or dehydrated, so I can't compare them . . . and I've never starved either for that matter. But I've skipped enough meals to wonder what on earth makes you think that starvation isn't intensely painful. That clawing sensation of your empty stomach cramping up begging for food? Ouch. Really, I think I'd prefer dehydration . . . I've never heard what it feels like, beyond that you'd be really really thirsty, which I imagine would be terribly unpleasant but much less painful than starvation. To try to take your question seriously, though, starvation is less acute a condition than dehydration. When you need water, you need it within a day or two at most to keep functioning. In the wild, animals often have to go for many days, longer at times, without food, and they need to be able to focus on other things and function in spite of the discomfort in order to keep up the search for food and other necessities and survive. In time of dehydration water is almost always going to be the first priority, but in starving times there are likely going to be many intervening priorities, as it may be quite some time before food can be found. So it wouldn't do to have hunger be as keen a drive. But in the final stages, of course it's intensely painful. And it's not terribly pleasant up to that point either . . . especially since it's drawn out over several weeks.


One major symptom of dehydration is extreme headaches - not pleasant at all. It is far quicker than starvation, though (about 3 days, rather than 3 months). --Tango (talk) 04:56, 8 March 2010 (UTC)[reply]
Dying of starvation is not at all pleasant and takes a very long time. The reason hunger is usually a very mild sensation is because you rarely get very hungry. Most people eat several times a day but they could go months without food, so clearly they are eating when hunger is still at a very early stage. See Starvation#Signs and symptoms for a description of some of the things that happen to your body when you don't eat. One part that jumps out at me is "All movements become painful due to muscle atrophy". --Tango (talk) 04:56, 8 March 2010 (UTC)[reply]

Ah, headaches. True enough. I've wondered often enough when I've downed a couple of Tylenol with a glass of water if it wasn't the water that relieved the headache at least as much as the medicine.

I've heard that for about the first three days of not having anything to eat, it is painful because your stomach starts to shrink and cramp, thus causing pain. Then, for the next 40 days, it stops hurting and your body starts to digest fat reserves. After that, it starts to hurt again, because your body starts digesting the muscles. Your eyes are one of the first to go. --The High Fin Sperm Whale 05:13, 8 March 2010 (UTC)[reply]
I have fasted for seven days at a stretch, water only, many times. It's not painful. Climbing stairs gets more tiring after 5 days or so. The hunger seems more psychologically strong in the first 3 days, then it gets weaker. Your sense of smell gets quite acute - it becomes disgusting to be near a fast food place or anywhere where there has been poison used recently. At 7 days, there are no problems with sight. —Preceding unsigned comment added by Marklawrence17 (talkcontribs) 08:06, 8 March 2010 (UTC)[reply]

Eyes aren't muscles. And look at that link . . . there's a lot more to the pain of starvation than the initial stomach cramps. —Preceding unsigned comment added by 71.108.171.138 (talk) 05:16, 8 March 2010 (UTC)[reply]

The eyes do have & need muscles however and if these go your vision will almost definitely go, see Extraocular muscles, Muscles of orbit, Ciliary muscle, Iris dilator muscle & Iris sphincter muscle for example. Whether there are really the first to go, I don't know but it's probably your vision will be strongly negatively affected if they start to atrophy whereas movement might be incredibly painful but possible even after your limb muscles have partially atrophied. Since headaches came up, I'm surprised no one mentioned hunger or fasting can cause headaches as well I presume due to mild hypoglycemia. However when it comes to real starvation, I don't know if this will persist since it seems likely when your body adapts it shouldn't be a problem. Nil Einne (talk) 06:43, 8 March 2010 (UTC)[reply]
Hypoglycemia is a very short term problem - your body will switch to reserves within hours, at most. --Tango (talk) 17:12, 8 March 2010 (UTC)[reply]
Actually I'm starting to wonder if it's so simple. [16] for example reports headaches as a problem in a semi-starvation diet. Similar reports with anorexia nervosa [17] and other starvation diets. I can't find any great source (lots of naturopath and similar websites and most stuff are discussing 'starvation' as in a few hours or an inadequate/semi-starvation diet rather then full starvation per se) but my impression is that it's likely to be a persistent problem for real starvation. Definitely from personal experience as well as from other people I know, I can say fasting for 12-24 hours or so can causeheadaches although it likely varies from person to person. Of course in the long term hypoglycemia may not be the primary cause but other things like inadequate electrolytes, dehydration (which can be a problem during starvation) and lacking essential vitamins and minerals. Of course, if you starving yourself by choice rather then by necessity, e.g. as part of a hunger strike you may consume drinks with sufficient electrolytes etc which would I presume change things. But for example, headaches are listed as a symptom of rabbit starvation which isn't starvation per se but it would seem likely whatever causes headaches there would happen in real starvation. Of course headaches may not be the worse symptom by a long shot over the long term but over the short term, I'm not convinced headaches due to dehydration are necessarily worse then due to fasting or starvation. In the 1+ day window where you're starting to become at risk at death and in the same time frame for dehydration things would be different I expect. Of course it almost definitely varies from person to person and as I hinted at above those adapted to it would probably fare better then the average person in the developed world Nil Einne (talk) 18:54, 8 March 2010 (UTC)[reply]
There are plenty of people who have voluntarily starved to death (See Hunger strike for some examples). To my mind, that sets limits on just how bad the symptoms could be. SteveBaker (talk) 04:04, 9 March 2010 (UTC)[reply]
Yes, it means that, by definition, the symptoms can't be unendurable. Sufficiently motivated people can endure a hell of a lot, though. --Tango (talk) 04:09, 9 March 2010 (UTC)[reply]
There are also people who burned themselves to death as protest. This is a very high upper limit. — DanielLC 21:45, 9 March 2010 (UTC)[reply]
Isn't there a destructive process of "[Ketosis]]" which sets in after a day or two of starvation, so that the breath smells a bit like acetone or fruit, due to the body metabolizing fat rather than carbohydrates? Could the headache come from the brain having to use ketones as fuel? Edison (talk) 17:46, 9 March 2010 (UTC)[reply]
Taking another side to the question, one thing that strikes me is that painful hunger is not conducive to the body getting more food. If it complains too much, it hinders you ability to help it. I am, however, not an expert on the subject, and am clearly applying personification to evolution. - Jarry1250 [Humorous? Discuss.] 17:52, 9 March 2010 (UTC)[reply]
Yes, I was thinking that if hunger was as unendurable as not being able to breathe, people would probably end up killing themselves trying to eat rocks or ashes or sticks. Googlemeister (talk) 21:42, 9 March 2010 (UTC)[reply]

An interesting website

I'm no sure if this is the right desk to ask this on, but I'm having trouble finding a website that i once visited. It was someone's personal website and he/she discussed topics including what a Neanderthal language might have been like, including a translation of genesis into his hypothetical neanderthal, and a list of scientific inaccuracies in Star Trek. There was also a fictional account of how civilization in the future was destroyed by people wanting to learn new words constantly to fit in with the latest dialect, so they overused neuroplasticisers and gave themselves brain damage. Does anyone know of anything like this? 74.14.109.146 (talk) 06:49, 8 March 2010 (UTC)[reply]

perhaps http://adamjamesnall.blogspot.com/ ? —Preceding unsigned comment added by Graeme Bartlett (talkcontribs) 07:07, 8 March 2010 (UTC)[reply]
You're thinking of http://www.xibalba.demon.co.uk/jbr/index.html. Coincidentally I just discovered this page two days ago, at the end of a chain of link-following starting with a random Wikipedia image which turned out to be one of the Sammarinese euro coins, which led me to San Marino, which led to Akademio Internacia de la Sciencoj San Marino, which got me curious about Esperanto, so I did a Google search for learn esperanto, and the fourth result was Learn Not to Speak Esperanto. (Now why in the world can I remember all of that?) —Bkell (talk) 07:21, 8 March 2010 (UTC)[reply]
Ah, yes, it's that one. As I believe is customary, I hereby grant you one (1) internet. 74.14.109.146 (talk) 08:12, 8 March 2010 (UTC)[reply]
Yay! I have an Internet! Bkell (talk) 08:29, 8 March 2010 (UTC)[reply]

menstrual period correlation with Metonic and other lunar cycles

How does the human female menstrual period compare with all of the various lunar cycle and with which one does it correlate best? 71.100.11.118 (talk) 08:43, 8 March 2010 (UTC)[reply]

THe Wikipedia article Menstrual cycle notes that the menstrual cycle varies from woman to woman and the average is 28 days. The moon takes 27.32 days to revolve around the Earth; the period between full moons is 29.53 days. Opinions differ on whether moonlight influences menstruation (see Lunaception) or the similarity of cysles is a coincidence. See also the article on the McClintock effect supposed to synchronise the menstrual cycles of women who live together.Cuddlyable3 (talk) 11:24, 8 March 2010 (UTC)[reply]
If the lunar cycle influenced periods then all women in the world would have their periods on the same day... 195.35.160.133 (talk) 11:49, 9 March 2010 (UTC) Martin.[reply]

scientists

Do all scientists keep notes? What are some famous scientists who never kept any notes and worked in a very haphazard way? —Preceding unsigned comment added by Milticcason (talkcontribs) 09:07, 8 March 2010 (UTC)[reply]

I would think that every scientist would want to keep notes, if they could. Perhaps there were some illiterate scientists (particularly in societies that hadn't yet developed a written language). Also, some might not want to take notes as that might provide proof that they were engaged in an illegal activity. Those studying anatomy, for example, often illegally obtained bodies for dissection. StuRat (talk) 09:21, 8 March 2010 (UTC)[reply]
To your first question, the short answer is "no". Just as there are different learners out there (some learn from lectures, some from reading, some from copious note-taking, etc) there are different researchers - and I've seen the full range of laboratory records, from none to compulsive. As indirect evidence, I cite the plethora of guidelines like this that admonish lab personnel to document what they do - because these would be superfluous if people weren't "haphazard". Certainly, there may be some who keep no records to hide malfeasance, but there are others who just don't keep anything we'd recognize as useful records (except when they publish - but that's not "notes"). -- Scray (talk) 11:55, 8 March 2010 (UTC)[reply]
Not a scientist, but Srinivasa Ramanujan (possibly one of the greatest mathematicians of the last century) kept very few notes for at least some time. See 'Ramanujan's notebooks' on that page. 131.111.248.99 (talk) 13:24, 8 March 2010 (UTC)[reply]
Usually scientists are now taught that good laboratory practice requires the maintenance of a laboratory notebook. While I'm sure there might have been famous scientists who succeeded despite their lack of note keeping (though I'm sorry that I can't think of any examples), this would make replicating experiments (one of the key points of the scientific method) difficult, and would limit the ability of the scientist to review past experiments to draw inferences on what might have caused things to turn out in an unexpected fashion. (It has been said that great science isn't heralded by "Eureka!", but by "Hmm, that's curious".) -- 174.21.235.250 (talk) 16:09, 9 March 2010 (UTC)[reply]
Although I was an engineer, not a scientist, I always found that how ever many notes I made in my lab book, they were never enough and I had to re-perform at least some tests!--79.76.188.14 (talk) 23:12, 9 March 2010 (UTC)[reply]

gravity versus electromagnetic forces

What is the formula that shows the point of balance between the electromagnetic repelling force of orbiting electrons and the pull of gravity between two atoms of hydrogen such that it can be applied to multiple atoms of hydrogen right up to the size of a star? Also, what is the formula for the density of hydrogen atoms due to gravity and the diameter of the whole? 71.100.11.118 (talk) 09:53, 8 March 2010 (UTC)[reply]

The high temperatures in a star mean that most of the hydrogen atoms are ionised - their electrons are stripped away, and the free electrons and protons form a plasma. The main force opposing gravitational collapse in a main sequence star, in which fusion is still active, is not electromagnetic repulsion - it is the thermal pressure arising from the energy released in the fusion reactions, which appears as kinetic energy in electrons and protons plus radiation pressure from photons. When a star has insufficient fuel to continue fusion, other forces such as electron degeneracy pressure become important. See our article on stellar evolution for more details. Gandalf61 (talk) 10:30, 8 March 2010 (UTC)[reply]
Okay, is there a thermal versus gravitational formula and a formula to show the rate of fusion versus the amount of hydrogen (star mass, diameter or size)? 71.100.11.118 (talk) 22:44, 8 March 2010 (UTC)[reply]

Lorentz transformations

If the velocities between two reference frames were such that the relative motion were not parallel to one of the axes, what would the lorentz transformations look like? —Preceding unsigned comment added by 173.179.59.66 (talk) 10:32, 8 March 2010 (UTC)[reply]

It would be a combination of a rotation and a normal Lorentz transformation, see Lorentz group and Poincaré group 157.193.173.205 (talk) 13:17, 8 March 2010 (UTC)[reply]

Read Lorentz Transformation#Matrix form. 174.58.105.234 (talk) 18:38, 8 March 2010 (UTC)[reply]

Okay, so how would you be able to derive this formula? 173.179.59.66 (talk) 19:32, 8 March 2010 (UTC)[reply]

For instance, like this. DVdm (talk) 19:53, 8 March 2010 (UTC)[reply]

about thrusters used in the spacecrafts

hey.. please someone let me know about the working of the thrusters in the vaccum(space) in the absence of any medium which would provide a reaction to the craft's linear motion.. e-mail address deleted —Preceding unsigned comment added by 116.73.242.109 (talk) 11:23, 8 March 2010 (UTC)[reply]

See the article about the Rocket engine which obtains thrust in accordance with Newton's third law that says "For every action force there is an equal, but opposite, reaction force". It needs no external material to form its jet and therefore it can drive a spacecraft. See the article Spacecraft propulsion. Cuddlyable3 (talk) 11:32, 8 March 2010 (UTC)[reply]
To help to visualize this, think of a ship as having forward thrust if it can eject material out the back faster than any material hits the front. In the case of boats and planes, water or air hits it at the front of the propeller or jet, at a low speed, and is then accelerated and leaves the end at a higher speed. In the case of a space ship in a vacuum, nothing (or very little) hits the front end, so ejecting anything from the end will give it forward thrust. Obviously, the faster you can eject material, such as burning rocket fuel, the more thrust is provided. StuRat (talk) 17:14, 8 March 2010 (UTC)[reply]
Think about it like this: Suppose there were two bowling balls out there in space with a big compressed spring between them. When the spring un-springs, the two balls both fly off in opposite directions - right? It wouldn't make sense for one ball to stay still and the other one to fly off into the distance. Well, in a rocket engine, the exhaust gasses are one bowling ball, the rocketship is the other and the hot expanding gasses inside the motor are the spring. The fuel acts both as the "reaction mass" (the second bowling ball) and the force that causes that mass to fly off in one direction and the rocketship to head off in the other. SteveBaker (talk) 03:54, 9 March 2010 (UTC)[reply]
Nice analogy Steve Stanstaple (talk) 19:05, 9 March 2010 (UTC)[reply]
My favorite analogy for understanding this idea: let's say you're on a little rowboat, and you're holding a brick, and you throw the brick out the back of the boat as far as you can.
What happens to the boat? It goes forward a little. Is it because the brick has something to push against (the air)? No, it's because *you* -- and the boat, which you're attached to through your butt -- were pushing against the brick!
Now keep a pile of bricks on your boat, and throw them all out the back, and that's your thruster. You can now boldly go to the other side of the lake. --Sean 19:53, 9 March 2010 (UTC)[reply]
It's something I puzzled over as a child. I used to imagine sitting in the back of a van and trying to propel myself be throwing stones out the back. I wondered whether I needed to hit the stones of the ground to get the 'push', and if so, how did the van know whether I hit the ground? The spring bound bowling balls released idea is really satisfying for some reason. Stanstaple (talk) 20:08, 9 March 2010 (UTC)[reply]

Happiness

I was watching the simpsons and there was a chart which showed that less intelligent people are happier. Is there any evidence to support this claim? Any studies? —Preceding unsigned comment added by Doorelore (talkcontribs) 11:56, 8 March 2010 (UTC)[reply]

If you search Pubmed there are plenty of studies which correlate intelligence with suicide rates and similar ideas, which may be relevant. Unfortunately, I don't have access to these. Perhaps someone else will :) Regards, --—Cyclonenim | Chat  12:17, 8 March 2010 (UTC)[reply]
I think someone once said 'Ignorance is bliss'. Is ignorance the same as lack of intelligence? —Preceding unsigned comment added by 79.76.232.131 (talk) 15:08, 8 March 2010 (UTC)[reply]
I would say ignorance was the lack of knowledge, rather than the lack of intelligence. --Tango (talk) 17:16, 8 March 2010 (UTC)[reply]
I would expect the correlation between intelligence and happiness to depend greatly on the culture and time period. That is, do we reward or punish stupidity and intelligence ? In a primitive society, the stupid aren't likely to survive, or, if they do, barely get by. In more developed societies they may have all their basic needs met and may also be legally protected from those who would cheat them. As for intelligence, this was far more valued by the US, during the Space Race, when engineers and scientists were needed to beat the Soviet Union. Now the intelligent are often ridiculed as "nerds", as in the TV show The Big Bang Theory. There have even been cultures in which the intelligent have been exterminated, as in Cambodia under Pol Pot. StuRat (talk) 17:00, 8 March 2010 (UTC)[reply]
Matt Groening, in addition to creating The Simpsons, touched upon this topic several times in Life In Hell. I can't find it online, but one strip featured — and I'm sure I've piled on inaccurate details, but you'll get the idea — one relatively happy guy watching TV, compared with a morose grad student, speaking about Nietzsche and the futility of existence and the inexorable approach of death, followed by total nothingness until the heat death of the universe; and the caption was, "Which one had too much education?" Anyway, sorry, I don't have a reference to a study about this. It is WP:OR on the part of Matt Groening. Comet Tuttle (talk) 18:58, 8 March 2010 (UTC)[reply]
I think I would be much happier if I grew up stupid in some working class family, than I am now. I would never have questioned and left my religion, I would never suddenly descend into worries about how meaningless life is, I would never suddenly start wondering about how did the universe get here in the first place, if not by supernatural powers, etc. And StuRat, nerds are ridiculued as nerds, but most intelligent people aren't regarded as nerds. If you I guess it's because you work in a computer proffession and wear glasses. Well that's sickening, maybe get contacts?--92.251.221.135 (talk) 21:42, 8 March 2010 (UTC)[reply]
You'd be hard-pressed to find a current US TV show where physicists are shown to be cool. And, incidentally, I do wear contacts. StuRat (talk) 22:01, 8 March 2010 (UTC)[reply]
Strange, over here (Ireland) there are lots of programs advocating science. Most of them seem to be BBC though.--92.251.221.135 (talk) 23:18, 8 March 2010 (UTC)[reply]
We have science documentaries here, mainly on PBS, but a sitcom portraying math and science experts in a positive way ? No. StuRat (talk) 02:13, 9 March 2010 (UTC)[reply]
Please remember that they aren't just physicists on The Big Bang Theory, they are nerds. They speak Klingon, go to Comicon and spend probably thousands of dollars on comic books. Those traits seem to be more coupled with social ineptness, not intelligence per se (which many real people seem to appreciate). --Mark PEA (talk) 19:03, 9 March 2010 (UTC)[reply]
It's not a sitcom and she is female, which makes a difference, but Samantha Carter is pretty cool. Daniel only gets cool later on and McKay is not cool, but at least there is one! --Tango (talk) 04:19, 9 March 2010 (UTC)[reply]
This is most likely going to vary between time and place, as StuRat stated. However, it would seem that if one were more intelligent, they would more effectively deal with problems in their life, and thus more likely than not be happier. 66.69.254.68 (talk) 04:39, 9 March 2010 (UTC)[reply]
Happiness is a cigar called Hamlet —Preceding unsigned comment added by 79.76.188.14 (talk) 23:14, 9 March 2010 (UTC)[reply]

Artificial diamond

How is artificial diamond made? -- Extra999 (talk) 18:10, 8 March 2010 (UTC)[reply]

http://en.wikipedia.org/wiki/Artificial_diamond#Manufacturing_technologies —Preceding unsigned comment added by 157.193.173.205 (talk) 13:12, 8 March 2010 (UTC)[reply]

What is meant by decreasing the "vacuum"???

What is meant by decreasing the vacuum?? Is it increasing the vacuum pressure hence decreasing the quality of it?? Or decreasing the emptiness??? —Preceding unsigned comment added by 119.235.54.67 (talk) 13:10, 8 March 2010 (UTC)[reply]

It depends entirely on the speaker who makes that comment. This is identical to the vagueness over "decrease the air conditioning." It could mean "decrease the amount of cold air produced by the air conditioner to make it warmer" or it could mean "decrease the temperature setting on the air conditioner to make it colder." -- kainaw 13:38, 8 March 2010 (UTC)[reply]

Seasonal fruits

Is it possible to fool plants into growing and fruiting at the wrong time of year, using environmental control and temperature etc? Or do plants somehow know if it's the wrong time of year even if the conditions are perfect for growth in their artificial environment? —Preceding unsigned comment added by Techcolis (talkcontribs) 13:45, 8 March 2010 (UTC)[reply]

You can certainly trick the plants. Mostly it takes the right temperatures, lighting and water. If the plant has that, it will be quite happy. For fruiting, you might need to artificially pollinate since the natural pollinators (usually wind or insects) might not be present in the artificial environment. Googlemeister (talk) 14:22, 8 March 2010 (UTC)[reply]
I believe polytunnels are used for this purpose to grow strawberries year-round. They don't taste as nice, though. Vimescarrot (talk) 19:23, 8 March 2010 (UTC)[reply]
To be pedantic, I don't think that's much of a trick at all. Seasonal could be taken to refer to seasonal conditions. DRosenbach (Talk | Contribs) 19:58, 8 March 2010 (UTC)[reply]
From what I was told years ago, this is done all the time with poinsettia plants to make them blossom near Christmas. Dismas|(talk) 00:37, 9 March 2010 (UTC)[reply]

English name of an object

What is the English name of the green object that you see here? It need to put fuel on the vehicles. Do you know if there is a similar image on Commons? --Aushulz (talk) 15:39, 8 March 2010 (UTC)[reply]

There has to be a better word, but the only thing coming to mind at the moment is "nozzle". Falconusp t c 15:43, 8 March 2010 (UTC)[reply]
According to gas pump it is indeed called a "nozzle." Falconusp t c 15:52, 8 March 2010 (UTC)[reply]
(ec) Agree with "nozzle" or "fuel nozzle" - most would understand this. U.S. fuel pump labeling generally refers to it as the "handle" (as in "Remove handle and begin fueling"), but that's not specific enough out of context to be useful. -- Scray (talk) 15:54, 8 March 2010 (UTC)[reply]
(UK) I've never heard it referred to as anything other than the (fuel) nozzle. —Preceding unsigned comment added by 131.111.248.99 (talk) 16:22, 8 March 2010 (UTC)[reply]

UNLESS, of course, the OP is referring specifically to only the green plastic part of the handle assembly, and not all the metal parts? DaHorsesMouth (talk) 03:11, 9 March 2010 (UTC)[reply]

On commons there is an image for the hydrogen fueling nozzle, but I am searching one image of gasoline one.
@DaHorsesMouth: I am referring to the green part and the metal part, too, so the object to take fuel except the tube. --Aushulz (talk) 14:46, 9 March 2010 (UTC)[reply]
I added my request here. Thank you everybody. :) --Aushulz (talk) 14:48, 9 March 2010 (UTC)[reply]

Charting unknown part of space

In this video, at 3:08, why is it unknown part of the universe in a Bow tie shape? --Reticuli88 (talk) 16:49, 8 March 2010 (UTC)[reply]

Probably because it's blocked by something with a bow tie shape, from Earth's POV, such as the most dense portion of the Milky Way galaxy. StuRat (talk) 17:18, 8 March 2010 (UTC)[reply]

adaptaion of bones to our environment

how do bones adapt to our environment —Preceding unsigned comment added by Chibu86 (talkcontribs) 18:09, 8 March 2010 (UTC)[reply]

They don't really. Bones are essentially the same in all humans, whatever environment they grew up/live in. Do you mean how have bones adapted to our environment, as the human race evolved? --Tango (talk) 18:13, 8 March 2010 (UTC)[reply]

You will fail if you turn in that answer. Bones vary greatly according to the environment, lifestyles, and diet of all humans. Bones remodel constantly and adapt to the environment in several important ways. Some examples: 1. One of the most obvious, if exotic, is the steady loss of bone mass in a weightless environment. This is currently a substantial limit to the time humans can spend in space. 2. Bones remodel in response to stress and use. The bones of a person who gets little exercise or physical work are lighter and weaker. Muscular activity is an important factor in maintenance of bone mineralization. 3. A new scientific frontier is the role of bones in regulating energy metabolism (i.e., calorie intake, expenditure, and adiposity). Here is an example [18] 4. Bones change shape in response to environmental shortages of minerals or vitamin D. This reshaping is called rickets. How many more do you need, or have i just done your homework for you? alteripse (talk) 18:37, 8 March 2010 (UTC)[reply]

Can you be more precise in your question?
If you carry out heavy labour ( or weight-lifting/training etc.) then those bone that carry the load will thicken to compensate or in zero gravity de- mineralize. Is that what you mean? --Aspro (talk) 18:31, 8 March 2010 (UTC)[reply]
In concert with the above stated answers save for the first, bones certainly adapt -- see functional matrix hypothesis and wolff's law. DRosenbach (Talk | Contribs) 19:55, 8 March 2010 (UTC)[reply]
Your second point is good, although I'm not sure I would count behaviour as environmental (although I suppose it is affected by environment). Your other points don't sound like adaptations to me, just changes. A change needs to be beneficial in some way to count as an adaptation, in my book. --Tango (talk) 22:59, 8 March 2010 (UTC)[reply]
An adaptation is a controlled, "active" response to an environmental change. All 4 examples are such; none is simply "damage suffered passively". Whether in some combination of circumstances the adaptation has more negative than positive questions does not make it not an adaptation. Broaden your understanding of the word. alteripse (talk) 02:03, 9 March 2010 (UTC)[reply]
The one definition in wikt:adaptation that doesn't just reference another word concurs with my understanding of the word. --Tango (talk) 04:23, 9 March 2010 (UTC)[reply]
Adjustment to extant conditions: as, adjustment of a sense organ to the intensity or quality of stimulation; modification of some thing or its parts that makes it more fit for existence under the conditions of its current environment Thanks for the link. I assume this is the def you were using. I agree completely. It was the only def that made any sense in the context. I provided 4 examples of adjustment by bone to conditions. Not sure I understand your difficulty with this concept. alteripse (talk) 11:33, 9 March 2010 (UTC)[reply]
It's perhaps worth remembering that if we're discussing biology here, acclimatisation is probably the better term to use since it sounds like were discussing changes in an individual in response to the environment, not adaptation by natural selection of populations Nil Einne (talk) 12:21, 9 March 2010 (UTC)[reply]
Looks like we're getting into a semantic argument here. FWIW, the use of the term "adaptation" to describe physiologic changes to environmental conditions is perfectly valid (see light adaptation for a widely accepted example). I think Alteripse gave a good answer in terms of examples of how bones can be altered by environmental influences. However, if the OP is interested in the mechanistic "how", as in "how is bone remodeling accomplished?" then the articles on osteoclast and osteoblast will provide some insight. --- Medical geneticist (talk) 14:10, 9 March 2010 (UTC)[reply]
Just to be clear, I wasn't saying the term is invalid but that (in my experience which is primarily in molecular biology and genetics area) the use of the term in the not evolutionary context is often avoided in biology for clarity and because of the confusion it can cause. In medicine and physiology, things may be different, I can't speak to those areas (hence why I specifically mentioned biology). Light adaptation is perhaps not the best example, since you're discussing something that occurs primarily in minutes (human adaptation to spaceflight which is partially relevant here would be a better example). I would note adaptive system says something similar although it's unsourced and the whole article is in a rather poor state. Of course asking how something adapts, is different from calling it an adaptation, the later is the only one I would avoid. Of course acclimitisation isn't perfect either since it seems to suggest climate rather then generic environmental changes and adaptation is widely used when it comes to bones Nil Einne (talk) 21:11, 9 March 2010 (UTC)[reply]

Truth or myth about rebuilding the brain?

«we are learning that hard exercise also stimulates increases in neurotransmitters, like serotonin, norepinephrine and dopamine, all of which we worry about in all of the above maladies. But the research he details in Spark shows further effects on the whole range of biochemistry beneficial to well-being and behavior. Some of these chemicals literally rebuild the brain.»

is that that easy?--Mr.K. (talk) 18:59, 8 March 2010 (UTC)[reply]

"Analogy is the core of cognition," but this is an example of a relatively simplistic description of the process of brain remodelling that occurs continuously throughout life. Not sure what you are asking. alteripse (talk) 19:08, 8 March 2010 (UTC)[reply]
Agreed. There is certainly some rebuilding of the brain, as in all our other organs, but it's not 100% in all cases, of course. StuRat (talk) 19:11, 8 March 2010 (UTC)[reply]
There is some science behind this, but most popular accounts way overstate the story. We have an article, brain fitness, that's supposed to be about this topic, but unfortunately it's a sucky article. Looie496 (talk) 22:48, 8 March 2010 (UTC)[reply]
Well, they can't "literally" rebuild the brain. They might improve it's functioning or something - but they could only possibly "figuratively" rebuild it. But think about this - would you really want your brain "literally" rebuilt? Doing that would erase all of your memories, wipe your personality, delete all of your learned skills. You might as well die! SteveBaker (talk) 03:15, 9 March 2010 (UTC)[reply]

So how did we all get here in the first place?

Why did the big bang happen? What caused it? Why did that exist? Why doesn't nothing exist? Doesn't this pretty much discredit atheists?--92.251.221.135 (talk) 21:49, 8 March 2010 (UTC)[reply]

I'm not sure how questions can discredit a philosophical statement. Normally evidence is required to discredit something. that being said we can address the earlier questions :
  • Why did it happen? - Why not?
  • What caused it? - Big Bang has a fairly decent description, but the short answer is either "we don't know" or that question might not have meaning.
  • Why did that exist? - This seems to be a statement saying that teh creator of the creator (big bang) had to exist. The counter-argument athesists would present is, if everything needs to have been made by something, then what made "God"?
  • Why doesn't nothing exist? - Maybe it did at some point, but noone was there to ask that.
Chris M. (talk) 22:00, 8 March 2010 (UTC)[reply]
(edit conflict) We don't know, we don't know, we don't know, we don't know, and no. Why would it discredit atheists? I've always seen it as more of an argument for discrediting theists, since it doesn't match up with any creation story in anything much more than a metaphorical sense. Vimescarrot (talk) 22:01, 8 March 2010 (UTC)[reply]
It seems to me that it would very much discredit atheists, because, is it the First law of thermodynamics, that states that matter and energy cannot be taken out of nowhere. --The High Fin Sperm Whale 22:16, 8 March 2010 (UTC)[reply]
No one said anything about anything coming out of nowhere. Beach drifter (talk) 22:35, 8 March 2010 (UTC)[reply]
  • A) Not knowing where the Bigbang came from doesn't mean it came from nowhere.
  • B) Conservation of energy is a consequence of time homogeneity. If time started at the Bigbang than there cannot be time homogeneity at the instant of the Bigbang because it is a boundary. Therefore the Bigbang is not required to obey conservation of energy.
  • C) As far as we know the total energy of the universe might be exactly zero.
  • D) Within the framework of general relativity time homogeneity is not really required at the scale of the universe as a whole.
  • E) God coming from nowhere is a problem for theists as well.
Any of the answers above make your point moot. Dauto (talk) 22:57, 8 March 2010 (UTC)[reply]
The Fist Law of Thermodynamics really only makes sense in very few possible spacetimes. It is meaningless in a space that is expanding or contracting (see ADM mass, General Theory of Relativity). 74.14.109.18 (talk) 22:52, 8 March 2010 (UTC)[reply]
You have the exact same questions with or without God. That is "Who created God ?", or, if you assume that God always existed, why not skip the middleman and assume that the universe always existed (maybe in the oscillating universe mode). StuRat (talk) 21:57, 8 March 2010 (UTC)[reply]
Except God would be spiritual, not physical, so laws of physics would not apply. --The High Fin Sperm Whale 22:43, 8 March 2010 (UTC)[reply]
If God does not have a physical component, then how can he interact with the physical universe? If he does have a physical component, then how did that physical component arise? —Bkell (talk) 23:39, 8 March 2010 (UTC)[reply]

I don't believe in God. Just sayin you know, how did we get here. Nothing can just come from nowhere, so how did we come from nowhere?--92.251.221.135 (talk) 23:13, 8 March 2010 (UTC)[reply]

As Beach drifter and Dauto wrote above, the Big Bang theory doesn't say it "came from nowhere". The little dense point of matter and energy may have been around for 92842049820958 quadrillion years, or infinite years, or there may have been no movement of time at all before the Big Bang — we do not know. Comet Tuttle (talk) 23:19, 8 March 2010 (UTC)[reply]
Yes but why did it exist?--92.251.221.135 (talk) 23:21, 8 March 2010 (UTC)[reply]
The short answer is that we don't know. However, the other side as long as you are making comparisons to atheists is that the same problem exists for any other notion (such as a deity) just as much. An eternal chain of causation going backwards is logically consistent if a bit counterintuitive. No matter how far you go back you will always be able to ask "so why that?" JoshuaZ (talk) 23:24, 8 March 2010 (UTC)[reply]
The reason I mentioned atheists and not theists is because theists can argue that well their God is supernatural why does he have to have a begining?--92.251.221.135 (talk) 23:25, 8 March 2010 (UTC)[reply]
But did you honestly expect an answer to any of this? Beach drifter (talk) 23:35, 8 March 2010 (UTC)[reply]
Of course not==92.251.221.135 (talk) 23:35, 8 March 2010 (UTC)[reply]
I created the world. Bus stop (talk) 23:47, 8 March 2010 (UTC)[reply]
May be you didn't expect an answer, but I gave you five answers. Dauto (talk) 23:48, 8 March 2010 (UTC)[reply]
Actually, one of the attractive things about the Big Bang as an explanation is that it offers the possibility to cut off that eternal regress of "What created the thing that created the thing that created the Big Bang?" - the idea that time itself was also created in the Big Bang would means that there was no "Before" - problem solved. Or the idea that two universes were created - one with time running backwards - and ours with time running forwards...again, no "before". We don't (yet) know that either of those things are true - but if they are then that's it. Origins problem solved! Any "God" explanation really begs the question "Where did God come from?" and if you answer "He just is" or "He always was" we have to slap you! When science says that the big ban "Just is" or "Always was" - we're told it's an inadequate answer - but when cross out "Big Bang" and write "God" instead - suddenly that's an OK answer for the religious guys. I don't get it! SteveBaker (talk) 03:08, 9 March 2010 (UTC)[reply]
"There was no before" doesn't solve the problem "why was there an after?". Is there a theoretical difficulty with nothing at all ever existing? It doesn't sound very problematic, on the face of it. 81.131.20.244 (talk) 00:40, 10 March 2010 (UTC)[reply]
"We don't know - so it must be God." is a deeply unsatisfying answer. Why not "We don't know - so it must have been created by a mad scientist from the year 3010 who had a terrible accident with his time machine" ? Or "We don't know - so it must have been Harry Potter who magiked into existance because HE'S REAL!!!", or "We don't know - but we strongly suspect it has to do with the nature of singularities." - all of those are every bit as valid as "...it must be God". So long as we don't know - it's really better not to make any assumptions whatever about what that means. It would be different if there was evidence of God or Gods doing other things in other parts of the physical world - but there isn't...not a scrap. Since there are a literal infinity of other possible explanations - we have to say "We don't know." and mean it. SteveBaker (talk) 02:45, 9 March 2010 (UTC)[reply]
I'm sorry no one else here has given you the correct answer. It all happened Last Thursday, actually, although some will try to convince you that it was just five minutes ago. Imagine Reason (talk) 12:41, 9 March 2010 (UTC)[reply]
If nothing existed, then it would continue to be nothing until something came along. Something came along (you're reading this), and being a stable something, it now it remains. The most relevant article I know of is Many worlds hypothesis, but it's rather bunged up with scientific abstractions. The final episode of Star Trek: The Next Generation, All Good Things... explores the concepts of being and non-being rather more accessibly. Vranak (talk) 18:29, 9 March 2010 (UTC)[reply]
Where do photons come from after spontaneous emission? Also, don't virtual particles spontaneously appear in vacuum without any prior cause? --Mark PEA (talk) 18:53, 9 March 2010 (UTC)[reply]
If this is addressed to me, I have no idea. Although I believe there are allowances made for the spontaneous creation of matter (or was it energy) on the rim of black hole. Little help? :) Vranak (talk) 20:27, 9 March 2010 (UTC) [reply]

White Sussex hen

[Query moved from Talk:Chicken#White Sussex]

I have a couple of White Sussex and one sits in her coup. She use to lay daily, now she has stopped and sits in her nest. I think she thinks she has chickens —Preceding unsigned comment added by 121.217.45.250 (talk) 01:40, 8 March 2010 (UTC)[reply]

She could be broody (wanting to sit on eggs to hatch them). If so, she'll flop back onto the nest when you lift her up, often making a long, low squawk, fluffing her feathers and pecking at your hand. If taken off the nest she'll cluck about low to the ground and fluff her feathers, or even flop onto the ground and brood there.
You could give her a dozen eggs and let her hatch them. They must of course be fertile, so you need eggs from hens who've been with a male, or if you don't have a male, you'll have to buy hatching eggs. All the eggs must go under her at once, so they hatch together (if she's sat on eggs for different lengths of time, it may be best to discard them and start again). If it takes you a week or so to get a clutch together you can keep her sitting by giving her golf balls. Usually broodies will only sit in the nest they have chosen, but sometimes you can move them to a more convenient place – if you do, put her in a dark, secluded place where she can't easily get out (I use a large bucket with hay in, and a board over the top). If other hens can get at the nest, mark her hatching eggs and keep collecting any new ones from the other hens, or they'll start to develop too. Put her off the nest every couple of days for ten minutes or so to eat and drink and do a dropping, or she may foul the nest. If she does, or if a broken egg contaminates the others, wash the dirty eggs in quite hot (not warm) clean water and dry them carefully, and some may survive if you're lucky (a bit grubby is OK: they only need cleaning if coated with muck). If you smell bad egg in the nest, sniff each egg carefully until you find the off one, then remove it, or it will burst and spoil everything.
The eggs will take 21 days to hatch from when they go under her. When they do hatch, let her brood the chicks on the nest, then after a couple of days remove any unhatched eggs, or move her and the chicks elsewhere (once they're cheeping she'll stick with them wherever they are). Feed them moistened poultry feed, which she will show to the chicks, and water of course; the food only has to be moistened for the first few days. Best not to feed layers' feed to chicks for any length of time, as it has too much calcium which can mess up the development of their joints; you don't however need the special chick food they sell, just ordinary growers' feed. Once running about they can go outside – she'll brood them to keep them warm, shelter them from rain and sun, and defend them valiantly from cats, dogs and any other passers-by. Keep them separate from the other hens at night in a safe corner or hutch until they're well feathered. The hen will keep the chicks together, but she will not always notice the one at the back, so don't let her explore long grass or piles of junk, as a lost chick will get cold or be eaten.
Alternatively if you don't want her to sit or can't get any fertile eggs, put her in an airy, well-lit, wire-floored cage off the ground until she goes off the idea (it may take a week or so).
Good luck! Richard New Forest (talk) 23:45, 8 March 2010 (UTC)[reply]
I think it might be a good idea to move this back to the article talk page - we're not allowed to give medical (including veterinary) advice here. Tevildo (talk) 23:59, 8 March 2010 (UTC)[reply]
I don't think this is a medical advice question. Comet Tuttle (talk) 00:02, 9 March 2010 (UTC)[reply]
Anytime you can eat your mistakes, it's just dietary advice. :) Franamax (talk) 00:23, 9 March 2010 (UTC)[reply]
It is an excellent answer and corresponds to my experience. It is animal husbandry, and not "veterinary advice." It is akin to advising how to grow tomatoes. My significant other says "It sounds like poetry." Edison (talk) 02:27, 9 March 2010 (UTC)[reply]

Yellow hands

A young girl I know has a peculiar yellow tint to her hands, and I'm wondering what it could be. She is east Asian, but there is a distinct almost "stained-yellow" color to her hands, as if she was soaking them in a brown ink every day. She is healthy, I was thinking it might just be increased carotene? Any ideas? 210.153.207.42 (talk) 00:58, 9 March 2010 (UTC)[reply]

I should note that only her hands are visibly colored. 210.153.207.42 (talk) 01:00, 9 March 2010 (UTC)[reply]

Saffron is that color, but I'm not aware of anyone dying their hands with it intentionally, although I suppose it could bleed into their hands from preparing foods with saffron that require lots of hand manipulation. StuRat (talk) 01:58, 9 March 2010 (UTC)[reply]
It's possible, though her hands have been like that (as far as my noticing goes) for at least a year. Maybe she really likes paella. 210.153.207.42 (talk) 02:43, 9 March 2010 (UTC)[reply]
The color you describe sounds a little like Henna. Paul Stansifer 03:14, 9 March 2010 (UTC)[reply]
Yes, to be honest I thought the same thing, but again I don't understand why it would last so long, nor why she would be continuously using henna! I was actually under the impression that it was likely to be an internal thing, which is why I mentioned carotene. It does look like the skin itself is colored, though I can't tell why it would only coloring the hands. 210.153.207.42 (talk) 03:58, 9 March 2010 (UTC)[reply]
There's certainly precedent for people having very yellow skin as a side-effect of eating much too much carotene, so that's a reasonable thought - but it still seems odd for it to be restricted only to the hands. ~ mazca talk 09:05, 9 March 2010 (UTC)[reply]
Mind you, it's not like I've seen her whole body. Her legs and face aren't yellow though. 210.254.117.185 (talk) 10:31, 9 March 2010 (UTC)[reply]
Turmeric is a pretty good yellow dye. It's dyed several of my pots and utensils and cups. It's common in SE Asian cooking, and is reputedly good for your skin. Indeterminate (talk) 05:21, 9 March 2010 (UTC)[reply]
Tobacco can stain fingers yellow, but that wouldn't usually be the whole hands. --Tango (talk) 05:38, 9 March 2010 (UTC)[reply]
I now have a great mental image of a small girl chain-smoking huge cigars held in her fist. ~ mazca talk 09:05, 9 March 2010 (UTC)[reply]

Localized jaundice. I have this, on my face and hands. —Preceding unsigned comment added by Fire2010 (talkcontribs) 19:50, 9 March 2010 (UTC)[reply]

Handling nitroglycerine or cordite? —Preceding unsigned comment added by 79.76.188.14 (talk) 23:17, 9 March 2010 (UTC)[reply]
How young is the girl? I've seen it more then a couple of times when girls have strangely yellowed hands and it was my fiancé's younger sister who gave me the answer. She was only 15 and started using this "sun cream" which was not only sun block but also "fake tan" in one bottle. Since you use your hands to apply it, your hands soak up a lion's share of the ointment, if you don't wash them immediately, which girls never do since girls never read instructions, especially 15 year old ones. ;) I'm not guaranteeing this is the case here, but everything you've said so far seems to fit with this theory.Vespine (talk) 00:46, 10 March 2010 (UTC)[reply]

plastic? it breaks weird

is the window in this

http://www.youtube.com/watch?v=PH5OMyuAppg


plastic? it breaks weird —Preceding unsigned comment added by Thekiller35789 (talkcontribs) 01:09, 9 March 2010 (UTC)[reply]

Yes - it certainly looks like plastic of some kind. When you hit a piece of glass, it's so brittle that cracks propagate outwards to the edges of the pane. But plastic can bend enough to stop cracks from propagating - so you get those kind of rounded chunks falling out. Also, you can see the intact panes bending in the wind. It's hard to be sure though. SteveBaker (talk) 02:26, 9 March 2010 (UTC)[reply]
They look like storm windows to me. They are commonly plastic on the outside and (usually) placed over glass on the inside. If there was glass on the inside, you wouldn't see it after the axe went through. Some people put up storm windows without real windows behind them. So that could be the case here. -- kainaw 06:25, 9 March 2010 (UTC)[reply]
Comments on YouTube suggest the window material is Plexiglass (Poly(methyl methacrylate)) but from its apparent flexibility I expect it is vinyl. Cuddlyable3 (talk) 21:52, 9 March 2010 (UTC)[reply]

plant growing

A technical? question about growing large plants. I assume that while more sunlight = more energy for the plants, and thus a brighter area can support a larger number of (non-shady) plants, too much sunlight evaporates too much water and thus there is a balance between the amount of sunlight a plant can get vs. the available water. If extra sunlight was redirected towards (not focused at) a large garden area, and care is taken to keep the ground cool/moist (the redirected light could be aimed parallel to the ground so as not to heat it directly), would this make the area more fertile? Am I thinking in the wrong direction? A large greenhouse isn't really feasible, and this can't really be a big expensive/mechanical operation. 210.153.207.42 (talk) 04:10, 9 March 2010 (UTC)[reply]

Sorry, but that just wouldn't work. The reason is is that the sun produces infrared waves (heat) along with the visible light. So even if the light is not focused, wherever you can see the sun's light, there will be heat. However, I believe the best thing to do would be to redirect the light at the plant's leaves, but not the ground. This way, the plant would take energy from the sun, but it would not dry up the soil. Hope this helps, --The High Fin Sperm Whale 04:20, 9 March 2010 (UTC)[reply]
Yeah, that's exactly what I was proposing, by aiming the reflected light parallel to the ground, rather than at the ground. 210.153.207.42 (talk) 05:06, 9 March 2010 (UTC)[reply]
Whether it will result in better plant growth will depend on whether it was sunlight that was the limiting factor before. It could be that they have plenty of sun and actually need more water, or more nutrients in the soil, or more physical space, etc. If one of those things is limiting the plants' growth then more sunlight won't help. --Tango (talk) 05:42, 9 March 2010 (UTC)[reply]
Thanks! What about in a more complex forest, with large tree growth, and smaller ferns and bushes underneath? I've seen a lot of Canadian shield forest (and badly planted artificial forest) where tall trees block out the sunlight leaving almost none for the undergrowth, so I'm thinking of a way to promote more plant variety. 210.153.207.42 (talk) 05:51, 9 March 2010 (UTC)[reply]
And also, what is the limiting factor for space? If there is plenty of sunlight/nutrients won't plants be able to grow around each other? 210.153.207.42 (talk) 05:55, 9 March 2010 (UTC)[reply]
You may be interested in Forest gardening. In biological systems, something may be held back by a limiting factor, but once that limiting factor is overcome then another limiting factor is in control. Difficult to know what the limiting factor may be, it could easily not be lack of sunlight. I take it you are in a hot dry area. You could consider trickle or Drip irrigation or mulching to conserve water. It may be simply that there is not enough diversity of seeds for what you want, or they are dormant for lack of water, or that fertilizer would help. I would consider scattering a variety of seeds, possibly with fertilizer, and supplying water. Too much sunlight may scorch the plants. Many plants seem to like moist warmth, but what is best will vary from species to species. You could consider a polytunnel, perhaps home-made, but perhaps your plants will wilt in the heat. For large plants it is better and quicker but more expensive to buy them already grown from nurseries. The polytunnel article only mentions small ones, but I was thinking of ones seven or eight feet in diameter, which are quite common where I am. 78.146.202.143 (talk) 19:59, 9 March 2010 (UTC)[reply]

price wiki

Is there a wiki where one can enter the price, date, store location and UPC or PLC of the item and that tracks price changes and/or differences over time and location? 71.100.11.118 (talk) 05:57, 9 March 2010 (UTC)[reply]

I don't know about a Wiki, but there are many price comparison websites on the net. —Preceding unsigned comment added by Fire2010 (talkcontribs) 19:48, 9 March 2010 (UTC)[reply]

March 9

Name of an "larva"

Resolved

Hi, about six or seven years ago my sister received a present that was like an aquarium for insects. Basically, it was like a plastic pool that you filled with water and then there was a packet of dried "insects" that, when applied to water, would "hatch" and eventually start swimming around. I remember the box saying that those "insects" (or "larvae" or whatever) were "millions of years old" but that could've been an exaggeration to boost sales. From my memory, the older ones had tails. Does anyone know the name of the insect/amphibian? 144.124.16.28 (talk) 07:47, 9 March 2010 (UTC)[reply]

Sea monkeys. Stupid me. Thanks anyways. 144.124.16.28 (talk) 07:53, 9 March 2010 (UTC)[reply]
That's the marketing name. They are typically really brine shrimp. I will mark this Q as resolved. StuRat (talk) 15:00, 9 March 2010 (UTC)[reply]

Mobile phones on airplanes

I have to turn off my mobile phone when on an airplane because it intefers with the plane's navigation system - But while it's turned off, I can receive SMS and also calls to my mailbox - why don't these signals cause problems for the airplane too? Thanks for info, --AlexSuricata (talk) 12:50, 9 March 2010 (UTC)[reply]

They do cause the same issues (to the extent that there are any real issues of concern - entirely another debate). What sort of phone do you have that when turned off, it is still on? Mine, fwiw, is very much off when it is off. --Tagishsimon (talk) 12:57, 9 March 2010 (UTC)[reply]
(ec) I'm pretty sure that, when your phone is off, it doesn't receive anything; messages get queued up by the phone company for delivery when you first turn it on. This is why there's some delay between turning the phone on and getting messages. Paul Stansifer 12:59, 9 March 2010 (UTC)[reply]
And as for the voicemail messages, those are not stored on your phone at all - if you do not answer your phone, the call gets directed to your mailbox in your cellular carrier's system. When you turn your phone back on, you're simply receiving the notification that there are more messages in said mailbox than there were before. Text messages (SMS) are stored on your phone itself, but they queue in your carrier's SMS delivery system while your phone is not connected to the network, such as when it is off or in an area with no service. They are released when your phone next makes contact with the network. Coreycubed (talk) 14:48, 9 March 2010 (UTC)[reply]
I've always suspected that the ban on cell phones in the plane was more about not annoying other passengers than safety. However, people are less likely to violate the ban if they think the plane will crash in flames when they turn it on. StuRat (talk) 14:56, 9 March 2010 (UTC)[reply]
I did not turn my phone off the last time I was flying, and noticed the battery drains really fast. Is that because the phone is moving to a different cell tower every 30 seconds? Googlemeister (talk) 15:06, 9 March 2010 (UTC)[reply]
It's for the same reason that it drains faster out in the middle of nowhere...it has to struggle to find reception...weak reception/searching for service kills phone batteries, and there's not much service 40,000 feet (12,000 m) in the air. Ks0stm (TCG) 15:13, 9 March 2010 (UTC)[reply]
Obligatory WP:WHAAOE link: Mobile phones on aircraft -- Coneslayer (talk) 15:19, 9 March 2010 (UTC)[reply]
And since we unsuccessfully avoided Tasishsimon's "entirely another debate" statement, the real reason we aren't supposed to use portable electronic devices on aircraft is because they emit harmful pilot killer rays! Coreycubed (talk) 17:04, 9 March 2010 (UTC)[reply]

Half life zpe device for book

Hi guys

What would be a good alternative that is more realistic ( i say more, but still) way of acheiving the effects of the zpe gun in Halflife 2? A machine that if it existed would be Like mechanical telekinesis. I know this is not possible with technology ( at least not at the moment) but what forces would you need in real life to acheive this kind of thing. would electron manipulation work? I ask this because In real life even if you could manipulate zero point energy or vacuum energy it wouldnt be strong enough?

So are there any Ideas ?

R —Preceding unsigned comment added by 79.67.141.98 (talk) 14:26, 9 March 2010 (UTC)[reply]

For those not familiar with Half-life, perhaps you might summarize what the thing does? Here's what I'm reading into it: you point the gun at something -- say, a crate 100 feet away. The crate glows with cool blue light, and then by swiveling the gun, the crate moves, staying in line with the barrel. Then you release the trigger and the crate falls. Good enough? So, with that sort of thing, you've got to cancel mass. That's the simplest (in terms of linguistic expression) means of lifting the crate and moving it around on the end of a really long arm. And it can't be done. There's no "mostly realistic" or "more realistic" about it. Worse, it's the sort of tech that absolutely breaks a story unless you can set hard limits on it (F=ma has just become absurd, and that's pretty core to how everything works). So if you want to use it, fine, any technobabble will do. Oh, and it needs a power source. I recommend tritanium batteries. They're rechargeable, you know. — Lomn 14:39, 9 March 2010 (UTC)[reply]
Oh, and to borrow from Baseball Bugs' recent comment on a related subject, if the writer is smart, he won't waste much time explaining the weird tech, as explanations often do little but further open the door to rebuttal. I'd say the key exception is where the explanation reveals a plot point and/or closes a plot hole. For example, in The Mote in God's Eye Niven and Pournelle spend a fair bit of time on the theory behind the Alderson drive because limiting FTL travel is essential both to their presentation of how the universe is explored and to keeping the "sci" in "sci-fi" from floating straight out the airlock. — Lomn 14:52, 9 March 2010 (UTC)[reply]
There is no possibility of creating a real gravity gun -- it violates the principle of conservation of momentum. Looie496 (talk) 20:39, 9 March 2010 (UTC)[reply]
Not necessarily. Perhaps it transfers momentum to the ground. — DanielLC 21:32, 9 March 2010 (UTC)[reply]

nuclear power station sites

Since it will take at least 10 times the number of nuclear power stations the US now has to displace the current usage of fossil fuel how many sites have been evaluated and/or selected to become nuclear power stations since George W Bush told the American Congress that the US was addicted to oil? 71.100.11.118 (talk) 16:15, 9 March 2010 (UTC)[reply]

Nuclear_power_in_the_United_States#Resurgence may help to answer your factual questions. Please note that the Reference Desk is not the place to start a debate or advance a particular position. -- Coneslayer (talk) 16:25, 9 March 2010 (UTC)[reply]
Please note that I can not prevent the news media from reporting that Barack Obama was taking another look at nuclear energy or that George W Bush told the American Congress that America is addicted to oil, both of which are facts I could care less about debating but merely want to know in which what, if anything is being done in accordance with pub.ic statements made by American Presidents. You on the other hand might want to start an argument over what you have been told you can and can not do as a good little Wikipedia boy or girl while I could care less. 71.100.11.118 (talk) 16:42, 9 March 2010 (UTC)[reply]
You're welcome for the help. -- Coneslayer (talk) 16:45, 9 March 2010 (UTC)[reply]

The question in its distilled form is "How many sites have been evaluated to become nuclear power stations?" The answer is found in the article linked above; 26 applications received, 7 more expected. Six reactors have been ordered. We currently have 104 nuclear power plants. By your own math, we would need an additional thousand. No need for anything other than AGF here; myself, I could not care less. Coreycubed (talk) 16:51, 9 March 2010 (UTC)[reply]

From the time the first electric power central station went in service in 1882 until the 1970's, utilities had to scramble to add an average of 7% more generation each year, which means doubling it every 10 years (varying with the economy and local conditions). This was due at first to more and more homes and business getting wired for electricity, then with its increased use for more purposes. Saturation eventually occurred, and efficiency /conservation efforts helped to limit growth. Transmission lines were planned and generation sites were selected and purchased years ahead to allow for this growth. Larger and larger generator were installed until they were over 1000 megawatts each. When the growth of demand for electricity fell off in the 1970s, the new generations sites were deferred and ultimately many were cancelled. A utility might have planned for ultimately 4 1000 megawatt nuclear units at a site, but cut it back to 2, while some other 4 unit site was cancelled entirely. Where you see a 2 unit nuclear plant, the ultimate may have provided for 4 units. The cancelled sites are likely just rented out as farmland at present, but transmission lines rights of way may have been purchased years ago which provide for the lines to crisscross nearby to allow for a variety of outlets for the power. "Ultimate" or "many years in the future" future system plans are still in drawers at the electric utilities, which could provide for much more nuclear generation than was ever built, perhaps double the existing number of nukes at a given nuclear utility. Load flow studies were done which modelled loads several times as great as present, noting what generating and transmission would be needed. Present nukes are designed to operate without trying to follow the daily peaks and troughs of demand, but rather as "base load" units ideally running night and day at a constant output. Smaller fossil units and peakers accomodate the peak loads and can cycle up and down. Better, cheaper and higher capacity energy storage would let the base load units do their constant output thing while storage near the load centers helped accomodate peaks without overtaxing transmission lines or running fossil units. Certainly the public would yell "Not in my backyard," as they always did, if utilities announced plans to build 346 and 765 KV AC and 500 KV DC transmission lines and nuke stations with cooling lakes or cooling towers near their communities. The upside of a nuke plant is that the property taxes enrich the local schools and municipalities. The downside could be more Chernobyls if design and operation are not adequate. Edison (talk) 17:22, 9 March 2010 (UTC)[reply]

What's the average amount of caffeine in a single medium roasted coffee bean?

From what I know, the amount of caffeine is actually reduced when brewing and roasting a bean, but how much is the amount left in a single roasted bean before brewing? IllusionalForce (talk) 17:50, 9 March 2010 (UTC)[reply]

There is lots of data about mass of caffeine per serving and mass-fraction of caffine in a bean. Either of those would allow you to calculate it, all you need to know (and I don't know it) is the typical mass of a coffee bean. DMacks (talk) 18:59, 9 March 2010 (UTC)[reply]

Convert to mhtml

Moved to WP:RD/C.

Black is black?

Black is black because it reflects nothing - right? So how come I can see a sheen of reflected light from some black painted objects? —Preceding unsigned comment added by 79.76.188.14 (talk) 21:12, 9 March 2010 (UTC)[reply]

Because it's not quite black. — DanielLC 21:26, 9 March 2010 (UTC)[reply]
See the article on Specular reflection. It is quite difficult to get a truly black non-reflective surface but a school's black Chalkboard is close. If you paint it with a transparent shiny varnish it becomes shiny black which is still 'black' but specular reflections. Cuddlyable3 (talk) 21:30, 9 March 2010 (UTC)[reply]

Black is black because of a low level of diffuse reflection, not because of a low level of specular reflection -- in other words, it's quite possible to be shiny and still be black. The level is never zero, though, it's just a matter of degree. Most things that we call black emit more light if placed in bright sunlight than a white object will if placed in a dim room, in fact. Looie496 (talk) 21:43, 9 March 2010 (UTC)[reply]

magnetic field

How strong of a magnetic field would be needed to rip the iron out of a human's blood a'la X men 2? Googlemeister (talk) 21:32, 9 March 2010 (UTC)[reply]

If it can be done at all (and I'm rather skeptical), it would take a stronger field than humans have ever been able to generate. Iron in blood is paramagnetic (weakly) and not ferromagnetic like bulk iron. Dragons flight (talk) 22:35, 9 March 2010 (UTC)[reply]

Reaction type

What type of reaction is 2Na2O2 + 2H2O -> 4NaOH + O2 ? --70.250.214.164 (talk) 21:34, 9 March 2010 (UTC)[reply]

I believe that would be a displacement reaction. --The High Fin Sperm Whale 22:18, 9 March 2010 (UTC)[reply]
(Edit conflict with below) Or more generally, an Oxidation-Reduction reaction. The oxygen in the sodium peroxide has an oxidation state of -1. Some of it is oxidized to the 0 state in O2, while some of it is reduced to a -2 state in the hydroxide. I'm not sure I'd call this a displacement reaction at all, really. Buddy431 (talk) 22:48, 9 March 2010 (UTC)[reply]

Lorentz dilation?

Suppose a rod is moving at a speed v relative to an observer, with the rod oriented towards the direction of motion. In the observer's reference frame, the length of the rod can be determined by measuring the time Δt it takes for the rod to pass the observer. Then, L=vΔt. In the rod's reference frame, the time it takes for the observer to pass the rod, Δt0, can also be measured. Likewise, the length of the rod can be calculated by L0=vΔt0. We then have L0/Δt0=L/Δt. Substituting Δt=γΔt0, we arrive at the equation L=γL0. However, it should be L=(1/γ)L0. What have I done wrong? 173.179.59.66 (talk) 21:41, 9 March 2010 (UTC)[reply]

Wait, I think I got it: It should be Δt0=γΔt, right? 173.179.59.66 (talk) 22:17, 9 March 2010 (UTC)[reply]

When you write L=γL0, it means that Δt0=0, which means that you are considering two measurement events occuring at the same time in the (x0,t0) system, which means that the rod is moving in that system, which means that it is at rest in the (x,t) system, which is not the case.
So you should have L0=γL (and thus L=(1/γ)L0), with Δt=0, so the rod is indeed moving in the (x,t) system.
Now you take this equation Δt=γΔt0. This is valid for two measurement events satisfying Δx0=0, i.o.w. occuring at the same place in the (x0,t0) system, but you said that the rod is passing the (x,t) observer. That means that you need two events satisfying Δx=0, for which the transformation indeed produces Δt0=γΔt. DVdm (talk) 22:52, 9 March 2010 (UTC)[reply]

Nomogram construction

I have a nomogram of height & weight = body area. The scales are arranged in three vertical columns, the idea being that one takes the known value of height on the far left, aligns a straight edge to the weight on the far right, and the central point on the straight edge is the surface area on the central column.

What I am trying to do is rearrange the whole nomogram to a different layout - ideally a circular one, something with turning discs maybe, but I'm damned if I can come up with a way of working it and I've about decided that it's impossible. But I'd thought I'd ask you clever chaps...? FreeMorpheme (talk) 22:25, 9 March 2010 (UTC)[reply]

I'm not sure I get what you mean by a circular layout, but it seems like you basically want something like a circular slide rule -- since area equals length times width, you need a mechanical way of doing multiplication; a slide rule does that by adding logarithms. Looie496 (talk) 23:40, 9 March 2010 (UTC)[reply]

Properties of metals

I noticed that, although many metals have useful properties, they are balanced out by severe disadvantages. For example, iron is extremely strong, but it corrodes easily and falls apart, lead is malleable, ductile, and corrosion-resistant, but highly toxic, etc. Why is it that every metal with any practical use is "balanced out" by having some significant flaw? --70.250.214.164 (talk) 22:49, 9 March 2010 (UTC)[reply]

What about Stainless steel or titanium or aluminium? —Preceding unsigned comment added by 79.76.188.14 (talk) 23:02, 9 March 2010 (UTC)[reply]

Everything in the universe can be described as having some good characteristics but lacking others. You are asking about an aspect of how our brains categorize things, not properties of metals. alteripse (talk) 23:08, 9 March 2010 (UTC)[reply]

But there's no pure metal which has good properties with no significant disadvantages. --70.250.214.164 (talk) 23:19, 9 March 2010 (UTC)[reply]

God set it up that way in order to fool us into believing that He doesn't exist. Looie496 (talk) 23:35, 9 March 2010 (UTC)[reply]

Did you actually look up titanium? I cant find any disadvantages--79.76.188.14 (talk) 00:08, 10 March 2010 (UTC)[reply]
When powdered or shaved, titanium has a high chance of burning or exploding. --70.250.214.164 (talk) 00:13, 10 March 2010 (UTC)[reply]

crazy raccoon

http://www.youtube.com/watch?v=El_m4iitN5k&feature=related

i saw this with a bird too when i was a kid is that why they go in the middle of the street so they can get killed because there suffering to much and want to die i know its weard how i have said it —Preceding unsigned comment added by Thekiller35789 (talkcontribs) 00:04, 10 March 2010 (UTC)[reply]

It is not very likely that the animal in the video is attempting suicide-by-car. The consensus in the youtube discussion is that it was rabid, the symptoms of which include malaise, headache and fever, progressing to acute pain, violent movements, uncontrolled excitement, depression, and hydrophobia. I hazard they are more cogent reasons for the sort of behaviour seen on the video. --Tagishsimon (talk) 00:14, 10 March 2010 (UTC)[reply]